Download as pdf or txt
Download as pdf or txt
You are on page 1of 200

• Centre of Mass

• Rotational Dynamics
Copyright © reserved with Motion Edu. Pvt. Ltd. and Publications

All rights reserved. No part of this work herein should be reproduced or used either
graphically, electronically, mechanically or by recording, photocopying, taping, web
distributing or by storing in any form and retrieving without the prior written permission of
the publisher. Anybody violating this is liable to be legally prosecuted.

Corporate Head Office


394 - Rajeev Gandhi Nagar Kota, (Raj.)
Ph. No. : 08003899588, 0744-2209671
IVRS No : 0744-2439051, 52, 53,
www. motioniitjee.com , info@motioniitjee.com
CONTENTS

CENTRE OF MASS
THEORY AND EXERCISE BOOKLET
S.NO. TOPIC ......................... PAGE NO.

 THEORY WITH SOLVED EXAMPLES ................................................... 05 – 47


 EXERCISE - 1 (JEE MAIN) ............................................... 48 – 60
 EXERCISE - 2 (JEE ADVANCED-OBJECTIVE) ........................ 61 – 73
 EXERCISE - 3 (JEE ADVANCED) ........................................................ 74 – 82
 EXERCISE - 4 (PRIVIOUS YEAR QUESTIONS) ................................ 83 – 88
 ANSWER KEY ....................................................................................... 89 – 90

ROTATIONAL DYNAMICS
THEORY AND EXERCISE BOOKLET
S.NO. TOPIC .............. PAGE NO.

 THEORY WITH SOLVED EXAMPLES ................................................... 91 – 139


 EXERCISE - 1 (JEE MAIN) ............................................... 140 – 150
 EXERCISE - 2 (JEE ADVANCED-OBJECTIVE) ........................ 151 – 167
 EXERCISE - 3 (JEE ADVANCED) ........................................ 168 – 181
 EXERCISE - 4 (PRIVIOUS YEAR QUESTIONS) ....................... 182 – 197
 ANSWER KEY ....................................................................................... 198 – 200
Page # 4 CENTRE OF MASS

Syllabus
Centre of mass
Systems of particles; Centre of mass and its motion; Impulse;
Elastic and inelastic collisions.

Rotational Dynamics
Rigid body, moment of inertia, parallel and perpendicular axes theorems,
moment of inertia of uniform bodies with simple geometrical shapes;
Angular momentum; Torque; Conservation of angular momentum;
Dynamics of rigid bodies with fixed axis of rotation; Rolling without
slipping of rings, cylinders and spheres; Equilibrium of rigid bodies;
Collision of point masses with rigid bodies.

Corporate Head Office : Motion Education Pvt. Ltd., 394 - Rajeev Gandhi Nagar, Kota-5 (Raj.)
CENTRE OF MASS Page # 5

CENTRE OF MASS
1. CENTRE OF MASS :
Every physical system has associated with it a certain point whose motion characterises the motion of
the whole system. When the system moves under some external forces, then this point moves as if the
entire mass of the system is concentrated at this point and also the external force is applied at this
point for translational motion. This point is called the centre of mass of the system.

1.1 Centre of Mass of a System of ‘N’ Discrete Particles :


Consider a system of N point masses m1, m2, m3, .................... mn whose position vectors from origin
   
O are given by r1, r2 , r3 .............. rn respectively. Then the position vector of the centre of mass C of
the system is given by.
Y
. . . .. . .. . . .
. . . ..m...1 .. ..C. ..m..2
.. .. . . .. . . . . .. .. .
. . .. .. . . . r2.. .
 .. . . . . . .. . .m.
r1 .. . .. . .. .. .n
. rn
m

rc

O x
n


  
m r  m 2 r2 ...........mn rn 
m r
i 1
i i
 1
n

rcm  11
m1  m2 .........mn
; rcm  n
rcm 
M m r i i

m i
i 1

 i 1
where, mi ri is called the moment of mass of particle with respect to origin.

 n 
M  

 m 
i 1
i is the total mass of the system.

 
Further, r i  xi i  yi j  zik and r COM  xCOM i  y COM j  zCOMk
So, the cartesian co-ordinates of the COM will be
n

m x
i 1
i i
m1x1  m2 x2 ......mn xn
xCOM =  n
m1  m 2 .......mn
m i 1
i

or xCOM =
m x
i1
i i

M
n n

Similarly, yCOM =
m yi1
i i
and
m z i i
i 1
z COM 
M M
Note :
n

• If the origin is taken at the centre of mass then m r
i1
i i = 0. hence, the COM is the point about which
the sum of “mass moments” of the system is zero.
   
• If we change the origin then r1, r2 , r3 ....... changes. So rcm also changes but exact location of center of
mass does not change.

: 0744-2209671, 08003899588 | url : www.motioniitjee.com, : info@motioniitjee.com


Page # 6 CENTRE OF MASS

1.2 Position of COM of two particles : -


Consider two particles of masses m1 and m2 separated by a distance l as shown in figure.
m1 C m2
l
Let us assume that m1 is placed at origin and m2 is placed at position (l, 0) and the distance of centre
of mass from m1 & m2 is r1 & r2 respectively.
m1x1  m2 x2
So xCOM = m1  m 2
r1 C r2
0  m2 l m2 l
r1 = m  m = m m ...(1) (0,0) m1
1 2 1 2 m2
l
m2 l m1l
r2 = l – = m m...(2)
m1  m2 1 2
From the above discussion, we see that

l
r1 = r2 = if m1 = m2, i.e., COM lies midway between the two particles of equal masses.
2
Similarly, r1 > r2 if m1 < m2 and r1 < r2 if m2 < m1 i.e., COM is nearer to the particle having larger mass.
From equation (1) & (2)
m1r1 = m2r2 ...(3)
Centre of mass of two particle system lie on the line joining the centre of mass of two particle system.

Ex.1 Two particle of mass 1 kg and 2 kg are located at x = 0 and x = 3 m. Find the position of their
centre of mass.

m1=1kg COM m2=2kg

Sol. x=0 x=x x=3


r1=x r2=(3–x)
Since, both the particles lie on x-axis, the COM will also lie on x-axis. Let the COM is located at x = x,
then
r1 = distance of COM from the particle of mass 1 kg = x
and r2 = distance of COM from the particle of mass 2 kg = (3 – x)
r1 m2
Using 
r2 m1
x 2
or 
3–x 1
or x=2m
thus, the COM of the two particles is located at x = 2m.

Ex.2 Two particle of mass 4 kg & 2kg are located as shown in figure then find out the position of
centre of mass.
y 2kg
5m

4kg 37°
(0,0) x
Sol. First find out the position of 2 kg mass
x2kg = 5 cos 37° = 4 m

Corporate Head Office : Motion Education Pvt. Ltd., 394 - Rajeev Gandhi Nagar, Kota-5 (Raj.)
CENTRE OF MASS Page # 7

y2kg = 5 sin 37° = 3 m


So these system is like two particle system of mass 4 kg and 2kg are located (0, 0) and (4, 3)
respectively. then
m1x1  m2 x2 024 8 4
xcom = = = =
m1  m 2 42 6 3

m1y1  m2 y 2 032 4kg 2kg


ycom = = =1m r
m1  m 2 42 (0,0) C (4,3)
4 
So position of C.O.M is  , 1
3 

Ex.3 Two particles of mass 2 kg and 4 kg lie on the same line. If 4 kg is displaced rightwards by 5m
then by what distance 2 kg should be move for which centre of mass will remain at the same
position.
Sol. Let us assume that C.O.M. lie at point C and the distance of C from 2kg and 4kg particles are r1 & r2
respectively. Then from relation
2kg C 4kg
r1 r2
m1r1 = m2r2
2r1 = 4r2 ...(i)
Now 4kg is displaced rightwards by 5m then assume 2kg is displaced leftwards by x distance to keep
the C.O.M. at rest.
from relation m1r1 = m2r2
 m1(r1 + x) = m2 (r2 + y)
2(r1 + x) = 4(r2 + 5) ...(ii) 2kg C 4kg
2x = 20 r1 r2 y
x
x = 10 m
To keep the C.O.M at rest 2 kg displaced 10 m left wards
Aliter : If centre of mass is at rest then we can write
m1x = m2y
2×x=4×5
x = 10 m

Ex.4 Two particles of mass 1 kg and 2 kg lie on the same line. If 2kg is displaced 10m rightwards
then by what distance 1kg should displaced so that centre of mass will displaced 2m right
wards.
Sol. Initially let us assume that C.O.M is at point C which is r1 & r2 distance apart from mass m1 & m2
respectively as shown in figure.
1kg C 2kg
r1 r2

from relation m1 r1 = m2 r2
 (1) r1 = 2r2
Now 2kg is displaced 10 m rightwards then we assume that 1 kg is displaced x m leftward to move the
C.O.M 2m rightwards.
So from relation m1r1 = m2r2
1kg C C' 2kg
r1 r2 10m
x
2m

: 0744-2209671, 08003899588 | url : www.motioniitjee.com, : info@motioniitjee.com


Page # 8 CENTRE OF MASS

 1 (x + r1 + 2) = 2 (10 + r2 – 2)
 x + r1 + 2 = 20 + 2r2 – 4 ...(ii)
from eq. (i) & (ii) x = 14m (leftwards)

Ex.5 Three particles of mass 1 kg, 2 kg, and 3 kg are placed at the corners A, B and C respectively of
an equilateral triangle ABC of edge 1m. Find the distance of their centre of mass from A.
Sol. Assume that 1kg mass is placed at origin as shown in figure.
co-ordinate of A = (0, 0)

1 3
co-ordinate of B = (1cos60°,1sin60°) =  2 , 2 
 
co-ordinate of C = (1, 0)

B 2kg
y

1m 1m

A 60° C
(0,0) 1kg 1m 3kg x

Let us assume that position of C.O.M is given by



rcom = xcom i + ycom j

m A xA  mB xB  mC xC
Now xcom = m A  mB  m C

 1
1(0)  2   3(1) 4 2
=  2 = =
1 2  3 6 3

 3
1(0)  2   3(0) 3
ycom =  2  =
6
1 2  3
2 3
Position of centre of mass =  3 , 6 
 

2 2
 2  3 19
distance of C.O.M from point A =      = m
 3  6  6
1.3 Centre of Mass of a Continuous Mass Distribution
For continuous mass distribution the centre of mass can be located by replacing summation sign with
an integral sign. Proper limits for the integral are chosen according to the situation

xcm 
 xdm , y 
 y dm , z 
 z dm
cm cm ...(i)
 dm  dm  dm
 dm = M (mass of the body)
here x,y,z in the numerator of the eq. (i) is the coordinate of the centre of mass of the dm mass.

Corporate Head Office : Motion Education Pvt. Ltd., 394 - Rajeev Gandhi Nagar, Kota-5 (Raj.)
CENTRE OF MASS Page # 9

 1 
rcm =
M
rdm 
Note :
• If an object has symmetric mass distribution about x axis then y coordinate of COM is zero and vice-versa
(a) Centre of Mass of a Uniform Rod
Suppose a rod of mass M and length L is lying along the x-axis with its one end at x = 0 and the other at
M
x = L. Mass per unit length of the rod  =
L
Hence, dm, (the mass of the element dx situated at x = x is) =  dx
The coordinates of the element dx are (x, 0, 0). Therefore, x-coordinate of COM of the rod will be
dx

x=0 x=x x=L


L L
 xdm  (x )dx  1
0 0
L L
xCOM = =L =  L
xdx 
L 0 2
 dm
0   dx
0

The y-coordinate of COM is

yCOM =
 y dm = 0
 dm
Similarly, zCOM = 0

L
i.e., the coordinates of COM of the rod are  , 0, 0 , i.e, it lies at the centre of the rod.
2 

Ex.6 A rod of length L is placed along the x-axis between x = 0 and x = L. The linear density (mass/
length)  of the rod varies with the distance x from the origin as  = Rx. Here, R is a positive
constant. Find the position of centre of mass of this rod.
Sol. Mass of element dx situated at x = x is
dm =  dx = R x dx
The COM of the element has coordinates (x, 0, 0). Therefore, x-coordinates of COM of the rod will be
L L

x COM 
 xdm   ( x )(Rx )dx
0 0
L
 dm  (Rx )dx 0

y
L
x  3
L
x 2 dx  

R 0

 3  0

2L dx
L L 3 x=x x
R  xdx  x2  x=0 x=L
0  
 2  0

The y-coordinates of COM of the rod is y COM 


 y dm  0 (as y = 0)
 dm
Similarly, zCOM = 0

 2L 
Hence, the centre of mass of the rod lies at  , 0,0
3 

: 0744-2209671, 08003899588 | url : www.motioniitjee.com, : info@motioniitjee.com


Page # 10 CENTRE OF MASS

(b) Centre of mass of a Semicircular Ring :


Figure shows the object (semi circular ring). By observation we can say that the x-coordinate of the
centre of mass of the ring is zero as the half ring is symmetrical about y-axis on both sides of the
origin. Only we are required to find the y-coordinate of the centre of mass.

Y Y

Rd

y  R sin
ycm d
ycm

X X
R

To find ycm we use ycm 


(dm)y ...(i)
 dm
Here y is the position of C.O.M. of dm mass
Here for dm we consider an elemental arc of the ring at an angle  from the x-direction of angular width
d. If radius of the ring is R then its y coordinate-will be R sin, here dm is given as
dm  Rd
where  = mass density of semi circular ring.
So from equation ----(i), we have

 Rd(Rsin )
0 R

y cm   
 
sin  d


0
Rd 0

2R
y cm  ...(ii)

(c) Centre of mass of Semicircular Disc :
Figure shows the half disc of mass M and radius R. Here, we are only required to find the y-coordinate
of the centre of mass of this disc as centre of mass will be located on its half vertical diameter. Here to
find ycm, we consider a small elemental ring of mass dm of radius r on the disc (disc can be considered
to be made up such thin rings of increasing radii) which will be integrated from 0 to R. Here dm is given
as
dm   rdr
where  is the mass density of the semi circular disc.
M 2M
= 2

R / 2 R2
Y Y
ycm

ycm dr

r
X X
R R
2r
Now the y-coordinate of the element is taken as , (as in previous section, we have derived that the

Corporate Head Office : Motion Education Pvt. Ltd., 394 - Rajeev Gandhi Nagar, Kota-5 (Raj.)
CENTRE OF MASS Page # 11

2R
centre of mass of a semi circular ring is concentrated at )
R 
 dm . y
y cm  0 R

 dm
0

Here y is the position COM of dm mass.


R
2r
0
 dm  R
4 4R
Here ycm is given as y cm  R r 2 dr  y cm 
  R 2 3
 rdr
0
0

(d) Centre of mass of a Hollow Hemisphere :


A hollow hemisphere of mass M and radius R. Now we consider an elemental circular strip of angular
width d at an angular distance  from the base of the hemisphere. This strip will have an area.
dS  2R cos  Rd
Y Y

Rcos
Rd

ycm

X X
R R
Its mass dm is given as dm  2RcosRd
Here  is the mass density of a hollow hemisphere
M
=
2R2
Here y-coordinate of this strip of mass dm can be taken as R sin. Now we can obtain the centre of
mass of the system as.
 
2 2

 2R 
2
 dmR sin  cos d R sin 
2
y cm  0  0
/2 / 2 R  sin  cos  d O
 2R cos d
2

0
dm 0

R
y cm 
2 y
(e) Centre of mass of a Solid Cone :
A solid cone has mass M, height H and base radius R. r H
Obviously the centre of mass of this cone will lie somewhere dy
on its axis, at a height less than H/2. To locate the centre
of mass we consider an elemental disc of width dy and
radius r, at a distance y from the apex of the cone. Let the
mass of this disc be dm, which can be given as

dm =  × r2 dy
Here  is the mass density of the solid cone
R

: 0744-2209671, 08003899588 | url : www.motioniitjee.com, : info@motioniitjee.com


Page # 12 CENTRE OF MASS

here ycm can be given as


H
1
y cm 
M  y dm
0
H
1  3M  Ry  2 
 dy y

M  

 R2H  H  

0
H
3 3 3H

H3 y dy 
4
0
(f) C.O.M of a solid Hemisphere : -
A hemisphere is of mass density  and radius R To find its centre of mass (only y co-ordinate) we
consider an elemental hollow hemispshere of radius r on the solid hemisphere (solid hemisphere can be
considered to be made up such hollow hemisphere of increasing radii) which will be integrate from O to
R.

solid
hemisphere
R

dr
Here y Co-ordinate of centre of mass of elemental hollow hemisphere is (0, r/2, 0)

dm =  2r2 dr

R R
2
 dm. y  (2r
0
)dr (r / 2)
3R
0 y CM 
yCM = R ; R ; yCM =
2
8
 dm  .2r
0
. dr
0

(g) Centre of mass of Triangular Plate :


A triangular plate has mass density  height H and base is 2R. Obviously the centre of mass of this
plate will lie some where on its axis at a height less than H/2. To locate the centre of mass we consider
an elemental rod of width dy and length 2r at a distance y from the apex of the plate. Let the mass of
this rod be dm which can be gives as
dm =  (2r) dy
from the theorem of triangle
H y

R r y
r H
Ry
 r= rod
H dy
Here YCM can be given as

H H

 0
ydm
 (2r )dy. y
yCM = ; yCM = 0
H H

 dm
0
 (2r) dy
0

Corporate Head Office : Motion Education Pvt. Ltd., 394 - Rajeev Gandhi Nagar, Kota-5 (Raj.)
CENTRE OF MASS Page # 13

H
2Ry
 (
0
H
)ydy
2H
yCM = ; yCM =
H 3
2Ry
 (
0
H
) dy

2. COMBINATION OF STRUCTURE :

Ex.7 Two circular disc having radius R and mass density  and 2 respectively are placed as shown
in figure. Then find out the position of COM of the system.

T
R R
O O'

A B
Sol. Mass of disc A mA = R 2

Mass of disc B mB = 2R2


Due to symmetry the COM of disc A lie at point O and COMO C
O'
of disc B lie at point O. So we realize the above mA mB
problem in a following way x
Centre of mass due to both the disc lie at point C (assume), 2R
having distance x from mA

mB (2R) 2R2 (2R) 4R


 x ; x  ; x
m A  mB ( R2  2R2 ) 3
4R
So the centre of mass lie in the disc B having distance from O.
3
Ex.8 Find out the position of centre of mass of the figure shown below.
2R

R
2R
B rectangular
A C plate
Sol.
Plate
2R
We divide the above problem in two parts
(i) First find out position of centre of mass of both semicircular plate and rectangular plate separately.
(ii) Then find the position of centre of mass of given structure .
4R
Centre of mass of semicircular disc lie at
3
4R
 AB 
3
Centre of mass of rectangular plate lie at the centre of plate at point C
 BC = R
 mSC mR

R 2
m SC  ; mR   4 R2
2
C
 m sc r1 mR

: 0744-2209671, 08003899588 | url : www.motioniitjee.com, : info@motioniitjee.com


Page # 14 CENTRE OF MASS

 Let us assume COM is at r1 distance from mR

R 2  4R 
. R  
2  3  R ( 3  4)
 r1   r1  Ans.
R 2 3(   8)
.   4R2
2
3. CAVITY PROBLEMS :
If some mass or area is removed from a rigid body then the position of centre of mass of the remaining
portion is obtained by assuming that in a remaining part +m & – m mass is there. Further steps are
explained by following example.

Ex.9 Find the position of centre of mass of the uniform lamina shown in figure. If the mass density of
the lamina is  .
y

a
x

Sol. We assume that in remaining portion a disc of radius a/2 having mass density + is there then we also
include one disc of a/2 radius having – mass density. So now the problem change in following form

–

a/2 O O' 
a –
O + O'

a/2
A B
So the centre of mass of both disc A & B lie in their respective centre such as O & O'.
Now
mA a / 2 O C O'
 C.O.M. of the lamina  m  m
A B m A a/2 m B

mA =  ( a2)
a2
mB = –  () (a/2)2 = –  
4
a2 . a / 2 a3 / 2 a3 4 2a
 c= 2 = ;  
a 3a / 4 2 2 3a 2 3
a2 – 
4
i.e., C.O.M lie on leftward side from point O.

Corporate Head Office : Motion Education Pvt. Ltd., 394 - Rajeev Gandhi Nagar, Kota-5 (Raj.)
CENTRE OF MASS Page # 15

Ex.10 Find out the position of centre of mass of the uniform lamina as shown in figure.

R
4R

Sol. We assume that a disc of radius R having mass density ±  is in the removed section.
Now the problem change in following form


 –
R – O
O O'
O' = 4R +
R

R
When disc of mass density + and radius R is include than a complete rectangular plate is make having
centre of mass at point O. When consider only disc having mass density –  and radius R then C.O.M of
this disc lie at point O
O' O
–R2 R (4R) 2
Then the position of C.O.M

( 4R) 2 .R 16R3 16 R
= 2 2 = =
– R  (4R ) R2 (16 – ) 16 – 
i.e., centre of mass lie in the rightwards side from the cavity.

Ex.11 The centre of mass of rigid body always lie inside the body. Is this statement true or false?
Sol. False.

Ex.12 The centre of mass always lie on the axis of symmetry if it exists. Is this statement true of
false?
Sol. True

Ex.13 If all the particles of a system lie in y-z plane, the x-coordinate of the centre of mass will be
zero. Is this statement true or not?
Sol. True

4. MOTION OF CENTRE OF MASS AND CONSERVATION OF MOMENTUM: -


The position of centre of mass is given by

   
m r  m 2 r 2  m 3 r 3  ........
r COM  1 1 ....(1)
m1  m 2  m 3  .......
  
Here m1, m2, m3 ..... are the mass in the system and r1, r2 , r3 ......... is the corresponding position vector of
m1, m2, m3 respectively

: 0744-2209671, 08003899588 | url : www.motioniitjee.com, : info@motioniitjee.com


Page # 16 CENTRE OF MASS

4.1 Velocity of C.O.M of system :


To find the velocity of centre of mass we differentiate equation (1) with respect to time
  
 dr dr dr
m1 1  m 2 2  m 3 3 .......
drcom dt dt dt

dt m1  m2  m3 .........
  
d r1 d r2 d r3
 m1  m2  m3  .......
 Vcom  dt dt dt
m1  m 2  m3  .........

   
m v  m2 v 2  m3 v 3  ......
Vcom  1 1 ...(2)
m1  m2  m3  ...
4.2 Acceleration of centre of mass of the system : -
To find the acceleration of C.O.M we differentiate equation (2)
  
 dv1 dv 2 dv 3
m1  m2  m3 ......
dVcom dt dt dt
 
dt m1  m2  m3 ........
  
 m a  m 2 a2  m3 a 3 ......
acom  1 1 ...(3)
m1  m 2  m 3 ........
   
Now (m1 + m2 + m3) acom = m1a1  m 2 a 2  m 3 a3 ......
   
Fnet (system)  F1net  F2 net  F3 net ........
The internal forces which the particles exert on one another play absolutely no role in the motion of
the centre of mass.
Ex.14 Two particles A and B of mass 1 kg and 2 kg respectively are projected in the directions shown
in figure with speeds uA = 200 m/s and uB = 50 m/s. Initially they were 90 m apart. Find the
maximum height attained by the centre of mass of the particles. Assume acceleration due to
gravity to be constant. (g = 10 m/s2)
Sol. Using mArA = mBrB
or (1) (rA) = (2) (rB) B
or rA = 2rB ...(i)
uB
and rA + rB = 90 m ...(ii) 90m
uA
Solving these two equations, we get
rA = 60 m and rB = 30 m A

i.e., COM is at height 60 m from the ground at time t = 0.


 
 m A a A  mB aB
Further, a COM 
m A  mB

= g = 10 m/s2 (downwards)
 
as a A  aB  g (downwards)
 
 m A u A  mBuB
u COM 
m A  mB

(1)(200 )  ( 2)(50 ) 100


  m / s (upwards)
1 2 3

Corporate Head Office : Motion Education Pvt. Ltd., 394 - Rajeev Gandhi Nagar, Kota-5 (Raj.)
CENTRE OF MASS Page # 17

Let, h be the height attained by COM beyond 60 m. Using,

v 2COM  u2COM  2a COMh

2
 100 
or 0   ( 2) (10)h
 3 

(100) 2
or h  55.55 m
180
Therefore, maximum height attained by the centre of mass is
H = 60 + 55.55 = 115.55 m Ans.

Case I : If Fnet = 0 then we conclude :



(a) The acceleration of centre of mass is zero ( acom  0 )
If a1, a2, a3.... is acceleration of m1, m2, m3 mass in the system then a1, a2, a3 may or may not be zero.
(b) K.E. of the system is not constant it may change due to internal force.

(c) Velocity of centre of mass is constant ( v com  cons tan t) but v1, v2, v3 may or may not constant. It may
y
be change due to internal force.
from eq (2)
  
m1v 1  m 2 v 2  m 3 v 3 ..........  cons tan t
This is called momentum conservation.
"If resultant external force is zero on the system, then the net momentum of the system must
remain constant".

Case II : When centre of mass is at rest.



(a) Vcom  0 then

drcom 
 0  rcom = constant.
dt
  
i.e. r1, r2 , r3 ......... may or may not change

Ex.15 A wooden plank of mass 20 kg is resting on a smooth horizontal floor. A man of mass 60 kg
starts moving from one end of the plank to the other end. The length of the plank is 10 m. Find
the displacement of the plank over the floor when the man reaches the other end of the plank.

10m

Sol. Here, the system is man +plank. Net force on this system is horizontal direction is zero and initially the
centre of mass of the system is at rest. Therefore, the centre of mass does not move in horizontal
direction.
Let x be the displacement of the Plank. Assuming the origin, i.e., x = 0 at the position shown in figure.

: 0744-2209671, 08003899588 | url : www.motioniitjee.com, : info@motioniitjee.com


Page # 18 CENTRE OF MASS

x
x=0 10m Initial position

x 10-x Final position


As we said earlier also, the centre of mass will not move in horizontal direction (x-axis). Therefore, for
centre of mass to remain stationary,
xi = xf
20x = 60 × (10 – x)

30
or x= m or x = 7.5 m Ans.
4
Ex.16 Mr. Verma (50 kg) and Mr. Mathur (60 kg) are sitting at the two extremes of a 4 m long boat (40
kg) standing still in water. To discuss a mechanics problem, they come to the middle of the
boat. Neglecting friction with water, how far does the boat move on the water during the
process?
Sol. Here the system is Mr. Verma + Mr. Mathur + boat. Net force on this system is in horizontal direction is
zero and initially the centre of mass of the system is at rest. Therefore the C.O.M does not move in
horizontal direction. Let x be the displacement of the boat. Then We can use the concept

m1x1  m v x v  mM xM

40 × x = 50 × ( 2 – x) – 60 (2 + x) Mr.Verma Mr. Mathur


40x = 100 – 50 x – 120 – 60x
(60 kg)
150 x = – 20 (50kg)

2
x– m
15
4m 40kg
x  13 cm (right wards)
Initially

x
x
2m 2m

Case III : When net force is zero only in one direction.


Let us assume that Fnet in x direction is zero then we conclude
(i) Acceleration of the system in x direction is zero (ax = 0)

(ii) v(com)x = constant


 m1v1 x + m2 v2 x + m3v3 x = constant.

i.e., momentum is conserved only in x direction

Corporate Head Office : Motion Education Pvt. Ltd., 394 - Rajeev Gandhi Nagar, Kota-5 (Raj.)
CENTRE OF MASS Page # 19

Ex.17 A man of mass m1 is standing on a platform of mass m2 kept on a smooth horizontal surface.
The man starts moving on the platform with a velocity vr relative to the platform. Find the
recoil velocity of platform.
Sol. Absolute velocity of man = vr – v where v = recoil velocity of platform. Taking the platform and the man
a system, net external force on the system in horizontal direction is zero. The linear momentum of the
system remains constant. Initially both the man and the platform were at rest.

vr –v
v

Hence,0 = m1(vr – v) – m2v

m1v r
 v = m m Ans.
1 2

Ex.18 A gun (mass = M) fires a bullet (mass = m) with speed vr relative to barrel of the gun which is
inclined at an angle of 60° with horizontal. The gun is placed over a smooth horizontal surface.
Find the recoil speed of gun.
Sol. Let the recoil speed of gun is v. Taking gun + bullet as the system. Net external force on the

vr
60°
vrsin60°

vrcos60°–v
v
Components of velocity
M of bullet relative to ground

system in horizontal direction is zero. Initially the system was at rest. Therefore, applying the principle
of conservation of linear momentum in horizontal direction, we get
Mv – m(vr cos 60° – v) = 0

mv r cos 60 mv r
 v= or v= Ans.
Mm 2(M  m)

Ex.19 A particle of mass m is placed at rest on the top of a smooth wedge of mass M, which in turn is
placed at rest on a smooth horizontal surface as shown in figure. Then the distance moved by
the wedge as the particle reaches the foot of the wedge is :
m

: 0744-2209671, 08003899588 | url : www.motioniitjee.com, : info@motioniitjee.com


Page # 20 CENTRE OF MASS

m
Sol. There is no external force in horizontal direction
on the wedge block system, So the x-coordinate
of the C.O.M of the wedge block system is at rest.
Let us assume that wedge move x when block M
reaches the ground. We can use the following
relation when
x - coordinate of C.O.M is at rest 
m1x1 = m2x2
Mx = m ( – x)
m
x
mM
M

m
x
x – x

Ex.20 A projectile is fired at a speed of 100 m/s at an angle of 37° above the horizontal. At the
highest point, the projectile breaks into two parts of mass ratio 1 : 3, the lighter piece coming
to rest. Find the distance from the launching point to the point where the heavier piece lands.
Sol. Internal force do not effect the motion of the centre of mass, the centre of mass hits the ground at
the position where the original projectile would have landed. The range of the original projectile is,
4m

37° m
3m
N com
3 4
2u 2 sin  cos  2  10 4  
5 5m
xCOM = =
g 10
= 960 m
The centre of mass will hit the ground at this position. As the smaller block comes to rest after
breaking, it falls down vertically and hits the ground at half of the range, i.e., at x = 480 m. If the
heavier block hits the ground at x2, then
m1x1  m2 x2 (m)(480)  (3m)( x2 )
xcom = m1  m 2  960 = x2 = 1120 m Ans.
(m  3m)
Ex.21 A shell is fired from a cannon with a speed of 100 m/s at an angle 60° with the horizontal
(positive x-direction). At the highest point of its trajectory, the shell explodes into two equal
fragments. One of the fragments moves along the negative x-direction with a speed of 50 m/
s. What is the speed of the other fragment at the time of explosion.
Sol. As we know in absence of external force the motion of centre of mass of a body remains unaffected.
Thus, here the centre of mass of the two fragments will continue to follow the original projectile path.
The velocity of the shell at the highest point of trajectory is vM = u cos  = 100 × cos 60° = 50 m/s Let
v1 be the speed of the fragment which moves along the negative x-direction and the other fragment
has speed v2,. which must be along positive x-direction. Now from momentum conservation, we have
–m m
mv  v1  v 2
2 2
or 2v = v2 – v1 or v2 = 2v + v1
= (2 × 50) + 50 = 150 m/s

Corporate Head Office : Motion Education Pvt. Ltd., 394 - Rajeev Gandhi Nagar, Kota-5 (Raj.)
CENTRE OF MASS Page # 21

Ex.22 A particle of mass 2 m is projected at an angle of 45° with horizontal with a velocity of 20 2m / s.
After 1 s explosion takes place and the particle is broken into two equal pieces.
As a result of explosion one part comes to rest. Find the maximum height attained by the other
part.
(Take g = 10 m/s2)
Sol. Applying conservation of linear momentum at the time of collision, or at t = 1 s,

m v  m(0)  2m( 20 î  10 ĵ)

 v  40ˆj  20ˆj
At 1 sec, masses will be at height :
1 1
v y t 2  (20)(1)  (–10)(1)2  15m
h1  u y t 
2 2
After explosion other mass will further rise to a height :
u2y
( 20)2
h2    20 m
2g 2  10
uy = 20 m/s just after collision.
 Total height h = h1 + h2 = 35 m
Ex.23 A plank of mass 5 kg placed on a frictionless horizontal plane. Further a block of mass 1 kg is
placed over the plank. A massless spring of natural length 2m is fixed to the plank by its one
end. The other end of spring is compressed by the block by half of spring's natural length. They
system is now released from the rest. What is the velocity of the plank when block leaves the
plank? (The stiffness contant of spring is 100 N/m)
1kg

5kg
4m
Sol. Let the velocity of the block and the plank, when the block leaves the spring be u and v respectively.
1 2 1 1
By conservation of energy kx = mu 2 + Mv 2 [M = mass of the plank, m = mass of the block]
2 2 2
 100 = u2 + 5 v2 ...(i)
By conservation of momentum mu + Mv = 0
 u=–5v ...(ii)
Solving Eqs(i) and (ii)
10
30v2 = 100  v  m/ s
3
From this moment until block falls, both plank and block keep their velocity constant.
10
Thus, when block falls, velocity of plank = m/s Ans.
3
Ex.24 Two identical blocks each of mass M = 9 kg are placed on a rough horizontal surface of frictional
coefficient  = 0.1. The two blocks are joined by a light spring and block B is in contact with a
vertical fixed wall as shown in figure. A bullet of mass m = 1kg and v0 = 10 m/s hits block A and
gets embedded in it. Find the maximum compression of spring. (Spring constant = 240 N/m, g
= 10 m/s2)
Sol. For the collision
1 × 10 = 10 × v  v = 1 m/s
If x be the maximum compression
v0 A B
1 1 2
× 10 × 1 = (m + M)gx+ kx
2
M M
2 2 m
1
5 = 10x + 120 x2  x  m Ans.
6

: 0744-2209671, 08003899588 | url : www.motioniitjee.com, : info@motioniitjee.com


Page # 22 CENTRE OF MASS

Ex.25 A flat car of mass M is at rest on a frictionless floor with a child of mass m standing at its edge.
If child jumps off from the car towards right with an initial velocity u, with respect to the car,
find the velocity of the car after its jump.
Sol. Let car attains a velocity v, and the net velocity of the child with respect to earth will be u – v, as u
is its velocity with respect to car.

m u
v
M M

Initially, the system was at rest, thus according to momentum conservation, momentum after jump
must be zero, as
m(u – v) = M v
mu
v
mM
Ex.26 A flat car of mass M with a child of mass m is moving with a velocity v1 on a friction less surface.
The child jumps in the direction of motion of car with a velocity u with respect to car. Find the
final velocities of the child and that of the car after jump.
Sol. This case is similar to the previous example, except now the car is moving before jump. Here also no
external force is acting on the system in horizontal direction, hence momentum remains conserved in
this direction. After jump car attains a velocity v2 in the same direction, which is less than v1, due to
backward push of the child for jumping. After jump child attains a velocity u + v2 in the direction of
motion of car, with respect to ground.
m
m u
v2
M v1 M

According to momentum conservation


(M + m) v1 = Mv2 + m (u + v2)
Velocity of car after jump is
(M  m)v 1  mu
v2 
Mm
(M  m)v 1  (M) u
Velocity of child after jump is u  v 2 
M m

Ex.27 Two persons A and B, each of mass m are standing at the two ends of rail-road car of mass M.
The person A jumps to the left with a horizontal speed u with respect to the car. There after,
the person B jumps to the right, again with the same horizontal speed u with respect to the car.
Find the velocity of the car after both the persons have jumped off.
m m

Corporate Head Office : Motion Education Pvt. Ltd., 394 - Rajeev Gandhi Nagar, Kota-5 (Raj.)
CENTRE OF MASS Page # 23

Sol. Let car attain the velocity v in right ward and velocity of man A with respect to ground is v then
v = v – u
from momentum conservation
0 = mv  + (M + m)v

mu
 m(v – u) + (M + m)v = 0  v
(M  2m)
After wards mass B jumps to the right with the same horizontal speed u with respect to car, than car
attain v" velocity from linear momentum conservation.
(M+m)v = m(u + v") + Mv"
 mu 
(M  m)   mu  (m  M)v "
 M  2m 
m 2u
Now v" =
(M  2m)(M  m )

Ex.28. A block of mass m is placed on a triangular block of mass M, which in turn is placed on a horizontal
surface as shown in figure. Assuming frictionless surfaces find the velocity of the triangular
block when the smaller reaches the bottom end.
m

h
M

Sol. Let us assume that wedge move leftward with velocity v and block move down ward with velocity u
with respect to wedge.
 Net force is horizontal direction is zero so momentum is conserved in x direction.
Now velocity of block with respect to ground is
  
v m  v mw  v w
  
Vm  u  v

Vm  (u cos  – v ) î – u sin ĵ
Now from momentum conservation in x direction
0 = – Mv + mVmx
 Mv = m (u cos  – v) ...(1)
From energy conservation M
v u cos 
1 1 
mgh = mv 2  mv m
2

2 2 u sin 
u
1 1
 mgh= mv 2  m(u 2  v 2 – 2uv cos ) ...(2)
2 2
from eq. (1) & (2)
1/ 2
 2m 2 gh cos  
v 2 
 (M  m)(M  m sin ) 

: 0744-2209671, 08003899588 | url : www.motioniitjee.com, : info@motioniitjee.com


Page # 24 CENTRE OF MASS

5. SPRING BLOCK SYSTEM :


A light spring of spring constant k and natural length l0 attached in a compressed condition between
two blocks of mass m1 & m2 on a smooth horizontal surface as shown in the figure. The spring is initially
compressed by a distance x0.
l0
m1 m2
l0–x0
When system is released the block acquire velocities in opposite direction. Let us assume that the
velocities of block m1 & m2 is v1 & v2 respectively at natural length of the spring and since no external
force acts on this system in horizontal direction. Hence the linear momentum remains constant. Then
from momentum conservation.
0 = m2v2 – m1v1
m2v2 = m1v1 ..(1)
From mechanical energy conservation
Ki + Ui = Kf + Uf
1 2 1 1
 0+ kx 0 = m1v12 + m 2 v 22 + 0
2 2 2
1 2 1 1
 kx0  m1v12  m2 v 22 ...(2)
2 2 2
In initial condition there is no external force on the system and both the block is at stationary
condition. Therefore centre of mass of the system is at rest. So we can write.
m1x1 = m2x2 ...(3)
l0
v1 v2
m1 m2
m1 m2
l0–x0
x1 x2
from above figure we can conclude
l0 – x0 + x1 + x2 = l0
 x0 = x1 + x2 (4)
Due to inertia both the block move further from the position of the natural length of the spring.
Maximum extension occur when both the blocks come to rest. Let us assume that x1' & x2' are the
extension in the spring from the initial position due to block m1 & m2 from natural length
So at maximum extension v1 = v2 = 0

l0
v1=0 v2=0

m1 m2

x1' l0–x0 x2'

 Centre of mass is at rest Therefore we can write


m1x1 = m2x2
x1 + x2 + l0 – x0 = l0 + x0
x1 + x2 = 2x0

Ex.29 A light spring of spring constant k is kept compressed between two blocks of masses m and M
on a smooth horizontal surface. When released, the blocks aquirse velocities in opposite
directions. The spring loses contact with the blocks when it acquires natural length. If the spring
was initially compressed through a distance x, find the final speeds of the two blocks.

Corporate Head Office : Motion Education Pvt. Ltd., 394 - Rajeev Gandhi Nagar, Kota-5 (Raj.)
CENTRE OF MASS Page # 25

Sol. Consider the two blocks plus the spring to be the system. No external force acts on this system in
horizontal direction. Hence, the linear momentum will remain constant. Suppose, the block of mass M
moves with a speed v1 and the other block with a speed v2 after losing contact with the spring. From
conservation of linear momentum in horizontal direction we have
m
Mv1 – mv2 = 0 or v1 = v 2 ...(i)
M
1 2
Initially, the energy of the system = kx
2
Finally, the energy of the system
1 1
= mv 2 2  Mv12
2 2
As there is no friction, mechanical energy will remain conserved.
1 1 1
Therefore, mv 2 2  Mv12  kx 2 ..(ii)
2 2 2
Solving Eqs. (i) and (ii), we get
1/ 2
 kM 
or, v2    x
 m(M  m) 
1/ 2
 kM 
and v1 =   x Ans.
 m(M  m) 
IInd Format : Figure shows two blocks of masses 2m and m are placed on a frictionless surface and connected
with a spring. An external kick gives a velocity v0 m/s to the m mass towards right
v0
B A
2m m
l0
Now velocity of centre of mass is
  
m v  m2 v 2 mv 0  0 v
Vcom  1 1  Vcom = = 0 m / sec
m1  m 2 2m  m 3
Due to kick on m mass block is starts moving with a velocity v0 towards right immediately but due to
inertia 2m block remain at rest at that moment. Thus velocity of block A & B with respect to the centre
v0 2v 0
of mass is vA = v 0 – = m/sec. (towards right)
3 3

v0 v v0
vB = 0 – =– 0 = (towards left)
3 3 3
Now the following figure shown the condition when centre of mass is rest.
v0/3
2v0 /3
2m m
B A
l0
If the maximum extension of the spring is x0 then at this position both the block come to rest condition
with respect to COM so from mechanical energy conservation
 Ki + Ui = Kf + Uf ...(1)
2 2
1  2v 0  1  v0 
Ki = m   2m 
2  3  2  3

Ui = 0 (spring is in natural length)

: 0744-2209671, 08003899588 | url : www.motioniitjee.com, : info@motioniitjee.com


Page # 26 CENTRE OF MASS

Kf = 0 (VA = VB = 0)

1 2
Uf = Kx0
2
Put the above value is equation 1
2 2
1  2v 0  1  v0  1 2 2 2 2
 m   2m   0 = kx 0  kx0  mv 0
2  3  2  3 2 3
2
Maximum extension x0 = v 0 m
3k
IIIrd format :
Example
A block of mass m is connected to another block of mass M by a massless spring of spring constant k.
The blocks are kept on a smooth horizontal plane and are at rest. The spring is unstretched when a
constant force F starts acting on the block of mass M to pull it. Find the maximum extension of the
spring.
F
m M

We solve the situation in the reference frame of centre of mass. As only F is the external force acting
on the system, due to this force, the acceleration of the centre of mass is F/(M+m). Thus with respect
to centre of mass there is a Pseudo force on the two masses in opposite direction, the free body
diagram of m and M with respect to centre of mass (taking centre of mass at rest) is shown in figure.

mF
m M MF
m M F–
mM

Taking centre of mass at rest, if m moves maximum by a distance x1 and M moves maximum by a
distance x2, then the work done by external forces (including Pseudo force) will be
mF
mM mF
m M
mM

mF  MF  mF
W= .x + F –  . x2 = .( x1  x 2 )
mM 1  m  M m M

This work is stored in the form of potential energy of the spring as

1
U= k ( x1  x2 )2
2
Thus on equating we get the maximum extension in the spring, as after this instant the spring starts
contracting.

1 mF
k ( x1  x2 )2 = .( x1  x 2 )
2 m M

2mF
xmax = x1 + x2 =
k (m  M)
Ex.30 Two blocks of equal mass m are connected by an unstretched spring and the system is kept at
rest on frictionless horizontal surface. A constant force F is applied on one of the blocks pulling
it away from the other as shown in figure.

Corporate Head Office : Motion Education Pvt. Ltd., 394 - Rajeev Gandhi Nagar, Kota-5 (Raj.)
CENTRE OF MASS Page # 27

k
m m F

(a) Find the displacement of the centre of mass at time t


(b) If the extension of the spring is x0 at time t, find the displacement of the two blocks at this instant.
Sol. (a) The acceleration of the centre of mass is
F
aCOM =
2m
The displacement of the centre of mass at time t will be

1 Ft 2
x= aCOM t 2 = Ans.
2 4m
(b) Suppose the displacement of the first block is x1 and that of the second is x2. Then,
mx1  mx2
x=
2m
Ft 2 x1  x 2
or, 
4m 2
Ft 2
or, x1 + x2 = ...(i)
2m
Further, the extension of the spring is x1 – x2. Therefore,
x 1 – x2 = x0 ...(ii)
From Eqs. (i) and (ii),

1  Ft 2  1  Ft 2 

x1 = 2  2m  x 0 
 and x2 = 2  2m – x0  Ans.
   

6. IMPULSE :

Impulse of a force F acting on a body for the time interval t = t1 to t = t2 is defined as
 t2 

I = F dt
t1
 

I = F dt

dv 

 m
dt
dt  mdv 
    
I = m(v 2  v1 )   P  change in momentum due to force F
 t2  
Also (impulse - momentum theorem)
IRe =  t1
FRe s dt   P
Note :
* Impulse applied to an object in a given time interval can also be calculated from the area under force
time (F-t) graph in the same time interval.

6.1 Instantaneous Impulse :


There are many cases when a force acts for such a short time that the effect is instantaneous, e.g.,
a bat striking a ball. In such cases, although the magnitude of the force and the time for which it acts
may each be unknown but the value of their product (i.e., impulse) can be known by measuring the
initial and final momentum. Thus, we can write.
    

I = F dt  P  Pf  Pi

: 0744-2209671, 08003899588 | url : www.motioniitjee.com, : info@motioniitjee.com


Page # 28 CENTRE OF MASS

Important Points :
(1) It is a vector quantity.
(2) Dimensions = [MLT–1]
(3) SI unit = kg m/s
(4) Direction is along change in momentum.
(5) Magnitude is equal to area under the F-t. graph.
   
 
(6) I = F dt  Fav dt  Fav t
(7) It is not a property of a particle, but it is a measure of the degree to which an external force
changes the momentum of the particle.

Ex.31 The hero of a stunt film fires 50 g bullets from a machine gun, each at a speed of 1.0 km/s. If he
fires 20 bullets in 4 seconds, what average force does he exert against the machine gun during
this period.
Sol. The momentum of each bullet
= (0.050 kg) (1000 m/s)
= 50 kg-m/s.
The gun has been imparted this much amount of momentum by each bullet fired. Thus, the rate of
change of momentum of the gun
( 50kg – m / s )  20
=
4s
= 250 N
In order to hold the gun, the hero must exert a force of 250 N against the gun.

Ex.32 A ball of mass m = 1kg strikes smooth horizontal floor shown in figure. Find out impulse exerted
on the floor is :
m
–1
5ms
m

53° 37°
Sol. As the ball strike on the surface on impulsive normal force is exerted on the ball as shown in figure.
N
v vx=5cos53°
vy =5sin53°

53° v'sin37
53° 37°
5m/sec
53° 37°
5m/s
v'cos 37
N
This normal force can change only the component vy. So in x direction momentum is conserved.
(Fnet x = 0)
 v cos 37° = 5 cos 53°
535 15
v = = m / sec
54 4
15 3 9
So, v'y = v sin37° =  = m / sec
4 5 4
Impulse = change in linear momentum in y direction
 9
I  N.dt = m(vy – (–v y )) = 1 4 
  = 6.25 N-sec
 4

Corporate Head Office : Motion Education Pvt. Ltd., 394 - Rajeev Gandhi Nagar, Kota-5 (Raj.)
CENTRE OF MASS Page # 29

6.2 Impulsive force :


A force, of relatively higher magnitude and acting for relatively shorter time, is called impulsive force.
An impulsive force can change the momentum of a body in a finite magnitude in a very short time
interval. Impulsive force is a relative term. There is no clear boundary between an impulsive and
Non-Impulsive force.
Note :
* Usually colliding forces are impulsive in nature.
Since, the application time is very small, hence, very little motion of the particle takes place.
Important points :
1. Gravitational force and spring force are always non-Impulsive.
2. Normal, tension and friction are case dependent.
3. An impulsive force can only be balanced by another impulsive force.
1. Impulsive Normal : In case of collision, normal forces at the surface of collision are always impulsive

m1 Ni Ni m2
e.g. Ni = Impulsive; Ng = Non-impulsive
N1g m1g N2g m2g

N1

N2 Both normals are Impulsive

N1
N1

N1, N3 = Impulsive; N2 = non-impulsive


N3
N2

N1
Both normals are Impulsive
N2
2. Impulsive Friction : If the normal between the two objects is impulsive, then the friction between the
two will also be impulsive

N1

Friction at both surfaces is impulsive


N2

N1
N1

N3
N2

Friction due to N2 is non-impulsive and due to N3 and N1 are impulsive

: 0744-2209671, 08003899588 | url : www.motioniitjee.com, : info@motioniitjee.com


Page # 30 CENTRE OF MASS

3. Impulsive Tensions :
When a string jerks, equal and opposite tension act suddenly at each end. Consequently equal and
opposite impulses act on the bodies attached with the string in the direction of the string. There are
two cases to be considered.
• One end of the string is fixed :
The impulse which acts at the fixed end of the string cannot change the momentum of the fixed
object. The object attached to the free end however will undergo a change in momentum in the
direction of the string. The momentum remains unchanged in a direction perpendicular to the string
where no impulsive forces act.
• Both ends of the string attached to movable objects :
In this case equal and opposite impulses act on the two objects, producing equal and opposite
changes in momentum. The total momentum of the system therefore remains constant, although the
momentum of each individual object is changed in the direction of the string. Perpendicular to the
string however, no impulse acts and the momentum of each particle in this direction is unchanged.

T
T is impulsive
A

T is non-impulsive B

C
T is non-impulsive

All normal are impulsive but tension


T is impulsive only for the ball A
For this example :
In case of rod, tension is always impulsive and in case of spring, tension is always non-impulsive.
Ex.33 A block of mass m and a pan of equal mass are connected by a string going over a smooth light
pulley. Initially the system is at rest when a particle of mass m falls on the pan and sticks to it.
If the particle strikes the pan with a speed v, find the speed with which the system moves just
after the collision.

m
v

m m

Sol. Let the required speed is V.


Further, let J1 = impulse between particle and pan
and J2 = impulse imparted to the block and the pan by the string
Using, Impulse = change in momentum
For particle J1 = mv – mV ...(i)
For pan J1 – J2 = mV ...(ii)
For block J2 = mV ...(iii)
v
Solving, these three equation, we get V Ans.
3
Alternative solution :
Applying conservation of linear momentum along the string ;
mv = 3mV
v
we get, V= Ans.
3

Corporate Head Office : Motion Education Pvt. Ltd., 394 - Rajeev Gandhi Nagar, Kota-5 (Raj.)
CENTRE OF MASS Page # 31

Ex.34 Two identical block A and B, connected by a massless string are placed on a frictionless horizontal
plane. A bullet having same mass, moving with speed u strikes block B from behind as shown.
If the bullet gets embedded into the block B then find :
m
m u m
C
A B

(A) The velocity of A, B, C after collision


(B) Impulse on A due to tension in the string
(C) Impulse on C due to normal force of collision.
(D) Impulse on B due to normal force of collision.
Sol. Let us assume that all the three are move with velocity v
v v
N N
T
A B
Take rightward direction is +ve. Then first we write impulse equation on bullet
–  N.dt  mv – mu ...(1)
Now impulse equation on block B

 (N – T)dt  mv ...(2)
Impulse equation on block A

 T.dt  mv ...(3)
(a) Add eq. (1), (2), (3) then
u
0 = 3mv – mu  v=
3
mu
(b) Impulse on A due to Tension in the string from eq. (3)  T.dt  3
(c) Impulse on C due to normal force of collision
u  2mu

from eq. (1) N. dt  m 3 – u = –
3
(d) Impulse on B due to normal force of collision
from eq. (2)
mu
 (N – T)dt 
3
mu mu mu 2mu
 N. dt 
3 
 T. dt =
3

3

3

7. COEFFICIENT OF RESTITUTION (e)


The coefficient of restitution is defined as the ratio of the impulses of reformation and deformation of
either body.

Im pulse of reformation  F dt
r
e= =
Im pulse of deformation  F dt
d

Velocity of separation of point of contact


e
Velocity of approach of point of contact

: 0744-2209671, 08003899588 | url : www.motioniitjee.com, : info@motioniitjee.com


Page # 32 CENTRE OF MASS

Example for calculation of e :


Two smooth balls A and B approaching each other such that their centres are moving along line CD in
absence of external impulsive force. The velocities of A and B just before collision be u1 and u2
respectively. The velocities of A and B just after collision be v1 and v2 respectively.
Just Before collision Just After collision
u1 u2 v1 v2
u1>u2

C D C D
A B A B v2 > v1
v 2 – v1
e = u –u
1 2

Note : Coefficient of restitution is a factor between two colliding bodies which is depends on the material of
the body but independent of shape.
We can say e is a factor which relates deformation and reformation of the body.
0 e1
Ex.35 If a body falls normally on a surface from height h, what will be the height regained after
collision if coefficient of restitution is e?

Sol. h

If a body falls from height h, from equations of motion we know that it will hit the ground with a
velocity say u = 2gh which is also the velocity of approach here.

Now if after collision it regains a height h1 then again by equations of motion v = 2gh1 which is also
the velocity of separation. So, by definition of e,
2gh1
e= or h1 = e2h
2gh

Ex.36 A block of mass 2 kg is pushed towards a very heavy


object moving with 2 m/s closer to the block (as 2m/s
shown). Assuming elastic collision and frictionless very
10m/s heavy
surface, find 2kg object
the final velocities of the blocks.
Sol. Let v1 and v2 be the final velocities of 2kg block
2m/s
and heavy object respectively then, very
14m/s heavy
v1 = u1 + 1 (u1 – u2) = 2u1 – u2 2kg object
= – 14 m/s
v2 = – 2m/s

Ex.37 A ball is moving with velocity 2 m/s towards a heavy wall moving towards the ball with speed
1 m/s as shown in fig. Assuming collision to be elastic, find the velocity of the ball immediately
after the collision.

2m/s 1m/s

Sol. The speed of wall will not change after the collision. So, let v be the velocity of the ball after collision
in the direction shown in figure. Since collision is elastic (e = 1).

Corporate Head Office : Motion Education Pvt. Ltd., 394 - Rajeev Gandhi Nagar, Kota-5 (Raj.)
CENTRE OF MASS Page # 33

2m/s 1m/s v 1m/s

Before Collision After Collision


separation speed = approach speed
or v–1=2+1
or v = 4 m/s Ans.
Ex.38 A ball is dropped from a height h on to a floor. If in each collision its speed becomes e times of
its striking value (a) find the time taken by ball to stop rebounding (b) find the total change in
momentum in this time (c) find the average force exerted by the ball on the floor using results of
part (a) and (b).
Sol. (a) When the ball is dropped from a height h, time taken by it to reach the ground will be

2h
t0 = and its speed v0 = 2gh
g

h v0 v1
v2

t0 t1 t0
Now after collision its speed will becomes e times, i.e., v1 = ev0 = e 2gh and so, it will take time to go
up till its speed becomes zero = (v1/g). The same time it will take to come down. So total time between
I and II collision will be t1 = 2v1/g. Similarly, total time between II and III collision t2 = 2v2/g.
So total time of motion
T = t0 + t1 + t2 +.........
2 v1 2v 2
or T = t0 + + .......
g g

2ev 0 2e 2 v 0
or T = t0 + + .......
g g
[as v2 = ev1 = e2v0]

2h
or T = [1  2e(1  e  e2 ....)]
g

2h   1  2h  1  e 
 1  2e  
g  1 e  g  1 – e 

(b) Change in momentum in I collision


= mv1 – (–mv0) = m (v1 + v0)
Change in momentum in II collision = m(v2 + v1)
Change in momentum in nth collision = m(vn + vn–1)
Adding these all total change in momentum
p = m[v0 + 2v1 + ....+ 2vn–1 + vn]
or p = mv0[1 + 2e + e2 + .....]

  1  1 e 
or p = mv0 1  2e 1 – e    m 2gh  1 – e  ...(2)
   

: 0744-2209671, 08003899588 | url : www.motioniitjee.com, : info@motioniitjee.com


Page # 34 CENTRE OF MASS


 p
(C) Now as F  d p so, Fav =
dt T
Substituting the value of T and p from Eqns. (1) and (2)

1 e  g 1 – e 
Fav = m 2gh  1 – e  × = mg ...(3)
  2h  1  e 

7.1 Line of Motion


The line passing through the centre of the body along the direction of resultant velocity.

7.2 Line of Impact


The line passing through the common normal to the surfaces in contact during impact is called line of
impact. The force during collision acts along this line on both the bodies.
Direction of Line of impact can be determined by :
(a) Geometry of colliding objects like spheres, discs, wedge etc.
(b) Direction of change of momentum.
If one particle is stationary before the collision then the line of impact will be along its motion after
collision.
Examples of line of impact
(i) Two balls A and B are approaching each
Line of impact other such that their centres are moving along line CD.
and
line of motion

C D
A
B

(ii) Two balls A and B are approaching each other such that their centre are moving along dotted lines
as shown in figure.

B Line of motion
of ball A
Line of motion
of ball B
D

A
Line of impact

(iii) Ball is falling on a stationary wedge.

Line of motion of ball

Line of impact

Corporate Head Office : Motion Education Pvt. Ltd., 394 - Rajeev Gandhi Nagar, Kota-5 (Raj.)
CENTRE OF MASS Page # 35

Note : In previous discussed examples line of motion is same as line of impact. But in problems in which line of
impact and line of motion is different then e will be

velocity of seperation along line of impact


e=
velocity of approach along line of impact
Ex.40 A ball of mass m hits a floor with a speed v making an angle of incident  with the normal. The
coefficient of restitution is e. Find the speed of the reflected ball and the angle of reflection of the
ball.
Sol. Suppose the angle of reflection is  and the speed after the collision is v  (shown figure) The floor
exerts a force on the ball along the normal during the collision. There is no force parallel to the surface.
Thus, the parallel component of the velocity of the ball remains unchanged. This gives
v  sin   = v sin  ...(i)
For the components normal to the floor, the velocity
of separation is v cos  and the velocity of approach is v cos . v v'
 '
Hence, v  cos   = ev cos 
From (i) and (ii), v  = v sin 2   e 2 cos 2  v sin v'cos '
tan v cos  v'sin '
Hence, tan   =
e Final velocity
Initial velocity
For elastic collision, e = 1, so that  =  and v = v.
Ex.41 A ball is projected from the ground at some angle with horizontal. Coefficient of restitution
between the ball and the ground is e. Let a, b and c be the ratio of times of flight, horizontal
range and maximum height in two successive paths. Find a, b and c in terms of e?

1
2

Sol. Let us assume that ball is projected with speed u at an angle  with the horizontal. Then
Before first collision with the ground.
2u y
Time fo flight T  euy
g
u
2u x u y uy  u sin  I
 ux II
Horizontal range R 
g
ux  u cos 
u 2y uy
Maximum Height Hmax = ...(1)
2g
After striking the ground the component uy is change into e uy, so

2eu y 2u x (eu y )
Time of flight T = , R' 
g g

'
(eu y )2
Hmax  ...(2)
2g
from eq (1) & (2)

T 1
Now a
T' e

: 0744-2209671, 08003899588 | url : www.motioniitjee.com, : info@motioniitjee.com


Page # 36 CENTRE OF MASS

R 1 Hmax 1
b ; '
 2 =c
R' e Hmax e

Ex.42 A ball is projected from the ground with speed u at an angle  with horizontal. It collides with a
wall at a distance a from the point of projection and returns to its original position. Find the
coefficient of restitution between the ball and the wall.
Sol. A ball is a projected with speed u at an angle  with horizontal. It collides at a distance a with a wall
parallel to y-axis as shown in figure.

Let vx and vy be the components of its velocity along x and y-directions at the time of impact with wall.
Coefficient of restitution between the ball and the wall is e.

Component of its velocity along y-direction (common tangent) vy will remain unchanged while component
of its velocity along x-direction (common normal) vx will becomes evx is opposite direction.

*Further, since vy does not change due to collision, the time of flight (time taken by the ball to return
to the same level) and maximum height attained by the ball remain same as it would had been in the
absence of collision with the wall. Thus,
vy v vy

B B
A vx C evx
u
y

O x O
a a
From O A B, R = a = u cos  . tOAB

from BCO, R = a = eucos. tBCO

T = tOAB + tBCO

2u sin a a a 2u sin a
or = + or = –
g u cos  eucos  eucos  g u cos 

a 2u2 sin  cos  – ag ag


or   e= 2
eu cos  gu cos  2u sin  cos  – ag

1
or e =  u 2 sin 2  Ans.
 – 1
 ag 
Ex.43 To test the manufactured properties of 10 N steel balls, each ball is released from rest as
shown and strikes a 45° inclined surface. If the coefficient of restitution is to be e = 0.8.
determine the distance s to where the ball must strike the horizontal plane at A. At what speed does
the ball strike at A? (g = 9.8 m/s2)

1.5m

1.0m
45°
A
s

Corporate Head Office : Motion Education Pvt. Ltd., 394 - Rajeev Gandhi Nagar, Kota-5 (Raj.)
CENTRE OF MASS Page # 37

Sol. v0 = 2gh = . = 5.42 m/s


2  9.8  15
Component of velocity parallel and perpendicular to plane at the time of collision.
v0
v1 = v2 = = 3.83 m/sec.
2
ev2=0.8v2

C C v1
v2 v1 1.0 m
v0 45° x
45° D E A
s
y
x
Component parallel to plane (v1) remains unchanged, while component perpendicular to plane becomes
ev2, where
ev2 = 0.8 × 3.83 = 3.0 m/s
 Component of velocity in horizontal direction after collision
( v1  ev 2 ) ( 3.83  3.0)
vx = = = 4.83 m/s
2 2
While component of velocity in vertical direction after collision.
v1 – ev 2 3.83 – 3.0
vy = = = 0.59 m/s
2 2
Let t be the time, the particle takes from point C to A, then
1
1.0 = 0.59 t + × 9.8 × t2 ; t = 0.4 sec
2
Solving this we get,
 DA = vxt = (4.83)(0.4) = 1.93 m
 S = DA – DE = 1.93 – 1.0
S = 0.93 m
vyA = vyc + gt = (0.59) + (9.8) (0.4) = 4.51 m/s
vxA = vxC = 4.83 m/s

 vA = ( v xA ) 2  ( v yA ) 2 = 6.6 m/s

Ex.44 A ball of mass m = 1 kg falling vertically with a velocity v0 = 2m/s strikes a wedge of mass
M = 2kg kept on a smooth, horizontal surface as shown in figure. The coefficient of restitution
1
between the ball and the wedge is e = . Find the velocity of the wedge and the ball immediately
2
after collision.

m
v0
M 30°

Sol. Given M = 2kg and m = 1kg

: 0744-2209671, 08003899588 | url : www.motioniitjee.com, : info@motioniitjee.com


Page # 38 CENTRE OF MASS

J
v3 Jcos30°
Jsin30°
v1 m v2 m
Jsin30°
M J 30°
30°

Jcos30°
Let, J be the impulse between ball and wedge during collision and v1, v2 and v3 be the components of
velocity of the wedge and the ball in horizontal and vertical directions respectively.
Applying impulse = change in momentum
we get J sin 30° = Mv1 = mv2

J
or = 2v1 = v2
2

J cos 30° = m(v3 + v0) ...(i)

3
or J = (v3 + 2) ...(ii)
2

Applying, relative speed of separation = e


(relative speed of approach) in common normal direction, we get

1
(v1 + v2) sin 30° + v3 cos30° = ( v 0 cos 30 )
2
Common normal
3 direction
or v1 + v2 + 3 v3 = ...(iii)
2
Solving Eqs. (i), (ii) and (iii), we get
30°
1
v1 = m/s
3

2
v2 = m / s and v = 0
3
3
Thus, velocities of wedge
1
and ball are v1 = m/s
3

1 2
v1  m/s v2  m/s
3 3

30°

2
and v2 = m / s in horizontal direction as shown in figure.
3

Corporate Head Office : Motion Education Pvt. Ltd., 394 - Rajeev Gandhi Nagar, Kota-5 (Raj.)
CENTRE OF MASS Page # 39

8. COLLISION OR IMPACT
Collision is an event in which an impulsive force acts between two or more bodies for a short time,
which results in change of their velocities.
Note :
• In a collision, particles may or may not come in physical contact.
• The duration of collision, t is negligible as compared to the usual time intervals of observation of
motion.
• In a collision the effect of external non impulsive forces such as gravity are not taken into account as
due to small duration of collision (t) average impulsive force responsible for collision is much larger
than external forces acting on the system.

The collision is in fact a redistribution of total momentum of the particle :

Thus law of conservation of linear momentum is indepensible in dealing with the phenomenon of
collision between particles. Consider a situation shown in figure.

Two balls of masses m1 and m2 are moving with velocities v1 and v2 (< v1) along the same straight line
in a smooth horizontal surface. Now let us see what happens during the collision between two particles.
v1 v2

m1 m2

figure (a)
v1 ' v2 '

N N N
N

figure(b)
figure(c)
figure (a) : Balls of mass m1 is behind m2. Since v1 > v2, the balls will collide after some time.

figure (b) : During collision both the balls are a little bit deformed. Due to deformation two equal and
opposite normal forces act on both the balls. These forces decreases the velocity of m1 and increase
the velocity of m2

figure (c): Now velocity of ball m1 is decrease from v1 to v1 and velocity of ball m2 is increase from v2
to v2. But still v1 > v2 so both the ball are continuously deformed.

figure(d) : Contact surface of both the balls are deformed till the velocity of both the balls become
equal. So at maximum deformation velocities of both the blocks are equal
v1 '' v 2 ''

figure(d)

at maximum deformation v1 ''  v 2 ''

figure(e) : Normal force is still in the direction shown in figure i.e. velocity of m1 is further decreased

: 0744-2209671, 08003899588 | url : www.motioniitjee.com, : info@motioniitjee.com


Page # 40 CENTRE OF MASS

and that of m2 increased. Now both the balls starts to regain their original shape and size.

v1 '' v 2 ''

N N v 2 ''  v1 ' '

figure(e)

figure (f) : These two forces redistributes their linear momentum in such a manner that both the
blocks are separated from one another, Velocity of ball m2 becomes more than the velocity of block m1
i . e . ,
v2 > v1
v1 v2

m1 m2 v2>v1

figure(f)
The collision is said to be elastic if both the blocks regain their original form, The collision is said to be
inelastic. If the deformation is permanent, and the blocks move together with same velocity after the
collision, the collision is said to be perfectly inelastic.

8.1 Classification of collisions


(a) On the basis of line of impact
(i) Head-on collision : If the velocities of the colliding particles are along the same line before and
after the collision.
(ii) Oblique collision : If the velocities of the colliding particles are along different lines before and
after the collision.
(b) On the basis of energy :
(i) Elastic collision :
(a) In an elastic collision, the colliding particles regain their shape and size completely after collision.
i.e., no fraction of mechanical energy remains stored as deformation potential energy in the bodies.
(b) Thus, kinetic energy of system after collision is equal to kinetic energy of system before collision.
(c) e = 1
(d) Due to Fnet on the system is zero linear momentum remains conserved.
(ii) Inelastic collision :
(a) In an inelastic collision, the colliding particles do not regain their shape and size completely after
collision.
(b) Some fraction of mechanical energy is retained by the colliding particles in the form of deformation
potential energy. Thus, the kinetic energy of the particles no longer remains conserved.
(c) However, in the absence of external forces, law of conservation of linear momentum still holds
good.
(d) (Energy loss)Perfectly Inelastic > (Energy loss)Partial Inelastic
(e) 0 < e < 1

Corporate Head Office : Motion Education Pvt. Ltd., 394 - Rajeev Gandhi Nagar, Kota-5 (Raj.)
CENTRE OF MASS Page # 41

(iii) Perfectly Inelastic collision :


(i) In this the colliding bodies do not return to their original shape and size after collision i.e. both the
particles stick together after collision and moving with same velocity
(ii) But due to Fnet of the system is zero linear momentum remains conserved.
(iii) Total energy is conserved.
(iv) Initial kinetic energy > Final K.E. Energy
(v) Loss in kinetic energy goes to the deformation potential energy
(vi) e = 0

8.2 Value of Velocities after collision :


Let us now find the velocities of two particles after collision if they collide directly and the coefficient
of restitution between them is given as e.
m1 m2 m1 m2
u1 u2 v1 v2

(a) (b)
Before Collision After Collision
u1 > u2 v2 > v1

v 2 – v1
e= u –u
1 2

 (u1 – u2)e = (v2 – v1) ...(i)


By momentum conservation
m1u1 + m2u2 = m1v1 + m2v2 ...(ii)
v2 = v1 + e(u1 – u2) ...(iii)
from above equation

m1u1  m 2u 2  m 2 e(u 2 – u1 )
v1 = m1  m 2
...(iii)

m 1u1  m 2 u 2  m 1e(u 1 – u 2 )
v2 = m1  m 2
...(iv)

Special cases :
u
1. If m1 >> m2 and u2 = 0 and u1 = u
and e=1 m1
m1 = m2 m2
from eq. (iii) & (iv)
m1u – m2u u(m1 – m2 )
v1 = m1  m 2 = m1  m 2

v1 ~– u

m1u  m 2u 2m1u
v2 = = ; v2 = 2u
m1  m 2 m1  m 2

: 0744-2209671, 08003899588 | url : www.motioniitjee.com, : info@motioniitjee.com


Page # 42 CENTRE OF MASS

2. If m1 = m2 = m and e = 1 then u1 u2

from eq. (iii) & (iv) m m

m(u1  u 2 )  m(u2 – u1 )
v1 =
2m

v1 = u2
In this way v2 = u1
i.e when two particles of equal mass collide elastically and the collision is head on, they exchange their
velocities.
8.3 Collision in two dimension (oblique) :
1. A pair of equal and opposite impulses act along common normal direction. Hence, linear momentum of
individual particles change along common normal direction. If mass of the colliding particles remain
constant during collision, then we can say that linear velocity of the individual particles change during
collision in this direction.

2. No component of impulse act along common tangent direction. Hence, linear momentum or linear
velocity of individual particles (if mass is constant) remain unchanged along this direction.

3. Net impulse on both the particles is zero during collision. Hence, net momentum of both the particles
remain conserved before and after collision in any direction.

4. Definition of coefficient of restitution can be applied along common normal direction, i.e., along common
normal direction we can apply Relative speed of separation = e (relative speed of approach)

Ex.45 A ball of mass m makes an elastic collision with another identical ball at rest. Show that if the
collision is oblique, the bodies go at right angles to each other after collision.

Sol. In head on elastic collision between two particles, they exchange their velocities. In this case, the
component of ball 1 along common normal direction, v cos  becomes zero after collision, while
v sin
v sin
1
v 1

v cos

2
2
v cos
Before collision After collision

that of 2 becomes v cos . While the components along common tangent direction of both the particles
remain unchanged. Thus, the components along common tangent and common normal direction of both
the balls in tabular form are given a head :

Component along common Component along common


Ball
tangent direction normal direction

Before collision After collision Before collision After collision


1 v sin  v sin  v cos  0
2 0 0 0 v cos 

Corporate Head Office : Motion Education Pvt. Ltd., 394 - Rajeev Gandhi Nagar, Kota-5 (Raj.)
CENTRE OF MASS Page # 43

From the above table and figure, we see that both the balls move at right angles after collision with
velocities v sin  and v cos .

Note : When two identical bodies have an oblique elastic collision, with one body at rest before collision, then
the two bodies will go in  directions.

Ex.46 Two spheres are moving towards each other. Both have same radius but their masses are 2kg
and 4kg. If the velocities are 4m/s and 2m/s respectively and coefficient of restitution is e =
1/3, find.
(a) The common velocity along the line of impact.

(b) Final velocities along line of impact.

2kg 4m/s
A
R 2m/s R
B
4kg

(c) Impulse of deformation.

(d) impulse of reformation

(e) Maximum potential energy of deformation

(f) Loss in kinetic energy due to collision.

BC R 1
Sol. In ABC sin = = =
AB 2R 2

or  = 30°

A 4m/s C Line of motion


2kg 
R R
R 4kg
Line of motion 2m/s B
Line of impact

(a) By conservation of momentum along line of impact.

LOI 4sin30°

2kg 4m/s 4sin30°


30°
4cos30°

2cos30° v
30° B 4kg 2cos30°
2m/s
Maximum Deformed
2sin30° State
Just Before Collision Along LOI

2(4 cos 30°) – 4(2cos30°) = (2 + 4)v


or v = 0 (common velocity along LOI)
(b) Let v1 and v2 be the final velocity of A and B respectively then, by conservation of momentum
along line of impact, 2(4 cos 30°) – 4(2cos30°) = 2(v1) + 4(v2)

: 0744-2209671, 08003899588 | url : www.motioniitjee.com, : info@motioniitjee.com


Page # 44 CENTRE OF MASS

4sin30°

A v1
2kg

4kg B
v2
2sin30°
Just After Collision
Along LOI

or 0 = v1 + 2v2 ........(1)
By coefficient of restitution,

velocity of separation along LOI


e=
velcoity of approach along LOI

1 v 2 – v1
or = or v 2 – v1 = 3 ...(2)
3 4 cos 302 cos 30
from the above two equations,

–2 1
v1 = m / s and v = m/s
2
3 3

(c) J0 = m1(v – u1) = 2 (0 – 4 cos 30°) = – 4 3 N-s

1 4
(d) JR = eJ0 = (–4 3 ) = – N s
3 3
(e) Maximum potential energy of deformation is equal to loss in kinetic energy during deformation upto
maximum deformed state,

1 1 1 1 1 1
U= m1(u1 cos ) 2  m2 (u2 cos ) 2 – (m1  m 2 )v 2 = 2(4 cos 30 ) 2  4(–2cos 30 )2 – ( 2  4)( 0)
2 2 2 2 2 2

or U = 18 Joule
(f) Loss in kinetic energy

1 1 1 2 1 2
KE = m1(u1 cos ) 2 + m2 (u 2 cos  )2 –  m1v 1  m 2 v 2 
2 2 2 2 

 1  2  2 1  1  2
1 1   
= 2 (4 cos 30°) + 4 (–2 cos 30°) –  2 2   4
 2   
2 2  3 3

KE = 16 Joule

9. VARIABLE MASS
In our discussion of the conservation linear momentum, we have so far dealt with systems whose
system whose mass remains constant. We now consider those mass is variable, i.e., those in which
mass enters or leaves the system. A typical case is that of the rocket from which hot gases keep on
escaping thereby continuously decreasing its mass.

In such problem you have nothing to do but apply a thrust force (Ft ) to the main mass in addition to
the all other force acting on it. This thrust force is given by,
   dm 
Ft  v rel  
 dt 

Corporate Head Office : Motion Education Pvt. Ltd., 394 - Rajeev Gandhi Nagar, Kota-5 (Raj.)
CENTRE OF MASS Page # 45


Here v rel is the velocity of the mass gained or mass ejected relative to the main mass. In case of

dm
rocket this is sometimes called the exhaust velocity of the gases. is the rate at which mass is
dt
increasing or decreasing.

v vr
dm v + dv
m m–dm
system
The expression for the thrust force can be derived from the conservation of linear momentum in the
absence of any external forces on a system as follows :

Suppose at some moment t = t mass of a body is m and its velocity is v . After some time at t = t + dt
 
its mass becomes (m – dm) and velocity becomes v  dv . The mass dm is ejected with relative velocity
   
v r . Absolute velocity of mass ‘dm’ is therefore ( v  v  dv) . If no external forces are acting on the
r
system, the linear momentum of the system will remain conserved,
       
or Pi  Pf or mv  (m  dm)( v  dv)  dm ( vr  v  dv)

       
or mv  mv  mdv  dmv  (dm)(dv)  dm v  vr dm  ( dm)((dv)


 
m dv   vr dm

 
or  dv    dm 
m    vr   
 dt   dt 
 

  dm
Here,  dv  
m   = thrust force (F1) and = rate at which mass is ejecting
 dt  dt
 

Problems related to variable mass can be solved in following three steps

1. Make a list of all the forces acting on the main mass and apply them on it.

   dm 
2. Apply an additional thrust force Ft on the mass, the magnitude of which is v r   dt  and direction is
 
given by the direction of v r in case the mass is increasing and otherwise the direction of  v r if it is
decreasing.

3. Find net force on the mass and apply


 dv
Fnet  m (m = mass at that particular instant)
dt

: 0744-2209671, 08003899588 | url : www.motioniitjee.com, : info@motioniitjee.com


Page # 46 CENTRE OF MASS

9.1 Rocket Propulsion

Let m0 be the mass of the rocket at time t = 0. m its mass at any time t and v its velocity at that
moment. Initially let us suppose that the velocity of the rocket is u.
u u

At t=0 At t=t
v=u m=m
m = m0 v=v

Exhaust velocity = vr
  dm 
Further, let  dt  be the mass of the gas ejected per unit time and vr the exhaust velocity of the

  dm 
gases. Usually  dt  and vr are kept constant throughout the journey of the rocket. Now, let us
write few equations which can be used in the problems of rocket propulsion. At time t = t
1. Thrust force on the rocket

 dm 
Ft  v r    (upwards)
 dt 

2. Weight of the rocket


W = mg (downwards)
3. Net force on the rocket
Fnet = Ft – W (upwards)
 – dm 
or Fnet  v r   – mg
 dt 

F
4. Net acceleration of the rocket a 
m
dv v r   dm 
or    g
dt m  dt 

  dm 
or dv  v r    g dt
 m 

v m dm t
or u
dv  vr 
m0 m 
 g dt
0

m 
or v – u = vr In  0   gt
 m

 m0 
Thus, v = u – gt + vr In   ...(i)
m

Corporate Head Office : Motion Education Pvt. Ltd., 394 - Rajeev Gandhi Nagar, Kota-5 (Raj.)
CENTRE OF MASS Page # 47

 dm  dm
Note : 1. Ft  v r    is upwards, as vr is downwards and is negative.
 dt  dt
 m0 
2. If gravity is ignored and initial velocity of the rocket u = 0, Eq. (i) reduces to v = vr In  m  .

Ex.47 A uniform chain of mass per unit length  begins to fall with a velocity v on the table. Find the
thrust force exerted by the chain on the table.
Sol. Let us assume that the mass of the chain is m and length .
We assume that after time t, x length of the chain has fallen on the table. Then the speed of the upper
part of the chain is 2gx as shown in figure.

x
m
2gx  v  v r

at t =0
at time t = t

Now its time t + dt, length of chain has fallen on the table is v dt. Then the mass of chain has fallen on
the table is

m
dm  .vdt
 x
t
Now the rate of increase of mass vdt
t + dt
dm m m
 v 2gx
dt  
Here v is downward and mass is increasing so thrust
force act in down ward direction and is given by

dm
ft  v r at time t + dt
dt
m
= 2gx ( 2gx )

ft =  v2

: 0744-2209671, 08003899588 | url : www.motioniitjee.com, : info@motioniitjee.com


Page # 48 CENTRE OF MASS

Exercise - I Objective Problems (JEE MAINS)


1. A man of mass M stands at one end of a plank of 3. Two balls A and B of masses 100gm and 250
length L which lies at rest on a frictionless surface. gm respectively are connected by a stretched
The man walks to other end of the plank. If the spring of negligible mass and placed on a smooth
M table. When the balls are released simultaneously
mass of the plank is
3
, then the distance that the the initial acceleration of B is 10 cm/sec2 west
man moves relative to ground is : ward. What is the magnitude and direction of
initial acceleration of the ball A-
3L L 4L L
(A) (B) (C) (D) (A) 25 cm/sec2 Eastward
4 4 5 3
Sol. (B) 25 cm/sec2 North ward
(C) 25 cm/sec2 West ward
(D) 25 cm/sec2 South ward
Sol.

4. A uniform square plate ABCD has a mass of


10kg. If two point masses of 3 kg each are
placed at the corners C and D as shown in the
adjoining figure, then the centre of mass shifts
2. The centre of mass of two particles lies to the point which is lie on -
(A) on the line perpendicular to the line joining
the particles
(B) on a point outside the line joining the particles
(C) on the line joining the particles.
(D) none of the above .
Sol.

(A) OC (B) OD (C) OY (D) OX


Sol.

Corporate Head Office : Motion Education Pvt. Ltd., 394 - Rajeev Gandhi Nagar, Kota-5 (Raj.)
CENTRE OF MASS Page # 49

5. A particle of mass 3m is projected from the ground 7. Two particles having mass ratio n : 1 are
at some angle with horizontal. The horizontal range interconnected by a light inextensible string that
is R. At the highest point of its path it breaks into passes over a smooth pulley. If the system is
two pieces m and 2m. The smaller mass comes to released, then the acceleration of the centre of
rest and larger mass finally falls at a distance x from mass of the system is :
the point of projection where x is equal to 2
 n  1
(A) (n –1)2 g (B)   g
3R 3R 5R  n – 1
(A) (B) (C) (D) 3R
4 2 4
2
Sol.  n – 1  n  1
(C)   g (D)  n – 1 g
 n  1
Sol.

6. A man weighing 80 kg is standing at the centre 8. Internal forces can change


of a flat boat and he is 20 m from the shore. He (A) the linear momentum but not the kinetic energy
walks 8 m on the boat towards the shore and then of the system.
halts. The boat weight 200 kg. How far is he from (B) the kinetic energy but not the linear momentum
the shore at the end of this time? of the system.
(A) 11.2 m (B) 13.8 m (C) linear momentum as well as kinetic energy of
(C) 14.3 m (D) 15.4 m the system.
Sol. (D) neither the linear momentum nor the kinetic
energy of the system.
Sol.

: 0744-2209671, 08003899588 | url : www.motioniitjee.com, : info@motioniitjee.com


Page # 50 CENTRE OF MASS

9. A small sphere is moving at a constant speed in a Sol.


vertical circle. Below is a list of quantities that could
be used to describe some aspect of the motion of
the sphere
I - kinetic energy
II - gravitational potential energy
III - momentum
Which of these quantities will change as this sphere
moves around the circle ?
(A) I and II only (B) I and III only
(C) III only (D) II and III only
Sol.

11. Two balls are thrown in air. The acceleration of


the centre of mass of the two balls while in air
(neglect air resistance)
(A) depends on the direction of the motion of the
balls
(B) depends on the masses of the two balls
(C) depends on the speeds of the two balls
(D) is equal to g
Sol.

10. Which of the following graphs represents the


graphical relation between momentum (p) and kinetic
energy (K) for a body in motion ?

ln p ln p

(A) (B)

ln K ln K

12. A bomb at rest explodes into two parts of


ln p masses m1 and m2 . If the momentums of the
two parts be p1 and p2, then their kinetic energies
(C) (D) none will be in the ratio of-
(A) m1 / m2 (B) m2 / m1
ln K (C) p1 / p2 (D) p2 / p1

Corporate Head Office : Motion Education Pvt. Ltd., 394 - Rajeev Gandhi Nagar, Kota-5 (Raj.)
CENTRE OF MASS Page # 51

Sol. Sol.

15. A bomb initially at rest explodes by it self


into three equal mass fragments. The velocities

of two fragments are ( 3 i + 2 j ) m/s and


13. Conservation of linear momentum is equivalent
(– i – 4 j ) m/s. The velocity of the third
to- 
(A) Newton's second law of motion fragment is (in m/s)-
(B) Newton's first law of motion
(C) Newton's third law of motion (A) 2 i + 2 j (B) 2 i – 2 j
(D) Conservation of angular momentum.
(C) – 2 i + 2 j (D) –2 i – 2 j
Sol.
Sol.

16. A stone of mass m1 moving with a uniform


speed v suddenly explodes on its own into two
fragments. If the fragment of mass m2 is at rest,
the speed of the other fragment is-
14. A body of mass m collides against a wall with
m1v m2 v
the velocity  and rebounds with the same speed. (A) (m  m ) (B) (m  m )
1 2 1 2
Its change of momentum is-
(A) 2 m (B) m m1v m1v
(C) (m  m ) (D) m
(C) – m (D) 0 1 2 2

: 0744-2209671, 08003899588 | url : www.motioniitjee.com, : info@motioniitjee.com


Page # 52 CENTRE OF MASS

Sol. Sol.

17. A nucleus of mass number A originally at rest


emits -particle with speed v. The recoil speed
of daughter nucleus is :
4v 4v
(A) (B)
A4 A4 19. A system of N particles is free from any external
v v
(C) (D) forces
A4 A4 (a) Which of the following is true for the magnitude
Sol.
of the total momentum of the system ?
(A) It must be zero
(B) It could be non-zero, but it must be constant
(C) It could be non-zero, and it might not be constant
(D) It could be zero, even if the magnitude of the
total momentum is not zero.
Sol.

18. There are some passengers inside a stationary


railway compartment. The track is frictionless. The
centre of mass of the compartment itself (without
the passengers) is C1, while the centre of mass of
the compartment plus passengers system is C2. If
the passengers move about inside the compartment
along the track.
(b) Which of the following must be true for the sum
(A) both C1 and C2 will move with respect to the
of the magnitudes of the momenta of the individual
ground
particles in the system ?
(B) neither C1 nor C2 will move with respect to the
(A) It must be zero
ground
(B) It could be non-zero, but it must be constant
(C) C1 will move but C2 will be stationary with respect
(C) It could be non-zero, and it might not be constant
to the ground
(D) The answer depends on the nature of the internal
(D) C2 will move but C1 will be stationary with respect
forces in the system
to the ground

Corporate Head Office : Motion Education Pvt. Ltd., 394 - Rajeev Gandhi Nagar, Kota-5 (Raj.)
CENTRE OF MASS Page # 53

Sol. (b) What is the average speed of the rail


car + bowling balls system some time after the
collision ?
(A) (M + Nm)v0/M
(B) Mv0/(Nm + M)
(C) Nmv0/M
(D) The speed cannot be determined because there
is not enough information
Sol.

21. A super-ball is to bounce elastically back and


forth between two rigid walls at a distance d from
each other. Neglecting gravity and assuming the
velocity of super-ball to be v 0 horizontally, the
average force being exerted by the super-ball on
each wall is :
1 mv 20 mv 20
20. An isolated rail car of mass M is moving along a (A) (B)
2 d d
straight, frictionless track at an initial speed v0. The
car is passing under a bridge when a crate filled 2mv 20 4mv 20
with N bowling balls, each of mass m, is dropped (C) (D)
d d
from the bridge into the bed of the rail car. The Sol.
crate splits open and the bowling balls bounce
around inside the rail car, but none of them fall out.
(a) Is the momentum of the rail car + bowling balls
system conserved in this collision ?
(A) Yes, the momentum is completely conserved
(B) Only the momentum component in the vertical
direction is conserved
(C) Only the momentum component parallel to the
track is conserved
(D) No components are conserved
Sol.

22. A force exerts an impulse I on a particle changing


its speed from u to 2u. The applied force and the
initial velocity are oppositely directed along the same
line. The work done by the force is
3 1
(A) Iu (B) Iu
2 2
(C) Iu (D) 2 Iu
Sol.

: 0744-2209671, 08003899588 | url : www.motioniitjee.com, : info@motioniitjee.com


Page # 54 CENTRE OF MASS

Sol.

23. A boy hits a baseball with a bat and imparts an


impulse J to the ball. The boy hits the ball again
with the same force, except that the ball and the
bat are in contact for twice the amount of time as
in the first hit. The new impulse equals.
(A) half the original impulse
(B) the original impulse
(C) twice the original impulse
(D) four times the original impulse
Sol.

25. A ball strikes a smooth horizontal ground at an


angle of 45° with the vertical. What cannot be the
possible angle of its velocity with the vertical after
the collision. (Assume e  1).
(A) 45° (B) 30°
(C) 53° (D) 60°
Sol.

24. A system of two blocks A and B are connected


by an inextensible massless strings as shown. The
pulley is masselss and frictionless. Initially the system
is at rest when, a bullet of mass 'm' moving with a
velocity 'u' as shown hits the block 'B' and gets
embedded into it. The impulse imparted by tension
force to the block of mass 3m is :

m
u
m B
26. In an inelastic collision-
A 3m (A) momentum is conserved but kinetic energy is not
5mu 4mu (B) momentum is not conserved but kinetic energy
(A) (B) is conserved
4 5
2mu 3mu (C) neighter momentum nor kinetic energy is conserved
(C) (D) (D) both the momentum and kinetic energy are
5 5
conserved

Corporate Head Office : Motion Education Pvt. Ltd., 394 - Rajeev Gandhi Nagar, Kota-5 (Raj.)
CENTRE OF MASS Page # 55

Sol. Sol.

27. Two perfectly elastic balls of same mass m 29. A ball hits the floor and rebounds after an
are moving with velocities u1 and u2. They collide inelastic collision . In this case-
elastically n times. The kinetic energy of the (A) the momentum of the ball just after the
system finally is : collision is the same as that just before the
1m 2 1m 2 collision
(A) u1 (B) (u1  u22 )
2 u 2 u (B) the mechanical energy of the ball remains the
same in the collision
1 1
(C) m(u12  u 22 ) (D) mn(u12  u 22 ) (C) the total momentum of the ball and the earth
2 2 is conserved.
Sol.
(D) the total energy of the ball and the earth is
conserved
Sol.

28. When two bodies collide elastically, then


(A) ki netic energy of the system alone is
conserved
(B) only momentum is conserved
(C) both energy and momentum are conserved
(D) neighter energy nor momentum is conserved

: 0744-2209671, 08003899588 | url : www.motioniitjee.com, : info@motioniitjee.com


Page # 56 CENTRE OF MASS

30. Six steel balls of identical size are lined up Sol.


long a straight frictionless groove. Two similar
balls moving with a speed V along the groove
collide with this row on the extreme left hand
then-

(A) all the balls will start moving to the right


with speed 1/8 each
(B) all the six balls initially at rest will move on
with speed V/6 each and two identical balls will
come to rest
(C) two balls from the extreme right end will
move on with speed V each and the remaining
balls will remain at rest
(D) one ball from the right end will move on with
speed 2V, the remaining balls will be at rest.
Sol.
32. Two balls A and B having masses 1 kg and 2 kg,
moving with speeds 21 m/s and 4 m/s respectively
in opposite direction, collide head on. After collision
A moves with a speed of 1 m/s in the same direction,
then the coefficient of restitution is
(A) 0.1 (B) 0.2
(C) 0.4 (D) None
Sol.

31. The bob of a simple pendulum of length l


dropped from a horizontal position strikes a block
of the same mass, placed on a horizontal table
(frictionless) as shown in the diagram, the block
shall have kinetic energy-
33. A truck moving on horizontal road east with
velocity 20ms–1 collides elastically with a light ball
moving with velocity 25 ms–1 along west. The velocity
of the ball just after collision
(A) 65 ms–1 towards east
(B) 25 ms–1 towards west
(A) Zero (B) mgl. (C) 65 ms–1 towards west
(C) 1/2 mgl. (D) 2mgl. (D) 20 ms–1 towards east

Corporate Head Office : Motion Education Pvt. Ltd., 394 - Rajeev Gandhi Nagar, Kota-5 (Raj.)
CENTRE OF MASS Page # 57

Sol. Sol.

36. Two billiard balls undergo a head-on collision.


Ball 1 is twice as heavy as ball 2. Initially, ball 1
moves with a speed v towards ball 2 which is at
34. A sphere of mass m moving with a constant rest. Immediately after the collision, ball 1 travels
velocity hits another stationary sphere of the same at a speed of v/3 in the same direction. What type
mass, if e is the coefficient of restitution, then ratio of collision has occured ?
of speed of the first sphere to the speed of the (A) inelastic
second sphere after collision will be (B) elastic
(C) completely inelastic
 1– e  1 e 
(A)  1  e  (B)  1 – e  (D) Cannot be determined from the information given
Sol.
 e  1  e – 1
(C)  e – 1 (D)  e  1

Sol.

37. A ball is dropped from a height h. As is bounces


off the floor, its speed is 80 percent of what it was
just before it hit the floor. The ball will then rise to a
height of most nearly
(A) 0.80 h (B) 0.75 h
(C) 0.64 h (D) 0.50 h
Sol.
35. Three blocks are initially placed as shown in the
figure. Block A has mass m and initial velocity v to
the right. Block B with mass m and block C with mass
4 m are both initially at rest. Neglect friction. All
collisions are elastic. The final velocity of block A is
V
A B C
m m 4m

(A) 0.60 v to the left (B) 1.4 v to the left


(C) v to the left (D) 0.4 v to the left

: 0744-2209671, 08003899588 | url : www.motioniitjee.com, : info@motioniitjee.com


Page # 58 CENTRE OF MASS

38. A ball is thrown vertically downwards with Sol.


velocity 2gh from a height h. After colliding with
the ground it just reaches the starting point.
Coefficient of restitution is
(A) 1 / 2 (B) 1/2 (C) 1 (D) 2
Sol.

41. An object of mass 5 kg and speed 10 ms-1


explodes into two pieces of equal mass. One
piece comes to rest. The kinetic energy added to
the system during the explosion is-
(A) Zero. (B) 50 J.
(C) 250 J. (D) 500 J.
Sol.

39. A ball is dropped from height 5m. The time after


which ball stops rebounding if coefficient of restitution
between ball and ground e = 1/2, is
(A) 1 sec (B) 2 sec (C) 3 sec (D) infinite
Sol.

42. Two particles of same mass m moving with


velocities u1 and u2 collide perfectly inelastically.
The loss of energy would be-
(A) 1/2m (u1 – u2 )2
(B) 1/4m (u1 – u2 )2
(C) m (u1 – u2 )2 .
(D) 2m (u1 – u1 )2 .
Sol.
40. A neutron travelling with a velocity v and
K.E. E. collides perfectly elastically head on with
the nucleus of an atom of mass number A at
rest. The fraction of total energy retained by
neutron is-

FG A  1IJ 2
F A  1IJ
(B) G
2

(A)
H A  1K H A  1K
FG A  1IJ 2
F A  1IJ
(D) G
2

(C)
H AK H AK
Corporate Head Office : Motion Education Pvt. Ltd., 394 - Rajeev Gandhi Nagar, Kota-5 (Raj.)
CENTRE OF MASS Page # 59

43. A ball of mass m moving with a speed u Sol.


undergoes a head-on elastic collision with a ball
of mass nm initially at rest. The fraction of the
incident energy transferred to the heavier ball is-

n n
(A) (B)
1 n (1  n) 2

2n 4n
(C) 2 (D)
(1  n) (1  n) 2

Sol.

45. A metal ball hits a wall and does not rebound


whereas a rubber ball of the same mass on
hitting the wall the same velocity rebounds back.
It can be concluded that -
(A) metal bal l sufferes greater change i n
momentum
(B) rubber bal l suffers greater change i n
momentum.
(C) the initial momentum of metal ball is greater
than the initial momentum of rubber ball.
(D) both suffer same change in momentum.
Sol.

44. Which of the following does not hold when


two particles of masses m1 and m2 undergo elastic
collision?
(A) when m1 = m2 and m2 is stationary, there is 46. Before a rubber ball bounces off from the
maximum transfer of kinetic energy in head an floor the ball is in contact with the floor for a
collision fracti on of second. Which of the fol lowi ng
(B) when m1 = m2 and m2 is stationary, there is statements are correct-
maximum transfer of momentum in head on collision (A) conservation of energy is not valid during this
(C) when m1 >> m2 and m2 is stationary, after period
head on collision m2 moves with twice the velocity (B) conservation of energy is valid during this
of m1 . period
(D) when the collision is oblique and m1 = m2 with (C) as ball compressed kinetic energy is converted
m2 stationary, after the collision the particle move compressed potential energy
in opposite directions. (D) B and C both

: 0744-2209671, 08003899588 | url : www.motioniitjee.com, : info@motioniitjee.com


Page # 60 CENTRE OF MASS

Sol. 49. Two billiard balls undergo a head-on collision.


Ball 1 is twice as heavy as ball 2. Initially, ball 1
moves with a speed v towards ball 2 which is at
rest. Immediately after the collision, ball 1 travels
at a speed of v/3 in the same direction. What type
of collision has occured ?
(A) inelastic
(B) elastic
(C) completely inelastic
(D) Cannot be determined from the information given
Sol.
47.A ball of 0.1kg strikes a wall at right angle
with a speed of 6 m/s and rebounds along its
original path at 4 m/s. The change in momentum
in Newton- sec is-
(A) 103 (B) 102 (C) 10 (D) 1
Sol.

50. The diagram shows the velocity - time graph for


two masses R and S that collided elastically. Which
of the following statements is true ?

V(ms–1)
1.2 S
48. A block of mass m starts from rest and slides 0.8
R
down a frictionless semi-circular track from a height 0.4
h as shown. When it reaches the lowest point of
the track, it collides with a stationary piece of putty 1 2 3 4 t(s)
also having mass m. If the block and the putty stick I. R and S moved in the same direction after the
together and continue to slide, the maximum height collision.
that the block-putty system could reach is II. The velocities of R and S were equal at the mid
time of the collision.
III. The mass of R was greater than mass of S.
h (A) I only (B) II only
(C) I and II only (D) I, II and III
(A) h/4 (B) h/2 Sol.
(C) h (D) independent of h
Sol.

Corporate Head Office : Motion Education Pvt. Ltd., 394 - Rajeev Gandhi Nagar, Kota-5 (Raj.)
CENTRE OF MASS Page # 61

Exercise - II JEE ADVANCED


1. A body has its centre of mass at the origin. The Sol.
x-coordinates of the particles
(A) may be all positive
(B) may be all negative
(C) may be all non-negative
(D) may be positive for some cases and negative in
other cases
Sol.

4. Consider following statements


[1] CM of a uniform semicircular disc of radius R = 2R/ from
the centre
[2] CM of a uniform semicircular ring of radius R = 4R/3
from the centre
[3] CM of a solid hemisphere of radius R = 4R/3
from the centre
2. An object comprises of a uniform ring of radius R [4] CM of a hemisphere shell of radius R = R/2 from
and its uniform chord AB (not necessarily made of the centre
the same material) as shown. Which of the following Which statements are correct?
can not be the centrey of mass of the object. (A) 1, 2, 4 (B) 1, 3, 4
B
(C) 4 only (D) 1, 2 only
Sol.

x
A

(A) (R/3, R/3) (B) (R3, R/2)


(C) (R/4, R/4) (D) (R / 2, R / 2 )
Sol.

5. If the external forces acting on a system have


zero resultant, the centre of mass
(A) must not move (B) must not accelerate
(C) may move (D) may accelerate
Sol.

3. In which of the following cases the centre of


mass of a an rod is certainly not at its centre ?
(A) the density continuously increases from left to
right
(B) the density continuously decreases from left to
right
(C) the density decreases from left to right upto
the centre and then increase
(D) the density increases from left to right upto the
centre and then decreases

: 0744-2209671, 08003899588 | url : www.motioniitjee.com, : info@motioniitjee.com


Page # 62 CENTRE OF MASS

Question No. 6 to 12 (7 Questions) 8. The maximum height h attained by the particle is


A particle of mass m moving horizontal with v0 strikes 2 2
a smooth wedge of mass M, as shown in figure.  m  v0  m  v0
(A)   (B)  
After collision, the ball starts moving up the inclined m  M 2g M 2g
face of the wedge and rises to a
height h.  M  v 20
(C)   (D) none of these.
m  M 2g
Sol.

h
m v0 M
6. The final velocity of the wedge v2 is

mv 0 mv 0
(A) (B)
M Mm
(C) v0 (D) insufficient data
Sol.

9. Identify the correct statement(s) related to the


situation when the particle starts moving downward.
(A) The centre of mass of the system remains sta-
tionary
(B) Both the particle and the wedge remain station-
ary with respect to centre of mass
(C) When the particle reaches the horizontal sur-
face its velocity relative to the wedge is v0
(D) None of these
Sol.
7. When the particle has risen to a height h on the
wedge, then choose the correct alternative(s)
(A) The particle is stationary with respect to ground
(B) Both are stationary with respect to the centre
of mass
(C) The kinetic energy of the centre of mass re-
mains constant
(D) The kinetic energy with respect to centre of
mass is converted into potential energy
Sol.

Corporate Head Office : Motion Education Pvt. Ltd., 394 - Rajeev Gandhi Nagar, Kota-5 (Raj.)
CENTRE OF MASS Page # 63

10. Suppose the particle when reaches the hori- 12. Choose the correct statement related to the
zontal surfaces, its velocity with respect to ground wedge M
is v1 and that of wedge is v2. Choose the correct
 4 m2 
statement (s) (A) Its kinetic energy is K f    gh
 m  M
 2m 
(B) v 2    v0
m  M
v2 (C ) It s gai n in ki ne ti c energy is
v1 m M  4 mM   1 
K     mv 20 
 (m  M) 2   2 
(A) mv1 = Mv2 (B) Mv2 – mv1 = mv0 (D) Its velocity is more that the velocity of centre
(C) v1 + v2 = v0 (D) v1 + v2 < v0 of mass
Sol. Sol.

11. Choose the correct statement(s) related to par-


ticle m
mM 
(A) Its kinetic energy is K f    gh
 m  M
 M  m
(B) v1  v 0  M  m  13. Two blocks A (5kg) and B(2kg) attached to the
ends of a spring constant 1120 N/m are placed on a
(C) The ratio of its final kinetic energy to its initial
2
smooth horizontal plane with the spring undeformed.
Kf  M  Simultaneously velocities of 3m/s and 10m/s along the
kinetic energy is  
K i  m  M line of the spring in the same direction are imparted
(D) It moves opposite to its initial direction of mo- to A and B then
tion
3m/s 10m/s
Sol.
A 5 2 B
(A) when the extension of the spring is maximum the
velocities of A and B are zero.
(B) the maximum extension of the spring is 25 cm
(C) maximum extension and maximum compression
occur alternately.
(D) the maximum compression occur for the first time

after sec.
56

: 0744-2209671, 08003899588 | url : www.motioniitjee.com, : info@motioniitjee.com


Page # 64 CENTRE OF MASS

Sol. 15. A ball moving with a velocity v hits a massive


wall moving towards the ball with a velocity u. An
elastic impact lasts for a time t.
(A) The average elastic force acting on the ball is
m(u  v)
t
(B) The average elastic force acting on the ball is
2m(u  v )
t
(C) The kinetic energy of the ball increases by 2mu
(u + v)
(D) The kinetic energy of the ball remains the same
after the collision.
Sol.

14. Two identical balls are interconnected with a


massless and inextensible thread. The system is in
gravity free space with the thread just taut. Each
ball is imparted a velocity v, one towards the other
ball and the other perpendicular to the first, at t = 16. A particle moving with kinetic energy = 3 joule
0. Then, makes an elastic head on collision with a stationary
(A) the thread will become taut at t = (L/v) particle when has twice its mass during the impact.
(A) The minimum kinetic energy of the system is 1
(B) the thread will become taut at some time t < (L/
joule
v).
(B) The maximum elastic potential energy of the
(C) the thread will always remain taut for t > (L/v)
system is 2 joule.
(D) the kinetic energy of the system will always
(C) Momentum and total kinetic energy of the sys-
remain mv2.
tem are conserved at every instant.
Sol. (D) The ratio of kinetic energy to potential energy
of the system first decreases and then increases.
Sol.

Corporate Head Office : Motion Education Pvt. Ltd., 394 - Rajeev Gandhi Nagar, Kota-5 (Raj.)
CENTRE OF MASS Page # 65

17. Two balls A and B having masses 1 kg and 2 kg, Sol.


moving with speeds 21 m/s and 4 m/s respectively
in opposite direction, collide head on. After collision
A moves with a speed of 1 m/s in the same direc-
tion, then correct statements is :
(A) The velocity of B after collision is 6 m/s opposite
to its direction of motion before collision.
(B) The coefficient of restitution is 0.2.
(C) The loss of kinetic energy due to collision is 200
J.
(D) The impulse of the force between the two balls
is 40 Ns.
Sol.

19. In an inelastic collision,


(A) the velocity of both the particles may be same
after collision.
(B) kinetic energy is not conserved
(C) linear momentum of the system is conserved.
(D) velocity of separation will be less than velocity
of approach.
Sol.

18. The diagram to the right shown the velocity-


time graph for two masses R and S that collided
elastically. Which of the following statements is true?
v(ms–1)
1.2 S
R
0.8
0.4
20. In a one-dimensional collision between two par-

1 2 3 4 ticles, their relative velocity is v1 before the colli-
t (milli sec) 
(I) R and S moved in the same direction after the sion and v 2 after the collision
collision.  
(A) v1  v 2 if the collision is elastic
(II) Kinetic energy of the system (R & S) is minimum  
at t = 2 milli sec. (B) v1  – v 2 if the collision is elastic
 
(III) The mass of R was greater than mass of S. (C) | v 2 | | v 1| in all cases
(A) I only (B) II only  
(D) v1  –kv 2 in all cases, where k  1
(C) I and II only (D) I, II and III

: 0744-2209671, 08003899588 | url : www.motioniitjee.com, : info@motioniitjee.com


Page # 66 CENTRE OF MASS

Sol. Sol.

(b) What happens to the speed of the rail car as


21. A set of n-identical cubical blocks lie at rest the sand pours out?
parallel to each other along a line on a smooth hori- (A) The car begins to roll faster
zontal surface. The separation between the near (B) The car maintains the same speed
surfaces of any two adjacent blocks is L. The block (C) The car begins to slow down
at one end is given a speed V towards the next one (D) The problem cannot be solved since momentum
at time t = 0. All collision are completely inelastic, is not conserved
then Sol.
L
(A) The last block starts moving at t = n(n–1)
2V
L
(B) The last block starts moving at t = (n – 1)
V
(C) The centre of mass of the system will have a
final speed v/n
(D) The centre of mass of the system will have a
final speed v
Sol.

23. A semicircular portion of radius 'r' is cut from a


uniform rectangular plate as shown in figure. The
distance of centre of mass 'C' of remaining plate,
from point 'O' is
r

22. An isolated rail car original moving with speed v0


on a straight, frictionless, level track contains a large
amount of sand. A release value on the bottom of 2r O
the car malfunctions, and sand begins to pour out C
straight down relative to the rail car.
(a) Is momentum conserved in this process?
(A) The momentum of the rail car along is conserved
(B) The momentum of the rail car + sand remaining 2r 3r
(A) (B)
within the car is conserved ( 3 – ) 2(4 – )
(C) The momentum of the rail car + all of the sand,
both inside and outside the rail car, is conserved 2r 2r
(D) None of the three previous systems have mo- (C) (D)
( 4  ) 3(4 – )
mentum conservation

Corporate Head Office : Motion Education Pvt. Ltd., 394 - Rajeev Gandhi Nagar, Kota-5 (Raj.)
CENTRE OF MASS Page # 67

Sol. 25. In the figure shown a hole of radius 2 cm is made


in semicircular disc of radius 6 at a distance 8 cm
from the centre C of the disc. The distance of the
centre of mass of this system from point C is

2cm
8cm

(A) 4 cm (B) 8 cm
(C) 6 cm (D) 12 cm
Sol.

24. From a circle of radius a, an isosceles right angled


triangle with the hypotenuse as the diameter of the
circle is removed. The distance of the centre of
gravity of the remaining position from the centre of
the circle is 26. Centre of mass of two thin uniform rods of same
(  – 1)a length but made up of different materials & kept as
(A) 3( – 1)a (B) shown, can be, if the meeting point is the origin of
6
a a co-ordinates
(C) (D) y
3(  – 1) 3(   1)
Sol.
L

x
L
(A) (L/2, L/2) (B) (2L/3, L/2)

(C) (L/3, L/3) (D) (L/3, L/6)


Sol.

: 0744-2209671, 08003899588 | url : www.motioniitjee.com, : info@motioniitjee.com


Page # 68 CENTRE OF MASS

Question No. 27 to 28 (2 questions) Question No. 29 to 32


A uniform chain of length 2L is hanging in equilibrium A small ball B of mass m is suspended with light
position, if end B is given a slightly downward inelastic string of length L from a block A of same
displacement the imbalance causes an acceleration. mass m which can move on smooth horizontal surface
Here pulley is small and smooth & string is inextensible as shown in the figure. The ball is displaced by angle
 from equilibrium position & then released.

 L
L
A B
u=0
27. The acceleration of end B when it has been
displaced by distance x, is B
x 2x x
(A) g (B) g (C) g (D) g 29. The displacement of block when ball reaches
L L 2
Sol. the equilibrium position is
L sin
(A) (B) L sin 
2
(C) L (D) none of these
Sol.

28. The velocity v of the string when it slips out of 30. Tension in string when it is vertical, is
the pulley (height of pulley from floor > 2L) (A) mg (B) mg(2 – cos )
gL (C) mg (3 – 2 cos) (D) none of these
(A) (B) 2gL Sol.
2
(C) gL (D) none of these
Sol.

Corporate Head Office : Motion Education Pvt. Ltd., 394 - Rajeev Gandhi Nagar, Kota-5 (Raj.)
CENTRE OF MASS Page # 69

31. Maximum velocity of block during subsequent Question No. 33 to 37 (5 questions)


motion of the system after release of ball is Two persons of mass m1 and m2 are standing at the
(A) [gl(1 – cos )]1/2 two ends A and B respectively, of a trolley of mass
(B) [2gl(1 – cos )]1/2 M as shown.
(C) [glcos]1/2 m1 m2
(D) informations are insufficient to decide
Sol.
M
A B

L
33. When the person standing at A jumps from the
trolley towards left with urel with respect to the
trolley, then
(A) the trolley moves towards right
m1urel
(B) the trolley rebounds with velocity m  m  M
1 2
(C) the centre of mass of the system remains stationary
(D) all the above
Sol.

32. The displacement of centre of mass of A + B


system till the string becomes vertical is
L
(A) zero (B) (1 – cos )
2
L
(C) (1 – sin ) (D) none of these
2
Sol.

: 0744-2209671, 08003899588 | url : www.motioniitjee.com, : info@motioniitjee.com


Page # 70 CENTRE OF MASS

34. When only the person standing at B jumps from 36. When both the persons jump simultaneously with
the trolley towards right while the person at A keeps urel with respect to the trolley, then the velocity of
standing, then the trolley is
(A) the trolley moves towards left |m1  m 2 |urel |m1  m 2 |urel
m2urel (A) m  m  M (B)
(B) the trolley mones with velocity 1 2 M
m1  m 2  M
m1urel m 2urel
(C) the centre of mass of the system remains (C) m  M  m  M (D) none of these
2 1
stationary
Sol.
(D) all the above
Sol.

37. Choose the incorrect statement, if m1 = m2 = m


and both the persons jump one by one, then
35. When both the persons jump simultaneously with
(A) the centre of mass of the system remains
same speed then
stationary
(A) the centre of fmass of the systyem remains
(B) the final velocity of the trolley is in the direction
stationary
of the person who jumps first
(B) the trolley remains stationary
(C) t he fi nal v el oc i t y of t he t rol l ey i s
(C) the trolley moves toward the end where the
person with heavier mass is standing  murel murel 
  
(D) None of these  M  m M  2m 
Sol. (D) none of these
Sol.

Corporate Head Office : Motion Education Pvt. Ltd., 394 - Rajeev Gandhi Nagar, Kota-5 (Raj.)
CENTRE OF MASS Page # 71

38. In the diagram shown, no friction at any contact 40. A parallel beam of particles of mass m moving
surface. Initially, the spring has no deformation. What with velocity v impinges on a wall at an angle  to
will be the maximum deformation in the spring ? its normal. The number of particles per unit volume
Consider all the strings to be sufficiency large. in the beam is n. If the collision of particles with the
Consider the spring constant to be K wall is elastic, then the pressure exerted by this
beam on the wall is
(A) 2 mn v2 cos  (B) 2 mn v2 cos2 
F (C) 2 mn v cos  (D) 2 mn v cos2 
Sol.
2M M

(A) 4F / 3K (B) 8F / 3K
(C) F / 3K (D) none
Sol.

39. In the figure (i), (ii) & (iii) shown the objects A, 41. As shown in the figure a body of mass m moving
B & C are of same mass. String, spring & pulley are vertically with speed 3 m/s hits a smooth fixed
massless. C strikes B with velocity ‘u’ in each case inclined plane and rebounds with a velocity vf in the
and sticks to it. The ratio of velocity of B in case (i) horizontal direction. If  of inclined is 30°, the
to (ii) to (iii) is velocity vf will be

(iii) vf
(i) (ii)
m
C C
A B
A B A B
C
(A) 1 : 1 : 1 (B) 3 : 3 : 2
(A) 3 m/s (B) 3 m/s
(C) 3 : 2 : 2 (D) none of these
Sol. (C) 1 / 3 m/s
(D) this is not possible
Sol.

: 0744-2209671, 08003899588 | url : www.motioniitjee.com, : info@motioniitjee.com


Page # 72 CENTRE OF MASS

42. Two massless string of length 5 m hang from


, Sol.
the ceiling very near to each other as shown in the
figure. Two balls A and B of masses 0.25 kg and 0.5
kg are attached to the string. The ball A is released
from rest at a height 0.45 m as shown in the figure.
The collision between two balls is completely elastic
Immediately after the collision. the kinetic energy of
ball B is 1 J. The velocity of ball A just after the
collision is

A
0.45m
B

(A) 5 ms–1 to the right (B) 5 ms–1 to the left


(C) 1 ms–1 to the right (D) 1 ms–1 to the left
Sol.

44. In the above, suppose that the smaller ball does


not stop after collision, but continues to move
downwards with a speed = v0/2, after the collision.
Then, the speed of each bigger ball after collision is
(A) 4 v 0 / 5 (B) 2v 0 / 5

(C) v 0 / 2 5 (D) none


Sol.

43. In the figure shown, the two identical balls of


mass M and radius R each, are placed in contact
with each other on the frictionless horizontal surface.
The third ball of mass M and radius R/2, is coming
down vertically and has a velocity = v0 when it
simultaneously hits the two balls and itself comes to
rest. The each of the two bigger balls will move
after collision with a speed equal to

(A) 4 v 0 / 5 (B) 2v 0 / 5

(C) v 0 / 5 (D) none

Corporate Head Office : Motion Education Pvt. Ltd., 394 - Rajeev Gandhi Nagar, Kota-5 (Raj.)
CENTRE OF MASS Page # 73

45. A body of mass ‘m’ is dropped from a height of 47. A ball is projected from ground with a velocity V
‘h’. Simultaneously another body of mass 2m is thrown at an angle  to the vertical. On its path it makes an
up vertically with such a velocity v that they collide elastic collision with a vertical wall and returns to
at the height h/2. If the collision is perfectly inelastic, ground. The total time of flight of the ball is
the velocity at the time of collision with the ground
2v sin 2v cos
will be (A) (B)
g g
5gh
(A) (B) gh
4 v sin2 v cos
(C) (D)
10gh g g
gh
(C) (D) Sol.
4 3
Sol.

46. In a smooth stationary cart of length d, a small


block is projected along it’s length with velocity v
towards front. Coefficient of restitution for each
collision is e. The cart rests on a smooth ground and
can move freely. The time taken by block to come
to rest w.r.t. cart is
d

ed ed
(A) (B)
(1 e) v (1 e) v

d
(C) (D) inifinite
e
Sol.

: 0744-2209671, 08003899588 | url : www.motioniitjee.com, : info@motioniitjee.com


Page # 74 CENTRE OF MASS

Exercise - III (JEE ADVANCED)

1. The mass of an uniform ladder of length l increases 3. A thin sheet of metal of uniform thickness is cut into the
uniformly from one end A to the other end B, shape bounded by the line x = a and y = ± k x2, as shown.
(a) Form an expression for linear mass density as a function Find the coordinates of the centre of mass.
y
of distance x from end A where linear mass density 0. The
density at one end being twice that of the other end. y=kx2
(b) find the position of the centre of mass from end A.
x
Sol. a
y=–kx2

Sol.

2. Find the distance of centre of mass of a uniform plate 4. Two balls of equal masses are projected upwards simulta-
having semicircular inner and other boundaries of radii a and neously, one from the ground with speed 50 m/s and other
b from the centre O. from a 40m high tower with initial speed 30 m/s. Find the
maximum height attained by their centre of mass.
Sol.

a
b
O
Sol.

Corporate Head Office : Motion Education Pvt. Ltd., 394 - Rajeev Gandhi Nagar, Kota-5 (Raj.)
CENTRE OF MASS Page # 75

5. In the figure shown, when the persons A and B exchange 7. A small cube of mass m slides down a circular path of
their positions, then radius R cut into a large block of mass M. M rests on a table
and both blocks move without friction. The blocks initially
A B are at rest and m starts from the top of the path. Find the
velocity v of the cube as it leaves the block.
m1 M m2 m
R
2m M
m1=50kg, m2 = 70kg, M = 80 kg
Sol.
(i) the distance moved by the centre of mass of the system
is__________.
(ii) the plank moves towards_________
(iii) the distance moved by the plank is _________.
(iv) the distance moved by A with respect to ground is
____________
(v) the distance moved by B with respect to ground is
____________.
Sol.

8. A (trolley + child) of total mass 200 kg is moving with a


uniform speed of 36 km/h on a frictionless track. The child of
mass 20 kg starts running on the trolley from one end to the
other (10 m away) with a speed of 10 ms–1 relative to the
trolley in the direction of the trolley's motion and jumps out of
the trolley with the same relative velocity. What is the final
speed of the trolley? How much has the trolley moved from
the time the child begins to run?
Sol.

6. In the arrangement, mA = 2 kg and mB = 1 kg. String is


light and inextensible. Find the acceleration of centre of mass
of both the blocks. Neglect friction everywhere.
Sol.

9. In the figure shown the spring is compressed by ‘x0’ and


released. Two blocks ‘A’ and ‘B’ of masses ‘m’ and ‘2m’
respectively are attached at the ends of the spring. Blocks
are kept on a smooth horizontal surface and released.
k
A B

(a) Find the speed of block A by the time compression of the


spring is reduced to x0/2.
(b) Find the work done by the spring on ‘A’ by the time com-
pression of the spring reduced to x0/2.

: 0744-2209671, 08003899588 | url : www.motioniitjee.com, : info@motioniitjee.com


Page # 76 CENTRE OF MASS

Sol. Sol.

12. A man hosing down his driveway hits the wall by mis-
take. Knowing that the velocity of the stream is 25 m/s and
the cross-sectional area of the stream is 300 mm2, deter-
mine the force exerted on the wall. Assume that
streamstrikes wall horizontally and after striking the wall,
stream comes to rest. Also find the pressure exerted on the
wall by stream.
10. The figure showns the force versus time graph for a par-
ticle.
(i) Find the change in momentum p of the particle
(ii) Find the average force acting on the particle Sol.

100 N

t(a)
0 0.2 0.4
Sol.

13. A bullet of mass m strikes an obstruction and deviates off


at 60° to its original direction. If its speed is also changed
from u to v, find the magnitude of the impulse acting on the
bullet.
11. A force F acts on an object (mass = 1kg) which is initially Sol.
at rest as shown in the figure. Draw the graph showing the
momentum of the object varying during the time for which
the force acts.

F(N)
5

50 100 t(ms)

Corporate Head Office : Motion Education Pvt. Ltd., 394 - Rajeev Gandhi Nagar, Kota-5 (Raj.)
CENTRE OF MASS Page # 77

14. A n eutro intially at rest, decays into a proton, an elec- 16. A particle A of mass 2 kg lies on the edge of a table of
tron and an antineutrio. The ejected electron has a momen- height 1m. It is connected by a light inelastic string of length
tum of p1 = 1.4 × 10–28 kg-m/s and the antineutrino p2 = 6.5 × 0.7 m to a second particle B of mass 3 kg which is lying on
10–27 kg-m/s. Find the recoil speed of the proton (a) if the the table 0.25 m from the edge (line joining A & B is perpen-
electron and the antineutrino are ejected along the the same dicular to the edge). If A is pushed gently so that it start
direection and (b) if they are ejected along perpendicular di- falling from table then, find the speed of B when it starts to
rection. Mass of the proton mp = 1.67 × 10–27 kg. move. Also find the imulsive tension in the string at that mo-
Sol. ment.
Sol.

17. Two particles, each of mass m, are connected by a light


inextensible string of length 2l. Initially they lie on a smooth
horizontal table at points A and B distant l apart. The particle
15. A steel ball of mass 0.5 kg is dropped from a height of 4 at A is projected across the table with velocity u. Find the
m on to a horizontal heavy steel slab. The collision is elastic speed with which the second particle begins to move if the
and the ball rebounds to its original height. direction of u is, (a) along BA, (b) at an angle of 120° with
(a) Calculate the impulse delivered to the ball during impact. AB, (c) perpendicular to AB. In each case calculate (in terms
(b) If the ball is in contact with the slab for 0.002 s, find the of m and u) the impulsive tension in the string.
average reaction force on the ball during impact. Sol.
Sol.

: 0744-2209671, 08003899588 | url : www.motioniitjee.com, : info@motioniitjee.com


Page # 78 CENTRE OF MASS

18. Two particle P of mass 2m and Q of mass m are sub- Sol.


jected to mutual force of attraction and no other act on them.
At time t = 0, P is at rest at a fixed O and Q is directly
moving away from O with a speed 5 u. At a later instant when
t = T before any collision has taken place Q is moving to-
wards O with speed u.
(a) Find in terms of m and u the total work done by the forces
of attraction during the time interval 0  t  T.
(b) At the instantt = T, impulses of magnitude J and K and
applied to P and Q bringing them to rest. Find the values of J
and K
Sol.

21. Three carts move on a frictioless track with inertias


and velocities as shown. The carts collide and stick together
after successive collisions.

m1 = 2 kg m2 = 1 kg m3 = 2 kg
v1 = 1 m/s v2 = 1 m/s v3 = 2 m/s

A B C

(a) Find loss of mechanical energy when B & C stick to-


gether.
(b) Find magnitude of impulse experienced by A when it sticks
to combined mass (B & C).
Sol.

19. A block of mass m moving with a velocity v, enters a


region where it starts colliding with the stationary dust par-
ticles. If the desnsity of dust particles is , & all colliding
particle stick to its front surface of cross-sectional area A.
The velocity of block after it has covered a distance x in this
region is __________________.
Sol.

22. A sphere of mass m1 in motion hits directly another sphere


of mass m2 at rest and sticks to it, the total kinetic energy
after collision is 2/3 of their total K.E. before collision. Find
the ratio of m1 : m2.
Sol.

20. A football approaches a player at v = 12 m/s. At what


speed u and in which direction should the player’s foot move
in order to stop the ball upon contact? Assume that the mass
of the foot is much greater than that of the ball and that the
collision is elastic.

Corporate Head Office : Motion Education Pvt. Ltd., 394 - Rajeev Gandhi Nagar, Kota-5 (Raj.)
CENTRE OF MASS Page # 79

23. A body is thrown vertically upwards from ground with a 25. A ball is thrown horizontally from a cliff 10 m high with a
speed of 10 m/s. If coefficient of restitution of ground, e = 1/ velocity of 10 m/s. It strikes the smooth ground and rebounds
2. Find as shown. The coefficient of restitution e for collision with the
(a) the total distance travelled by the time it almost stops. ground e = 1/ 2 . Find
(b) time elapsed (after the ball has been thrown) when it is at 10 m/s
its subsequent maximum height for the third time.
Sol.

(a) velocity of ball just before striking ground.


(b) angle of velocity vector with horizontal before striking.
(c) angle of velocity vector with horizotal after striking.
(d) range of ball after first collision.
Sol.

24. A ball of mass 'm' is suspended by a massless string of


length '' from a fixed point. A ball of mass 2m strikes in the
direction of  = 45° from horizontal and sticks to it.
(a) What should be the initial velocity of 2m so that system

deflects by  =
2
(b) If at  = 60° the stirng is cut then what will be the velocity
at highest point of trajectory.


2m

45° m 26. A wedge free to move of mass 'M' has one face making
Sol. an angle  with horizotnal and is resting on a smooth rigid
floor. A particle of mass 'm' hits the inclined face of the wedge
with a horizontal velocity v0. It is observed that the particle
rebounds in vertical direction after impact. Neglect friction
between particle and the wedge & take M = 2m, v0 = 10 m/
s, tan  = 2, g = 10 m/s2

 M

(a) Determine the coefficient of restitution for the impact.


(b) Assume that the inclined face of the wedge is sufficiently
long so that the particle hits the same face once more during
its downward motion. Calculate the time elapsed between
the two impacts.

: 0744-2209671, 08003899588 | url : www.motioniitjee.com, : info@motioniitjee.com


Page # 80 CENTRE OF MASS

Sol. Sol.

(b) The minimum initial speed of the bullet if the block and
the bullet are to successfully execute a complete ride on the
loop.
Sol.
27. A sphere A is released from rest in the position shown
and strikes the block B which is at rest. If e = 0.75 between
A and B and k = 0.5 between B and the support, determine

(a) the velocity of A just after the impact


A
2kg
90°
1.5 m

4 kg
B
(b) the maximum displacement of B after the impact. 29. A Cart of total mass M0 is at rest on a rough horizontal
Sol. road. It ejects bullets at rate of  kg/s at an angle  with the
horizontal and at velocity 'u' (constant) relative to the cart.
The coefficient of friction between the cart and the ground is
. Find the velocity of the cart in terms of time 't'. The cart
moves with sliding.

Sol.

28. A small block of mass 2m initially rests at the bottom of


a fixed circular, vertical track, which has a radius of R. The
contact surface between the mass and the loop is frictionless.
A bullet of mass m strikes the block horizontally with initial
speed v0 and remain embedded in the block as the block and
the bullet circle the loop. Determine each of the following in
terms m, v0, R and g.

m, v0

(a) The speed of the masses immediately after the impact.

Corporate Head Office : Motion Education Pvt. Ltd., 394 - Rajeev Gandhi Nagar, Kota-5 (Raj.)
CENTRE OF MASS Page # 81

Q.1 In a game of Carom Board, the Queen (a wooden Q.5 A massive vertical wall is approaching a man at
disc of radius 2 cm and mass 50 gm) is placed at the a speed u. When it is at a distance of 10m, the man
exact center of the horizontal board. The striker is throws a ball with speed 10 m/s an at angle of 37°
a smooth plastic disc of radius 3 cm and mass 100 which after completely elastic rebound reaches back
gm. The board is frictionless. Th striker is given an directly into his bands. Find the velocity u of the
initial velocity ‘u’ parallel to the sides BC or AD so wall.
that is hits the Queen inelastically with same
coefficient of restitution = 2/3. The impact parameter Q.6 Mass m1 hits & sticks with m2 while sliding
for the collision is ‘d’ (shown in the figure). The horizontally with velocity v along the common line
Queen rebounds from the edge AB of the board of centres of the three equal masses (m1 = m2 = m3
inelastically with same coefficient of restitution = 2/ =m). Initially masses m2 and m3 are stationary and
3. and enters the hole D following the dotted path the spring is unstretched. Find
shown. The side of the board is L. Find the value of
impact parameter ‘d’ and the time which the Queen v k
takes to enter hole D after collision with the striker. m1 m2 m3

A L B Frictionless
(a) the velocities of m1, m2 and m3 immediately after
impact.
u (b) the maximum kinetic energy of m3.
L d (c) the minimum kinetic energy of m2.
(d) the maximum compression of the spring.

D C Q.7 Two masses A and B connected with an


inextensible string of length l lie on a smooth
horizontal plane. A is gi ven a vel oci ty of v m/s
Q.2 A flexible chain has a length l and mass m. It is al ong the ground perpendicular to line AB as
lowered on the table top with constant velocity v. shown in figure. Find the tension in string during
Find the force that the chain exerts on the table as their sub sequent motion.
a function of time. B m

Q.3 A 24-kg projectile is fired at an angle of 53°


above the horizontal with an initial speed of 50 m/s. l
At the highest point in its trajectory, the projectile
explodes into two fragments of equal mass, the first
A 2m v
of which falls vertically with zero initial speed.
(a) How far from the point of firing does the second
fragment strike the ground? (Assume the ground is Q.8 The simple pendulum A of mass mA and length l
level.) is suspended from the trolley B of mass mB. If the
(b) How much energy was released during the system is released from rest at  = 0, determine the
explosion? velocity vB of the trolley and tension in the string
when  = 90°. Friction is negligible.
Q.4 A particle is projected from point O on level
ground towards a smooth vertical wall 50m from O
and hits the wall. The initial velocity of the particle
is 30m/s at 45° to the horizontal and the coefficient
of restitution between the particle and the wall is e. B
Find the distance from O of the point at which the
particle hits the ground again if (a) e = 0, (b) e = 1, l A
(c) e = 1/2

: 0744-2209671, 08003899588 | url : www.motioniitjee.com, : info@motioniitjee.com


Page # 82 CENTRE OF MASS

Q.9 A ball with initial speed 10m/s collides elastically Q.11 A cart is moving along +x direction with a
with two other identical ball whose centres are on a velocity of 4m/s. A person in the cart throws a stone
line perpendicular to the initial velocity and which with a velocity of 6m/s relative to himself. In the
are initially in contact with each other. All the three frame of reference of the cart the stone is thrown
ball are lying on a smooth horizontal table. The first in y-z plane making an angle of 30° with the vertical
ball is aimed directly at the contact point of the z-axis. At the highest point of its trajectory, the
other two balls All the balls are smooth. Find the stone hits an object of equal mass hung vertically
velocities of the three balls after the collision.
from branch of a tree by means of a string of length
L. A completely inelastic collosion occurs, in which
the stone gets embedded in the object. Determine
10 m/s (a) the speed of the combined mass immediately
after collision with respect to an observer on the
ground.
Q.10 A mass m1 with initial speed v0 in the positive (b) the length L of the string such that tension in
x-direction collides with a mass m2 = 2m1 which is the string becomes zero when the string becomes
initially at rest at the origin, as shown in figure. horizontal during the subsequent motion of the
After the collision m1 moves off with speed v1 = v0/2 combined mass.
in the negative y-direction, and m2 moves off with
speed v2 at angle . Q.12 Twp equal sphere of mass ‘m’ are suspended
(A) Find the velocity (magnitude and direction) of by vertical strings so that they are in contact with
the center of mass before the collision, as well as their centres at same level. A third equal spheres of
its velocity after the collision. mass m falls vertically and strikes the other two
simultaneously so that their centres at the instant
(B) Write down the x and y-components of the
of impact form an equilateral triangle in a vertical
equation of conservation of momentum for the
plane. If u is the velocity of m just before impact,
collision. find the velocities just after impact and the impulsive
(C) Determine tan, and find v2 in terms of v0. tension of the strings.
(D) Determine how much (if any) energy was gained
or lost in the collision, and state whether the collision
was elastic or inelastic.
u
y v2
y m2
m

x v
v m
v0 m
m1
m2 = 2m1 After
Before

Corporate Head Office : Motion Education Pvt. Ltd., 394 - Rajeev Gandhi Nagar, Kota-5 (Raj.)
CENTRE OF MASS Page # 83

Exercise - IV PREVIOUS YEAR QUESTIONS

LEVEL - I JEE MAIN


1. Two identical particles move towards each other Sol.
with velocity 2v and v respectively. The velocity of
centre of mass is [AIEEE 2002]
v v
(A) v (B) (C) (D) zero
3 2
Sol.

2. Consider the following two statements :


A. Linear momentum of a system of particles is zero. 4. A mass m moves with a velocity v and collides
B. kinetic energy of a system of particles is zero.Then inelastically with another identical mass. After collision
[AIEEE 2003]
(A) A does not imply B and B does not imply A v
(B) A implies B but B does not imply A the 1st mass moves with velocity in a direction
3
(C) A does not imply B but B implies A
(D) A implies B and B implies A perpendicular to the initial direction of motion. Find
Sol. the speed of the second mass after collision.
[AIEEE 2005]

2 v
(A) v (B) 3v (C) v (D)
3 3
Sol.

3. A body A of mass M while falling vertically


downwards under gravity breaks into two parts; a
1 2
body B of mass M and, a body C of mass M.
3 3
The centre of mass of bodies B and C taken together
shifts compared to that of body A towards
[AIEEE 2005]
(A) depends on height of breaking
(B) does not shift
(C) body C
(D) body B

: 0744-2209671, 08003899588 | url : www.motioniitjee.com, : info@motioniitjee.com


Page # 84 CENTRE OF MASS

5. A player caught a cricket ball of mass 150 g 8. A player caught a cricket ball of mass 150 g moving
moving at a rate of 20 m/s. If the catching process at a rate of 20 m/s. If the catching process is
is completed in 0.1 s, the force of the blow exerted completed in 0.1 s, the force of the blow exerted by
by the ball on the hand of the player is equal to the ball on the hand of the player is equal to
[AIEEE 2006] [AIEEE 2006]
(A) 150 N (B) 3 N (C) 30 N (D) 300 N (A) 150 N (B) 3 N (C) 30 N (D) 300 N
Sol. Sol.

6. Consider a two particle system with particles 9. A circular disc of radius R is removed from a bigger
having masses m1 and m2. If the first particle is c i rcul ar di sc of radi us 2 R , such t hat the
pushed towards the centre of mass through a circumference of the discs coincide. The centre of
distance d, by what distance should the second
mass of the new disc is R from the centre of the
particle be moved, so as to keep the centre of mass
at the same position? [AIEEE 2006] bigger disc. The value of  is [AIEEE 2007]

m2 m1 1 1 1 1
d d (A) (B) (C) (D)
(A) (B) 3 2 6 4
m1 m1  m2
Sol.
m1
(C) d (D) d
m2
Sol.

10. A thin rod of length L is lying along the x-axis


with its ends at x = 0 and x = L. Its linear density
n
 x
(mass/length) varies with x as k   , where n
7. A bomb of mass 16 kg at rest explodes into two  L
pieces of masses 4 kg and 12 kg. The velocity of can be zero or any positive number. If the position
the 12 kg mass is 4 ms–1. The kinetic energy of the xCM of the centre of mass of the rod is plotted
other mass is [AIEEE 2006]
against n, which of the following graphs best
(A) 144 J (B) 288 J (C) 192 J (D) 96 J
Sol. approximates the dependence of xCM on n?
[AIEEE 2008]
XCM XCM

L L
L L
(A) 2 (B) 2

n n
O O

Corporate Head Office : Motion Education Pvt. Ltd., 394 - Rajeev Gandhi Nagar, Kota-5 (Raj.)
CENTRE OF MASS Page # 85

XCM XCM Sol.

L L

L L
(C) 2 (D) 2

n n
O O
Sol.

13. Statement-I : A point particle of mass m moving


11. A block of mass 0.50 kg is moving with a speed with speed  collides with stationary point particle
of 2.00 ms–1 on a smooth surface. It strikes another of mass M. If the maximum energy loss possible is
mass of 1.00 kg and then they move together as a
single body. The energy loss during the collision is 1 2  m 
given as f  m  then f =  .
[AIEEE 2008] 2  M  m
(A) 0.16 J (B) 1.00 J Statement-II : Maximum energy loss occurs when
(C) 0.67 J (D) 0.34 J the particles get stuck together as a result of the
Sol. collision. [JEE MAIN 2013]
(A) Statement-I is true, Statement-II is false
(B) Statement-I is false, Statement-II is true.
(C) Statement-I is true, Statement-II is a correct
explanation of Statement-I
(D) Statement-I is true, Statement-II is ture,
Statement-II i s not a correct explanation of
Statement-I.
Sol.

12. Statement I : Two particles moving in the same


direction do not lose all their energy in a completely
inelastic collision.
Statement II : Principle of conservation of
momentum holds true for all kinds of collisions.
[AIEEE 2010]
(A) Statement I is true, Statement II is ture; Statement
II is the correct explanation of Statement I.
(B) Statement I is true, Statement II is true; Statement
II is not correct explanation of Statement I.
(C) Statement I is false, Statement II is true.
(D) Statement I is the true, Statement II is false.

: 0744-2209671, 08003899588 | url : www.motioniitjee.com, : info@motioniitjee.com


Page # 86 CENTRE OF MASS

LEVEL - II JEE ADVANCED


1. Two block of masses 10 kg and 4 kg are con- (A) Statement-1 is True, Statement-2 is True; State-
nected by a spring of negligible mass and placed on ment-2 is a correct explanation for Statement-1
a frictionless horizontal surface. An impulse gives a (B) Statement-1 is True, Statement-2 is True;
velocity of 14 m/s to the heavier block in the direc- Statement-2 i s NOT correct explanati on for
tion of the lighter block. The velocity of the centre Statement-1
of mass is [IIT(Scr.)-2002] (C) Statement-1 is True, Statement-2 is False
(A) 30 m/s (B) 20 m/s (D) Statement-1 is False, Statement-2 is True
(C) 10 m/s (D) 5 m/s Sol.
Sol.

2. A particle of mass m, moving in a circular path of


radius R with a constant speed v2 is located at point
(2R, 0) at time t = 0 and a man starts moving with
velocity v1 along the +ve y-axis from origin at time 
4. The balls, having linear momenta p1  piˆ and
t = 0. Calculate the linear momentum of the particle 
w.r.t. the man as a function of time. [JEE-2003] p2  –pi , undergo a collision in free space. There is
 
y no external force acting on the balls. Let p'1 and p'2
v2
v1 be their final momenta. The following options(s) is
R (are) NOT ALLOWED for any non-zero value of p,
(0,0) a1, a2, b1, b2, c1 and c2. [JEE 2008]
m x  
(A) p'1  a1î  b1 ĵ  c1k̂ ; p' 2  a 2 î  b 2 ĵ
 
(B) p'1  c1k̂ ; p' 2  c 2k̂
Sol.
 
(C) p'1  a1î  b1 ĵ  c1k̂ ; p' 2  a 2 î  b 2 ĵ – c1k
 
(D) p'1  a1î  b1 ĵ ; p' 2  a 2 î  b1 ĵ
Sol.

3. STATEMENT-1
In an elastic collision between two bodies, the rela-
tive speed of the bodies after collision is equal to
the relative speed before the collision.
because
STATEMENT-2
In an elastic collision, the linear momentum of the
system is conserved

Corporate Head Office : Motion Education Pvt. Ltd., 394 - Rajeev Gandhi Nagar, Kota-5 (Raj.)
CENTRE OF MASS Page # 87

Paragraph for Question No. 5 to 7 Sol.


A small block of mass M moves on a frictionless
surface of an inclined plane, as shown in figure. The
angle of the incline suddenly changes from 60° to
30° at point B. The block is initially at rest at A.
Assume that collisions between the block and the
incline are totally inelastic (g = 10 m/s2). Figure :
[JEE 2008]
M
A
v
60° B

8. Look at the drawing given in the figure which has


30° C been drawn with ink of uniform line-thickness. The
mass of ink used to draw each of the two inner
3m 3 3m circles, and each of the two line segments is m. The
mass of the ink unsed to draw the outer circle is
5. The speed of the block at point B immediately 6m. The coordinates of the centres of the different
after it strikes the second incline is parts are outer circle (0, 0), left inner circle (–a, a),
(A) 60 m / s (B) 45 m / s right inner circle (a, a), vertical line (0, 0) and
horizontal line (0, – a). The y-coordinate of the
(C) 30 m / s (D) 15 m / s centre of mass of the ink in this drawing is
Sol. [JEE 2009]

m
m
(-a, a)
(a, a)
(0, 0) 7m

m
(0, -a)
Sol.

6. The speed of the block at point C, immediately


before it leaves the second incline is
(A) 120 m / s (B) 105 m / s

(C) 90 m / s (D) 75 m / s
Sol.

9. Two small particles of equal masses start moving


in opposite directions from a point A in a horizontal
circle orbit. Their tangential velocities are v and 2v,
respectively, as shown in the figure. Between
collisions, the particles move with constant speeds.
After making how many elastic collisions, other than
that at A, these two particulars will again reach the
point A ? [JEE 2009]
V A 2V
7. If collision between the block and the incline is
completely elastic, then the vertical (upward)
component of the velocity of the block at point B,
immediately after it strikes the second incline is
(A) 30 m / s (B) 15 m / s
(A) 4 (B) 3 (C) 2 (D) 1
(C) 0 (D) – 15 m / s

: 0744-2209671, 08003899588 | url : www.motioniitjee.com, : info@motioniitjee.com


Page # 88 CENTRE OF MASS

Sol.

12. A ball of mass 0.2 kg rests on a vertical post of


height 5 m. A bullet of mass 0.01 kg traveling with a
velocity V m/s in a horizontal direction, hits the centre
of the ball. After the collision, the ball and bullet
travel independently. The ball hits the ground at a
distance of 20 m and the bullet at a distance of 100
m from the foot of the post. The initial velocity V of
the bullet is [JEE 2011]
10. Three objects A, B and C are kept in a straight
line on a frictionless horizontal surface. These have v m/s
masses m, 2m and m, respectively. The object A
moves towards B with a speed 9 ms–1 and makess
an elastic collision with it. There after, B makes
completely inelastic collision with C. All motions occur
on the same straight line. Find the final speed
(in ms–1 ) of the object C. [JEE 2009]

2m 0 20 100
m m
A B C
(A) 250 m/s (B) 250 2 m/s
Sol.
(C) 400 m/s (D) 500 m/s
Sol.

11. A point mass of 1 kg collides elastically with a


stationary point mass of 5 kg. After their collision,
the 1 kg mass reverse its direction and moves with
a spee d of 2 m s – 1 . Whi ch of the fol l owi ng
statement(s) is (are ) correct for the system of
these two masses ?
(A) Total momentum of the system is 3 kg ms–1
(B) Momentum of 5 kg mass after collision is 4 kg ms–1
(C) Kinetic energy of the centre of mass is 0.75 J
(D) Total kinetic energy of the system is 4 J
[JEE 2010]
Sol.

Corporate Head Office : Motion Education Pvt. Ltd., 394 - Rajeev Gandhi Nagar, Kota-5 (Raj.)
CENTRE OF MASS Page # 89

Exercise - I OBJECTIVE PROBLEMS (JEE MAIN)


1. B 2. C 3. A 4. C 5. C 6. C 7. C
8. B 9. D 10. D 11. D 12. B 13. A 14. A
15. C 16. A 17. A 18. C 19. (a) B, (b) C 20. (a) C
21. B 22. B 23. C 24. D 25. B 26. A 27. C
28. C 29. C 30. C 31. B 32. B 33. A 34. A
35. A 36. B 37. C 38. A 39. C 40. A 41. C
42. B 43. D 44. D 45. B 46. D 47. D 48. A
49. B 50. D

Exercise - II
1. C,D 2. B,D 3. A,B 4. C 5. B,C 6. B 7. B,D
8. C 9. C 10. B,C 11. B 12. A,B,C,D 13. B,C 14. A,C
15. B,C 16. A,B,D 17. A,B,C 18. D 19. A,B,C,D 20. B,D 21. A,C
22. (a) A,C ;(b) B 23. D 24. C 25. B 26. D 27. A
28. C 29. A 30. D 31. A 32. B 33. D 34. D
35. A 36. A 37. D 38. B 39. B 40. B 41. B
42. D 43. C 44. C 45. D 46. D 47. B

Exercise - III (JEE ADVANCED)

x 5 4  b3  a3  3
1. (a) ( x)    , (b) L 2. y    3. a 4. 100 m
L 9 3  b 2 – a 2  4

2gR
5. (i) zero; (ii) right ; (iii) 20 cm ; (iv) 2.2 m ; (v) 1.8 m 6. g/9 downwards 7. v 
m
1
M
kx 20 Kx20
8. 9 m/s, 9 m 9. (a) , (b) 10. (i) 20Ns, (ii) 50 N
2m 4

P(N-sec)
0.25

0.125
11. 12. 187.5 N, 625 kPa 13. m  u2  uv  v 2

50 100 t(ms)

: 0744-2209671, 08003899588 | url : www.motioniitjee.com, : info@motioniitjee.com


Page # 90 CENTRE OF MASS

2 2
p1  p 2 p1  p 2
14. (a)  12.3 m / sec , (b) = 9.4 m/s 15. (a) 4 5 N , (b) 2000 5 N
mp mp

16. 1.5 m/s, 3.6 Ns 17. (a) u/2, mu/2; (b) u 13 / 8 , m u 13 / 8 (c) u 3 / 4 , mu 3 / 4
mv
18. W = –3mu2; J = 6 mu, K = mu 19. (m  Ax) 20. 6 m/s in the direction of football’s velocity

12 40
21. (a) 3J, (b) Ns 22. 2 : 1 23. (a) m , (b) te = 3.25 s
5 3
gl
24. (a) v  3 gl , (b) v = 25. (a) 10 3 , (b) tan–1 2 , (c) 45º, (d) 20 m
2
m 3
26. (a) e = cot2 + = , (g) t = 3 sec. 27. v A  g / 12 m / s , Smax = 49/48 m
M 4
 M0 
28. (a) v0/3, (b) 3 5gR 29. v = (ucos –  u sin  ) ln  M – t  – gt
 0 

m
1. ( 5 / 17 cm , 153L / 80u 2. v( v  gt) 3. (a) 360 m, (b) 10800 J 4. (a) 50 m, (b) 10 m, (c) 30 m

5. 13/3 m/s 6. (a) v/2, v/2, 0; (b) 2mv2/9, (c) mv2/72, (d) x  m / 6k v

mA 2gl 2m 2A g
7. 2mv2/3l 8. vB  m 1+ m A / mB ; T = 3m A
g + 9. -2m/s, 6.93 m/s 30º
B mB

mv 0 1 5 mv 20
10. (a) v0/3, (b) mv0 = 2mv2 cos, 0 = 2mv2sin– , (c) , v 0 , (d)
2 2 4 16

2 3u 5u 6
11. 2.5 m/sec, 0.312 m 12. v  ,u  , T  mv
7 7 7

Exercise - IV PREVIOUS YEAR QUESTIONS

LEVEL - I JEE MAIN

1. C 2. C 3. B 4. C 5. C 6. C 7. B
8. C 9. A 10. A 11. C 12. A 13. B

LEVEL - II JEE ADVANCED



1. C 2. PPM  mv PM  mv 2 sint î  m( v 2 cos t  v1 ) ĵ 3. B 4. A, D 5. B 6. B

7. C 8. a/10 9. C 10. 4 11. A,C 12. D

Corporate Head Office : Motion Education Pvt. Ltd., 394 - Rajeev Gandhi Nagar, Kota-5 (Raj.)
ROTATIONAL DYNAMICS Page # 91

ROTATIONAL DYNAMICS
1. RIGID BODY :
Rigid body is defined as a system of particles in which distance between each pair of particles
remains constant (with respect to time) that means the shape and size do not change,
during the motion. Eg. Fan, Pen, Table, stone and so on.
Our body is not a rigid body, two blocks with a spring attached between them is also not a
rigid body. For every pair of particles in a rigid body, there is no velocity of seperation or
approach between the particles. In the figure shown velocities of A and B with respect to
 
ground are VA and VB respectively

A
VA sin1
A
A VA cos 1
 1 VA
B B
VBA
VB  2 B
VB sin 2
VB cos 2
If the above body is rigid
VA cos 1 = VB cos 2
Note : With respect to any particle of rigid body the motion of any other particle of that rigid body is
circular.
VBA = relative velocity of B with respect to A.
Types of Motion of rigid body

Pure Translational Pure Rotational Combined Translational and


Motion Motion Rotational Motion

1.1. Pure Translational Motion :


A body is said to be in pure translational motion if the displacement of each particle is same
 
during any time interval however small or large. In this motion all the particles have same s, v

& a at an instant.
example.
A box is being pushed on a horizontal surface.
10
6 6
10
16
   
Vcm  V of any particle, acm  a of any particle
 
Scm  S of any particle
For pure translational motion :-
v
v m2 m2
m1 v m1
v m3 m3
m4 v v m4
m5 vm6 m5 m6
v m8 m8
m7 m7
   
Fext  m1a1  m 2 a2  m3 a 3 .............

: 0744-2209671, 08003899588 | url : www.motioniitjee.com, : info@motioniitjee.com


Page # 92 ROTATIONAL DYNAMICS

Where m1, m2, m3, ......... are the masses of different particles of the body having accelerations
  
a1, a2 , a3 ,............... respectively..
   
But acceleration of all the particles are same So, a1  a 2  a 3  .........  a
 
Fext  Ma
Where M = Total mass of the body

a = acceleration of any particle or of centre of mass of body
   
P  m1v1  m2 v 2  m 3 v 3 .............
Where m1, m2, m3 ...... are the masses of different particles of the body having velocities
  
v1, v 2 , v 3 ............. respectively
   
But velocities of all the particles are same so v1  v 2  v 3 ..........  v
 
P  Mv

Where v = velocity of any particle or of centre of mass of the body..
1 1 1
Total Kinetic Energy of body = m1v 12  m 2 v 22  .......... .  Mv 2
2 2 2

1.2. Pure Rotational Motion :


A body is said to be in pure rotational motion if the perpendicular distance of each particle
remains constant from a fixed line or point and do not move parallel to the line, and that line
  
is known as axis of rotation. In this motion all the particles have same ,  and  at an
instant. Eg. : - a rotating ceiling fan, arms of a clock.
For pure rotation motion :-
s
 Where  = angle rotated by the particle
r
m3

s = length of arc traced by the particle. m2


m2

r = distance of particle from the axis of rotation. m1


m5


m3
m4
d
m1
m6

 Where  = angular speed of the body.. m5 m6


dt
m4

d
 Where  = angular acceleration of the body..
dt
All the parameters ,  and  are same for all the particles. Axis of rotation is perpendicular to
the plane of rotation of particles.
Special case : If  = constant,
 = 0 +  t Where 0 = initial angular speed
1 2
  0t  t t = time interval
2
2 = 02 + 2  
1 1
Total Kinetic Energy  m1v12  m2 v 22 .................
2 2
1
 [m1r12  m 2r22 ................]  2
2
1 2
 I Where I = Moment of Inertia = m1r12  m 2r22 .......
2
 = angular speed of body.

Corporate Head Office : Motion Education Pvt. Ltd., 394 - Rajeev Gandhi Nagar, Kota-5
ROTATIONAL DYNAMICS Page # 93

1.3 Combined translation and rotational Motion


A body is said to be in translation and rotational motion if all the particles rotates about an
axis of rotation and the axis of rotation moves with respect to the ground.

2. MOMENT OF INERTIA
Like the centre of mass, the moment of inertia is a property of an object that is related to its
mass distribution. The moment of inertia (denoted by I) is an important quantity in the study
of system of particles that are rotating. The role of the moment of inertia in the study of
rotational motion is analogous to that of mass in the study of linear motion. Moment of inertia
gives a measurement of the resistance of a body to a change in its rotaional motion. If a body
is at rest, the larger the moment of inertia of a body the more difficuilt it is to put that body
into rotational motion. Similarly, the larger the moment of inertia of a body, the more difficult
to stop its rotational motion. The moment of inertia is calculated about some axis (usually the
rotational axis).
Moment of inertia depends on :
(i) density of the material of body
(ii) shape & size of body
(iii) axis of rotation
In totality we can say that it depends upon distribution of mass relative to axis of rotation.
Note :
Moment of inertia does not change if the mass :
(i) is shifted parallel to the axis of the rotation
(ii) is rotated with constant radius about axis of rotation
2.1 Moment of Inertia of a Single Particle
r
For a very simple case the moment of inertia of a
single particle about an axis is given by,
I = mr2 ...(i)
Here, m is the mass of the particle and r its distance from the axis under consideration.
2.2 Moment of Inertia of a System of Particles
The moment of inertia of a system of particles about an axis is given by,
2
I= m r
i
ii ...(ii)
r1
m1
r2
m2
r3
m3

where ri is the perpendicular distance from the axis to the ith particle, which has a mass mi.
Ex.1 Two heavy particles having masses m1 & m2 are situated in a plane perpendicular to
line AB at a distance of r1 and r2 respectively.
C A

r1 r2
E F
m1 m2

D B
(i) What is the moment of inertia of the system about axis AB?
(ii) What is the moment of inertia of the system about an axis passing through m1
and perpendicular to the line joining m1 and m2 ?
(iii) What is the moment of inertia of the system about an axis passing through m1
and m2?

: 0744-2209671, 08003899588 | url : www.motioniitjee.com, : info@motioniitjee.com


Page # 94 ROTATIONAL DYNAMICS

Sol. (i) Moment of inertia of particle on left is I1 = m1r12.


Moment of Inertia of particle on right is I2 = m2r22.
Moment of Inertia of the system about AB is
I = I1+ I2 = m1r12 + m2r22
(ii) Moment of inertia of particle on left is I1 = 0
Moment of Inertia of the system about CD is
I = I1 + I2 = 0 + m2(r1 + r2)2
(iii) Moment of inertia of particle on left is I1 = 0
Moment of inertia of particle on right is I2 = 0
Moment of Inertia of the system about EF is
I = I1 + I2 = 0 + 0

Ex.2 Three light rods, each of length 2, are joined together to form a triangle. Three particles
A, B, C of masses m, 2m, 3m are fixed to the vertices of the triangle. Find the moment
of inertia of the resulting body about
(a) an axis through A perpendicular to the plane ABC,
(b) an axis passing through A and the midpoint of BC. X
A
Sol. (a) B is at a distant 2 from the axis XY so the moment of m
inertia of B (IB) about XY is 2 m (2)2
Y
Similarly Ic about XY is 3m (2)2 and IA about XY is m(0)2
2l
2l
Therefore the moment of inertia of the body about XY is
2m (2)2 + 3 m(2)2 + m(0)2 = 20 m2
(b) IA about X' Y' = m(0)2 B
C
IB about X' Y' = 2m ()2 2m 3m
IC about X' Y' = 3m ()2
Therefore the moment of inertia of the body about X' Y' is
m(0)2 + 2m()2 + 3m()2 = 5 m2
X'
A m

B C
2m 3m
Y'
Ex.3 Four particles each of mass m are kept at the four corners of a square of edge a. Find
the moment of inertia of the system about a line perpendicular to the plane of the
square and passing through the centre of the square.
Sol. The perpendicular distance of every particle from
the given line is a / 2 . The moment of inertia of m
m
2 1 2
one particle is, therefore, m(a / 2 ) = ma . The
2
moment of inertia of the system is,
2
/
a

1
therefore, 4  ma2 = 2 ma2. m m
2

Corporate Head Office : Motion Education Pvt. Ltd., 394 - Rajeev Gandhi Nagar, Kota-5
ROTATIONAL DYNAMICS Page # 95

2.3 Moment of Inertia of Rigid Bodies


For a continuous mass distribution such as found
in a rigid body, we replace the summation of
2
I m r
i
ii by an integral. If the system is divided

r
into infinitesimal element of mass dm and if r is
the distance from a mass element to the axis of
rotation, the moment of inertia is,
2
I= r dm

where the integral is taken over the system.

(A) Uniform rod about a perpendicular bisector


Consider a uniform rod of mass M and length l figure and suppose the moment of inertia is to
be calculated about the bisector AB. Take the origin at the middle point O of the rod. Consider
the element of the rod between a distance x and x + dx from the origin. As the rod is uniform,
Mass per unit length of the rod = M / l A
so that the mass of the element = (M/l)dx. x dx
The perpendicular distance of the element from 0
the line AB is x. The moment of inertia of this B
element about AB is
M
dI  dx x 2 .
l
When x = – l/2, the element is at the left end of the rod. As x is changed from – l/2 to l/2, the
elements cover the whole rod.
Thus, the moment of inertia of the entire rod about AB is

l /2 l /2
M 2  M x3  Ml 2
I

l / 2
l
x dx   
 l 3  –l / 2

12

(B) Moment of inertia of a rectangular plate about a line parallel to an edge and passing
through the centre
The situation is shown in figure. Draw a line parallel to AB at a distance x from it and another
at a distance x + dx. We can take the strip enclosed between the two lines as the small
element.
A x

B dx
l
It is “small” because the perpendiculars from different points of the strip to AB differ by not
more than dx. As the plate is uniform,
M
its mass per unit area =
bl
M M
Mass of the strip = b dx  dx .
bl l

: 0744-2209671, 08003899588 | url : www.motioniitjee.com, : info@motioniitjee.com


Page # 96 ROTATIONAL DYNAMICS

The perpendicular distance of the strip from AB = x.


M
The moment of inertia of the strip about AB = dI = dx x 2 . The moment of inertia of the given
l
plate is, therefore,
l/2
M 2 Ml 2
I 
l / 2
l
x dx 
12
The moment of inertia of the plate about the line parallel to the other edge and passing
through the centre may be obtained from the above formula by replacing l by b and thus,
Mb 2
.
I
12
(C) Moment of inertia of a circular ring about its axis (the line perpendicular to the plane of
the ring through its centre)
Suppose the radius of the ring is R and its mass is M. As all the elements of the ring are at the
same perpendicular distance R from the axis, the moment of inertia of the ring is

I  r 2 dm  R2 dm  R2 dm  MR2 .
  
(D) Moment of inertia of a uniform circular plate about its axis
Let the mass of the plate be M and its radius R. The centre is at O and the axis OX is
perpendicular to the plane of the plate.
X

dx
0
x

R
Draw two concentric circles of radii x and x + dx, both centred at O and consider the area of
the plate in between the two circles.
This part of the plate may be considered to be a circular ring of radius x. As the periphery of
the ring is 2 x and its width is dx, the area of this elementary ring is 2xdx. The area of the
plate is  R2. As the plate is uniform,
M
Its mass per unit area =
 R2

M 2M x dx
Mass of the ring  2 xdx 
 R2 R2
Using the result obtained above for a circular ring, the moment of inertia of the elementary
ring about OX is
 2Mx dx  2
dI   2 x .
 R 
The moment of inertia of the plate about OX is
R
2M MR 2
I R 2
x 3 dx  .
2
0

(E) Moment of inertia of a hollow cylinder about its axis


Suppose the radius of the cylinder is R and its mass is M. As every element of this cylinder is
at the same perpendicular distance R from the axis, the moment of inertia of the hollow
cylinder about its axis is

I  r 2 dm  R 2 dm  MR 2
 

Corporate Head Office : Motion Education Pvt. Ltd., 394 - Rajeev Gandhi Nagar, Kota-5
ROTATIONAL DYNAMICS Page # 97

(F) Moment of inertia of a uniform solid cylinder about its axis


Let the mass of the cylinder be M and its radius R. Draw two cylindrical surface of radii x and
x + dx coaxial with the given cylinder. Consider the part of the cylinder in between the two
surface. This part of the cylinder may be considered to be a hollow cylinder of radius x. The
area of cross-section of the wall of this hollow cylinder is 2 x dx. If the length of the cylinder
is l, the volume of the material of this elementary hollow cylinder is 2 x dxl.
The volume of the solid cylinder is  R2 l and it is uniform, hence its mass per unit volume is

M

 R2 l
The mass of the hollow cylinder considered is

M 2M
2 x dx l  x dx .
 R2 l R2
dx
As its radius is x, its moment of inertia about the given axis is
x
 2M 
dI   2 xdx x 2 .
R 
The moment of inertia of the solid cylinder is, therefore,
R
2M MR 2
I R x 3 dx 
2
2 .
0

Note that the formula does not depend on the length of the cylinder.
(G) Moment of inertia of a uniform hollow sphere about a diameter
Let M and R be the mass and the radius of the sphere, O its centre and OX the given axis
(figure). The mass is spread over the surface of the sphere and the inside is hollow.
Let us consider a radius OA of the sphere at an angle  with the axis OX and rotate this radius
about OX. The point A traces a circle on the sphere. Now change  to  + d and get another
circle of somewhat larger radius on the sphere. The part of the sphere between these two
circles, shown in the figure, forms a ring of radius R sin. The width of this ring is Rd and its
periphery is 2R sin. Hence,
the area of the ring = (2R sin) (Rd).
x
M R sin
Mass per unit area of the sphere  . A
4 R 2 Rd

M M R
The mass of the ring  (2R sin )(Rd)  sin  d. d
4 R 2 2 0

The moment of inertia of this elemental ring about OX is

M 
d I   sin  d. (R sin ) 2  M R 2 sin3  d
2  2
As  increases from 0 to , the elemental rings cover the whole spherical surface. The
moment of inertia of the hollow sphere is, therefore,
   
M 2 MR2   MR 2 
I  R sin3  d   (1  cos 2 ) sin  d  
  (1  cos 2 ) d(cos )

2 2   2  
0 0    0 

MR 2  cos 3   2 2
 cos     MR
2  3  3
0

: 0744-2209671, 08003899588 | url : www.motioniitjee.com, : info@motioniitjee.com


Page # 98 ROTATIONAL DYNAMICS

(H) Moment of inertia of a uniform solid sphere about a diameter


Let M and R be the mass and radius of the given solid sphere. x
Let O be centre and OX the given axis. Draw two spheres of
radii x and x + dx concentric with the given solid sphere. The dx
thin spherical shell trapped between these spheres may be treated
0 x
as a hollow sphere of radius x.
The mass per unit volume of the solid sphere

M 3M
= 
4
R 3 4 R 3
3
The thin hollow sphere considered above has a surface area 4x2 and thickness dx. Its volume
i s
4  x2 dx and hence its mass is

 3M  3M
=   (4  x2 dx) = 3 x 2 dx
 4  R3  R

Its moment of inertia about the diameter OX is, therefore,


2  3M 2  2M 4
dl = 3  3 x dx x2 =x dx
R  R3
If x = 0, the shell is formed at the centre of the solid sphere. As x increases from 0 to R, the
shells cover the whole solid sphere.
The moment of inertia of the solid sphere about OX is, therefore,
R
2M 2
I= x 4 dx = MR2 .
R
0
3 5

Ex.4 Find the moment of Inertia of a cuboid along the axis as shown in the figure.

I
b

a
c

M(a 2  b 2 )
Sol. After compressing the cuboid parallel to the axis I =
12

3. THEOREMS OF MOMENT OF INERTIA


There are two important theorems on moment of inertia, which, in some cases enable the
moment of inertia of a body to be determined about an axis, if its moment of inertia about
some other axis is known. Let us now discuss both of them.

Corporate Head Office : Motion Education Pvt. Ltd., 394 - Rajeev Gandhi Nagar, Kota-5
ROTATIONAL DYNAMICS Page # 99

3.1 Theorem of parallel axes


A very useful theorem, called the parallel axes theorem relates
the moment of inertia of a rigid body about two parallel axes,
one of which passes
through the centre of mass. COM

Two such axes are shown in figure for a body of mass M. If r is


the distance between the axes and ICOM and I are the respective r
moments of inertia about them, these moments are related by,
I = ICOM + Mr2
* Theorem of parallel axis is applicable for any type of rigid body whether it is a two dimensional
or three dimensional

Ex 5. Three rods each of mass m and length l are joined A


together to form an equilateral triangle as shown in
figure. Find the moment of inertia of the system
about an axis passing through its centre of mass and
perpendicular to the plane
COM
of triangle.
Sol. Moment of inertia of rod BC about an axis perpendicular
B C
to plane of triangle ABC and passing through the mid-
point of rod BC (i.e., D) is
ml 2
I1 =
12
From theorem of parallel axes, moment of inertia of this A
rod about the asked axis is
2
ml 2  l  ml 2 COM
I2 = I1 + mr2 =  m  
12 2 3 6
r
 Moment of inertia of all the three rod is 30°
B D C
 ml 2  ml 2
I  3I2  3  
 6  2

Ex.6. Find the moment of inertia of a solid sphere of mass M and radius R about an axis XX
shown in figure.

x
x
Sol. From theorem of parallel axis,
IXX = ICOM + Mr2
2 COM
= MR 2  MR2
5
7 x
= MR 2
5
r=R

: 0744-2209671, 08003899588 | url : www.motioniitjee.com, : info@motioniitjee.com


Page # 100 ROTATIONAL DYNAMICS

Ex.7. Consider a uniform rod of mass m and length 2l with two particles of mass m each at
its ends. Let AB be a line perpendicular to the length of the rod passing through its
centre. Find the moment of inertia of the system about AB.
Sol. IAB = Irod + Iboth particles A

m(2l )2
  2(ml 2 ) I I
12
m m
7
 ml 2 Ans.
3 B

3.2 Theorem of perpendicular axes


The theorem states that the moment of inertia of a plane lamina about an axis perpendicular
to the plane of the lamina is equal to the sum of the moments of inertia of the lamina about
two axes perpendicular to each other, in its own plane and intersecting each other, at the
point where the perpendicular axis passes through it.
Let x and y axes be chosen in the plane of the body and z-axis perpendicular, to this plane,
three axes being mutually perpendicular, then the theorem states that.
z
y

xi
P
ri yi
O x

Iz = Ix + Iy
Important point in perpendicular axis theorem
(i) This theorem is applicable only for the plane bodies (two dimensional).
(ii) In theorem of perpendicular axes, all the three axes (x, y and z) intersect each other and this
point may be any point on the plane of the body (it may even lie outside the body).
(iii) Intersection point may or may not be the centre of mass of the body.

Ex.8 Find the moment of inertia of uniform ring of mass M and radius R about a diameter.
B
Z

C 0 D

A
Sol. Let AB and CD be two mutually perpendicular diameters of the ring. Take them ax X and Y-
axes and the line perpendicular to the plane of the ring through the centre as the Z-axis. The
moment of inertia of the ring about the Z-axis is I = MR2. As the ring is uniform, all of its
diameter equivalent and so Ix = Iy, From perpendicular axes theorem,

Iz MR2
Iz = Ix + Iy Hence Ix = =
2 2
Similarly, the moment of inertia of a uniform disc about a diameter is MR2/4

Corporate Head Office : Motion Education Pvt. Ltd., 394 - Rajeev Gandhi Nagar, Kota-5
ROTATIONAL DYNAMICS Page # 101

Ex.9 Two uniform identical rods each of mass M and length  are joined to form a cross as
shown in figure. Find the moment of inertia of the cross about a bisector as shown
dotted in the figure.

Sol. Consider the line perpendicular to the plane of the figure through the centre of the cross. The
M 2
moment of inertia of each rod about this line is and hence the moment of inertia of the
12

M 2
cross is . The moment of inertia of the cross about the two bisector are equal by
6
symmetry and according to the theorem of perpendicular axes, the moment of inertia of the
M 2
cross about the bisector is .
12

Ex.10 In the figure shown find moment of inertia of a plate having mass M, length  and
width b about axis 1,2,3 and 4. Assume that C is centre and mass is uniformly distributed

4 2
1
C
3 b


Sol. Moment of inertia of the plate about axis 1 (by taking rods perpendicular to axis 1)
l1 = Mb2/3
Moment of inertia of the plate about axis 2 (by taking rods perpendicular to axis 2)
I 2 = M2/12
Moment of inertia of the plate about axis 3 (by taking rods perpendicular to axis 3)

Mb2
I3 
12
Moment of inertia of the plate about axis 4(by taking rods perpendicular to axis 4)
I4 = M2/3
3.3 Moment of Inertia of Compound Bodies
Consider two bodies A and B, rigidly joined together. The moment of inertia of this compound
body, about an axis XY, is required. If IA is the moment of inertia of body A about XY. IB is the
moment of inertia of body B about XY.Then, moment of Inertia of compound body I = IA + IB
Extending this argument to cover any number of bodies rigidly joined together, we see that
the moment of inertia of the compound body, about a specified axis, is the sum of the
moments of inertia of the separate parts of the body about the same axis.

A
X Y

: 0744-2209671, 08003899588 | url : www.motioniitjee.com, : info@motioniitjee.com


Page # 102 ROTATIONAL DYNAMICS

Ex.11 Two rods each having length l and mass m joined together at point B as shown in
figure.Then findout moment of inertia about axis passing thorugh A and perpendicular
to the plane of page as shown in figure.
 A
B ×

C
Sol. We find the resultant moment of inertia I by dividing in two parts such as
I = M.I of rod AB about A +
M.I of rod BC about A
I = I1 + I2 ... (1)
first calculate I1 :
B  A
×
m 2
I1 = ...(2)
3
Calculation of I2 :  ×
use parallel axis theorem /2
d
I2 = ICM + md2 COM ×
m 2  2 2 m  2 5 2
= 12  m 4    =  m ...(3)
  12 4
Put value from eq. (2) & (3) into (1)
m  2 m  2 5 2 m
I=  
3 12 4
m 2 5m 2
I= ( 4  1  15)  I=
12 3

4. CAVITY PROBLEMS :

Ex.12 A uniform disc having radius 2R and mass density  as shown in figure. If a small disc
of radius R is cut from the disc as shown. Then find out the moment of inertia of
remaining disc around the axis that passes through O and is perpendicular to the plane
of the page.

2R O R

Sol. We assume that in remaning part a disc of radius R and mass density ±  is placed. Then
M1  (2R) 2
M2  –R2

2R O   R 2R I2 R
× I1 ×
 +

when – is takes
when  is taken
Total Moment of Inertia I = I1 + I2
M1( 2R) 2
I1 =
2

Corporate Head Office : Motion Education Pvt. Ltd., 394 - Rajeev Gandhi Nagar, Kota-5
ROTATIONAL DYNAMICS Page # 103

4R2 .4R2
I1 = = 8  R4
2
To calculate I2 we use parallel axis theorem.
I2 = ICM + M2R2

M2R2
I2 = + M2R2
2
3 3 3
I2 = M2R 2 = (– R2 )R 2 I2 = – R 4
2 2 2
Now I = I1 + I2

4 3 13
I = 8R – R 4 I= R 4
2 2
Ex.13 A uniform disc of radius R has a round disc of radius R/3 cut as shown in Fig. The mass
of the remaining (shaded) portion of the disc equals M. Find the moment of inertia of
such a disc relative to the axis passing through geometrical centre of original disc and
perpendicular to the plane of the disc.

O
3
R/

Sol. Let the mass per unit area of the material of disc
be . Now the empty space can be considered as
having density –  and .
Now I0 = I + I– 
(R2)R2/2 = M.I of  about O

– (R / 3) 2 (R / 3)2
I– =  [– (R / 3) 2 ]( 2R / 3) 2
2
= M.I of –  about 0
4
 I0 = R4 Ans.
9
5. TORQUE :
Torque represents the capability of a force to produce change in the rotational motion of the
body

Line of action
of force
P F
r
r sin
Q

5.1 Torque about point :



Torque of force F about a point   
  r F

where F = force applied
P = point of application of force
Q = point about which we want to calculate the torque.

: 0744-2209671, 08003899588 | url : www.motioniitjee.com, : info@motioniitjee.com


Page # 104 ROTATIONAL DYNAMICS


r = position vector of the point of application of force from the point about which we
want to determine the torque.

  rF sin  = rF = rF
where  = angle between the direction of force and the position vector of P wrt. Q.
r = perpendicular distance of line of action of force from point Q.
F = force arm
SI unit to torque is Nm
Torque is a vector quantity and its direction is determined using right hand thumb rule.

Ex.14 A particle of mass M is released in vertical plane from a point P at x = x0 on the x-axis
it falls vertically along the y-axis. Find the torque  acting on the particle at a time t
about origin?
O x0 P
x

 r

Sol.

mg
Torque is produced by the force of gravity

  rF sin  k

or   r F  x0 mg

Ex.15 Calculate the total torque acting on the body shown in figure about the point O
10N
15N
37°
m

90°
6c

O
3c
4cm

30° m
150°
5N
20N

15sin37°
10N 15N
37°
m

90°
6c

O
Sol. 4cm 5N
4cm

30° 20N
150°
20sin30°

0 = 15sin37 × 6 + 20 sin 30° × 4 – 10 × 4


= 54 + 40 – 40 = 54 N-cm
0 = 0.54 N-m

Corporate Head Office : Motion Education Pvt. Ltd., 394 - Rajeev Gandhi Nagar, Kota-5
ROTATIONAL DYNAMICS Page # 105

Ex.16 A particle having mass m is projected with a velocity


v0 from a point P on a horizontal ground making an
angle  with horizontal. Find out the torque about V0
the point of projection acting on the particle when

(a) it is at its maximum height ?
P Q
(b) It is just about to hit the ground back ?
Sol.
(a) Particle is at maximum height then  about point P is  p  rF v0

R r
mg
F = mg ; r 
2 
R v 2 sin 2 P
 P = mg = mg  0
2 2g
mv 20 sin 2
p =
2
(b) when particle is at point Q then  about point P is  p'  rF
r  R ; F = mg Q
P
v 02 sin2 mg
 p'  mgR = mg
g

Ex.17 In the previous question, during the motion of particle from P to Q. Torque of
gravitational force about P is :
(A) increasing (B) decreasing
(C) remains constant (D) first increasing then decreasing
Sol. Torque of gravitational force about P is increasing because r is increasing from O to R.
(Range)

5.2 Torque about axis :


  
  r F

where  = torque acting on the body about the axis of rotation

r = position vector of the point of application of force about the axis of rotation.

F = force applied on the body..
   
 net   1   2   3 .....
To understand the concept of torque about axis we
take a general example which comes out in daily life.
Figure shows a door ABCD. Which can rotate about A D
axis AB. Now if we apply force. F at point. r × y
in inward direction then AB = r F and direction of this
AB is along y axis from right hand thumb rule. Which x
is parallel to AB so gives the resultant torque.
Now we apply force at point C in the direction as shown B C
 
figure. At this time r & F are perpendicular to each other
F
which gives
 AB  rF

: 0744-2209671, 08003899588 | url : www.motioniitjee.com, : info@motioniitjee.com


Page # 106 ROTATIONAL DYNAMICS

But door can’t move when force is applied in this direction because the direction of  AB is
perpendicular to AB according to right hand thumb rule.
So there is no component of  along AB which gives  res  0

Now conclude Torque about axis is the component of r  F parallel to axis of rotation.
Note : The direction of torque is calculated using right hand thumb rule and it is always
perpendicular to the plane of rotation of the body.

F2
r2

F3 r3 × r1 F1

If F1 or F2 is applied to body, body revolves in anti-clockwise direction and F3 makes body


revolve in clockwise direction. If all three are applied.

 resul tan t  F1r1  F2r2 – F3r3 (in anti-clockwise direction)

6. BODY IS IN EQUILIBRIUM : -
We can say rigid body is in equillibrium when it is in
(a) Translational equilibrium

i.e. Fnet  0
Fnet x = 0 and Fnet y = 0 and
(b) Rotational equillibrium

 net  0
i.e., torque about any point is zero

Note :
(i) If net force on the body is zero then net torque of the forces may or may not be zero.
example.
A pair of forces each of same magnitude and acting in opposite direction on the rod.
F
A B C

2
F
 A  2F
(2) If net force on the body is zero then torque of the forces about each and every point is same
 about B B  F + F
 B  2F
 about C  C  2F

Corporate Head Office : Motion Education Pvt. Ltd., 394 - Rajeev Gandhi Nagar, Kota-5
ROTATIONAL DYNAMICS Page # 107

Ex.18 Determine the point of application of third force for which body is in equillibrium when
forces of 20 N & 30 N are acting on the rod as shown in figure
20N

A 10cm C 20cm B
30N
Sol. Let the magnitude of third force is F, is applied in upward direction then the body is in the
equilibrium when

(i) Fnet  0 (Translational Equillibrium)
 20 + F = 30  F = 10 N
So the body is in translational equilibrium when 10 N force act on it in upward direction.
(ii) Let us assume that this 10 N force act. 10N
Then keep the body in rotational equilibrium 20N
x
So Torque about C = 0
i.e. c = 0 A C 20cm B
 30 × 20 = 10 x
30N
x = 60 cm
so 10 N force is applied at 70 cm from point A to keep the body in equilibrium.

Ex.19 Determine the point of application of force, when forces are acting on the rod as shown in
figure.
10N
5N
5cm 5cm

3N

Sol. Since the body is in equillibrium so we conclude F net  0 and torque about any point is zero

i.e.,  net  0

10N
5N 6
F2 x 37°
A  8N
F
F1 3N
Let us assume that we apply F force downward at A angle  from the horizontal, at x distance
from B

From F net  0
 Fnet x = 0 which gives
F2 = 8 N
From Fnet y = 0  5 + 6 = F1 + 3
 F1 = 8 N
If body is in equillibrium then torque about point B is zero.
 3 × 5 + F1. x – 5 × 10 = 0
 15 + 8x – 50 = 0
35
x=  x = 4.375 cm
9

: 0744-2209671, 08003899588 | url : www.motioniitjee.com, : info@motioniitjee.com


Page # 108 ROTATIONAL DYNAMICS

Ex.20 A uniform rod length , mass m is hung from two strings of equal length from a ceiling
as shown in figure. Determine the tensions in the strings ?

/4

A B
Sol. Let us assume that tension in left and right string is TA and TB respectively. Then
 
Rod is in equilibrium then Fnet  0 & net  0

From Fnet  0
mg = TA + TB ...(1)
From net = 0 about A TA TB
/2 /4
 3
mg  TB  0
2 4 A B
2mg mg
 TB =
3
mg
from eq. (1) TA  2mg = mg  TA =
3 3

Ladder Problems :
Ex.21 A stationary uniform rod of mass ‘m’, length ‘’ leans against a smooth vertical wall
making an angle  with rough horizontal floor. Find the normal force & frictional force
that is exerted by the floor on the rod?
smooth


rough
Sol. As the rod is stationary so the linear acceleration and angular acceleration of rod is zero.
i.e., acm = 0 ;  = 0.
A
N2 = f N2
 acm=0
N = mg
1

Torque about any point of the rod should also be zero N1


mgcos 

=0 
mg 
 B
A = 0  mg cos  + f  sin  = N1 cos . 
2 f
Free Body Diagram
mgcos 
N1 cos  = sin  f +
2

mgcos  mgcot 
f= =
2 sin  2

Corporate Head Office : Motion Education Pvt. Ltd., 394 - Rajeev Gandhi Nagar, Kota-5
ROTATIONAL DYNAMICS Page # 109

Ex.22 The ladder shown in figure has negligible mass and rests on a frictionless floor. The
crossbar connects the two legs of the ladder at the middle. The angle between the two
legs is 60°. The fat person sitting on the ladder has a mass of 80 kg. Find the contanct
force exerted by the floor on each leg and the tension in the crossbar.

W
1m
60°

N T N
1m °
30

Sol. The forces acting on different parts are shown in figure. Consider the vertical equilibrium of
“the ladder plus the person” system. The forces acting on this system are its weight (80 kg)
g and the contact force N + N = 2 N due to the floor. Thus
2 N = (80 kg) g or N = (40 kg) (9.8 m/s2) = 392 N
Next consider the equilibrium of the left leg of the ladder. Taking torques of the forces acting on
it about the upper end,

2 2
N (2m) tan 30° = T (1m) or T=N = (392 N) × = 450 N
3 3

Ex.23 A thin plank of mass m and length  is pivoted at one end and it is held stationary in
horizontal position by means of a light thread as shown in the figure then find out the
force on the pivot.

Sol. Free body diagram of the plank is shown in figure. N2 T


 Plank is in equilibrium condition
So Fnet & net on the plank is zero N1 O A
(i) from Fnet = 0 mg
 Fnet x = 0
N1 = 0

Now Fnet  0
y

 N2 + T = mg ...(i)
from net = 0
 net about point A is zero
so N2 .  = mg . /2

mg
 N2 
2

: 0744-2209671, 08003899588 | url : www.motioniitjee.com, : info@motioniitjee.com


Page # 110 ROTATIONAL DYNAMICS

Ex.24 A square plate is hinged as shown in figure and it is held stationary by means of a light
thread as shown in figure. Then find out force exerted by the hinge.

square plate

T
Sol. F.B.D.
 Body is in equilibrium and
N
T and mg force passing through one line so
from net = 0, N=0

mg

7. RELATION BETWEEN TORQUE AND ANGULAR ACCELERATION


The angular acceleration of a rigid body is directly proportional to the sum of the torque
components along the axis of rotation. The proportionality constant is the inverse of the
moment of inertia about that axis, or


I
Thus, for a rigid body we have the rotational analog of Newton's second law ;
   I ...(iii)
Following two points are important regarding the above equation.
(i) The above equation is valid only for rigid bodies. If the body is not rigid like a rotating tank
of water, the angular acceleration  is different for different particles.
(ii) The sum  in the above equation includes only the torques of the external forces,
because all the internal torques add to zero.
Ex.25 A uniform rod of mass m and length  can rotate in vertical plane about a smooth
horizontal axis hinged at point H.

×
H A
(i) Find angular acceleration  of the rod just after it is released from initial horizontal position
from rest?
(ii) Calculate the acceleration (tangential and radial) of point A at this moment.
Sol. (i) H = IH 

 m 2 3g
mg. =  =
2 3 2

3g 3g
(ii) aA = a = . =
2 2
aCA = 2r = 0. = 0 (  = 0 just after release)

Corporate Head Office : Motion Education Pvt. Ltd., 394 - Rajeev Gandhi Nagar, Kota-5
ROTATIONAL DYNAMICS Page # 111

Ex.26 A uniform rod of mass m and length  hinged at point H can rotate in vertical plane
about a smooth horizontal axis. Find force exerted by the hinge just after the rod is
released from rest, from an initial position making an angle of 37° with horizontal ?

37°
×
H
Sol. Just After releasing at 37º from horizontal F.B.D. of plank 

from net = I
N1
R 37º
 m2
 about point A = A = mg cos 37° = . mg mgcos 37º
2 3
A N2
6g
= rad/sec2
5
Now Tangential acceleration of centre of mass
 3g
at = . = m / s2
2 5
just after release vcm = 0  ar = 0
Now resolving of at in horizontal and vertical direction as shwon in figure

9g
a t || 
25 N1
N1 37°
R 3g/5 mg
12g N2
N2 at 
25
from Fnet = ma in both horizontal and vertical direction
 9g  13mg
N2= m 25   N1 =
25

Now R= N12  N22

mg 10
R=
5
PULLEY BLOCK SYSTEM
If there is friction between pulley and string and pulley have some mass then tension is
different on two sides of the pulley.

Reason : To understand this concept we take a pulley block system as shown in figure.


B R
C

T1
A D a M>m
a
M m

: 0744-2209671, 08003899588 | url : www.motioniitjee.com, : info@motioniitjee.com


Page # 112 ROTATIONAL DYNAMICS

Let us assume that tension induced in part AB of the string is T1 and block M move downward.
If friction is present between pulley and string then it opposes the relative slipping between
pulley and string, take two point e and f on pulley and string respectively. If friction is there
then due to this, both points wants to move together. So friction force act on e and d in the
direction as shown is figure
This friction force f acting on point d increases the tension T2
T1 by a small amount dT.
f
Then T1 = T2 + dT
or we can say T = T – f d e C
2 1

f
In this way the tension on two side of pulley is different
If there is no relative slipping between pulley and string T1
a a
then   t =
R R

Ex.27 The pulley shown in figure has moment of inertia l about its axis and radius R. Find the
acceleration of the two blocks. Assume that the string is light and does not slip on the
pulley.
Sol. Suppose the tension in the left string is T1 and that in the right string is T2. Suppose the block
of mass M goes down with an acceleration a and the other block moves up with the same
acceleration. This is also the tangential acceleration of the rim of the wheel as the string
does not slip over the rim.
a
The angular acceleration of the wheel  = .
R
The equations of motion for the mass M,
the mass m and the pulley are as follows ; R
Mg – T1 = Ma ...(i)
T2 – mg = ma ...(ii)
m
Ia
T1R – T2R = I = ...(iii) M
R
Substituting for T1 and T2 from equations (i) and (ii) in equation (iii)
Ia
[M(g – a) – m (g + a)]R =
R
Solving, we get
(M – m)gR2
a=
I  (M  m)R2

8. ANGULAR MOMENTUM
8.1 Angular momentum of a particle about a point.
  
L  r P  L = r p sin 
  Pcos
|L|  r  P  |L|  P  r
 
 P
Where P = momentum of partilcle
 
r = position of vector of particle with respect to point about which r Psin
angular momentum is to be calculated.
 
 = angle between vectors r & p
O
r = perpendicular distance of line of motion of particle from point O.
P = perpendicular component of momentum.
SI unit of angular momentum is kgm2/sec.

Corporate Head Office : Motion Education Pvt. Ltd., 394 - Rajeev Gandhi Nagar, Kota-5
ROTATIONAL DYNAMICS Page # 113

Ex.28 A particle of mass m is moving along the line y = b, z = 0 with constant speed v. State
whether the angular momentum of particle about origin is increasing, decreasing or
constant.
 y P 
Sol. | L |  mvr sin  v
= mvr 
r r  b
mvb
 
 | L | = constant as m, v and b all are constants. O
X
 
Direction of r  v also remains the same. Therefore, angular momentum of particle about
origin remains constant with due course of time.

Note : In this problem | r | is increasing,  is decreasing but r sin , i.e., b remains constant. Hence,
the angular momentum remains constant.

Ex.29 A particle of mass m is projected with velocity v at an angle  with the horizontal. Find
its angular momentum about the point of projection when it is at the highest point of
its trajectory.
Sol. At the highest point it has only horizontal velocity
vx = v cos . Length of the perpendicular to the y
horizontal velocity from 'O' is the maximum height,
where

v 2 sin2 
Hmax 
2g H

mv 3 sin 2  cos  O x
 Angular momentum L =
2g

8.2 Angular Momentum of a rigid body rotating about a fixed axis


Suppose a particle P of mass m is going in a circle of radius r and at some instant the speed
of the particle is v. For finding the angular momentum of the particle about the axis of
rotation, the origin may be chosen anywhere on the axis. We choose it at the centre of the
 
circle. In this case  
r and P are perpendicular to each other and r  P is along the axis.
 
Thus, component of r  P along the axis is mvr itself. The angular momentum of the whole
rigid body about AB is the sum of components of all particles, i.e.,

L= m r v
i
i i i

Here, vi = ri 
2 2
 L= m ri
ii  i or L =  m r
i
i i

or L = I
Here, I is the moment of inertia of the rigid body about AB.


Note : Angular momentum about axis is the component of I along the axis. In most of the cases
angular momentum about axis is I.

: 0744-2209671, 08003899588 | url : www.motioniitjee.com, : info@motioniitjee.com


Page # 114 ROTATIONAL DYNAMICS

Ex.30 Two small balls A and B, each of mass m, are attached rigidly to the ends of a light rod
of length d. The structure rotates about the perpendicular bisector of the rod at an
angular speed . Calculate the angular momentum of the individual balls and of the
system about the axis of rotation.
d

Sol. A
O B

Consider the situation shown in figure. The velocity of the ball A with respet to the centre O
d
is v = .
2
The angular momentum of the ball with respect to the axis is
 d   d  1
L1 = mvr = m    = md2. The same the angular momentum L2 of the second ball. The
 2   2 4
angular momentum of the system is equal to sum of these two angular momenta i.e., L = 1/2
md2.

9. CONSERVATION OF ANGULAR MOMENTUM :

The time rate of change of angular momentum of a particle about some referenence point in
an inertial frame of reference is equal to the net torques acting on it.

 dL
or net  ....(i)
dt
 

Now, suppose that  net  0 , then d L  0 , so that L = constant.
dt
"When the resultant external torque acting on a system is zero, the total vector angular
momentum of the system remains constant. This is the principle of the conservation of
angular momentum.
For a rigid body rotating about an axis (the z-axis, say) that is fixed in an inertial reference
frame, we have
Lz = I
It is possible for the moment of inertia I of a rotating body to change by rearrangement of its
parts. If no net external torque acts, then Lz must remains constant and if I does change,
there must be a compensating change in . The principle of conservation of angular momentum
in this case is expressed.
I = constant.

Ex.31 A wheel of moment of inertia I and radius R is rotating about its axis at an angular
speed 0. It picks up a stationary particle of mass m at its edge. Find the new angular
speed of the wheel.
Sol. Net external torque on the system is zero. Therefore, angular momentum will remain conserved.
Thus,
I1 1
I11 = I22 or 2 = I
2
Here, I1 = I, 1 = 0, I2 = I + mR2
I 0
 2 =
I  mR 2

Corporate Head Office : Motion Education Pvt. Ltd., 394 - Rajeev Gandhi Nagar, Kota-5
ROTATIONAL DYNAMICS Page # 115

Note :
A Hinge
m

u u
O O
m m m
Case I Case II

Comments on Linear Momentum :


In case I : Linear momentum is not conserved just before and just after collision because during
collision hinge force act as an external force.
In case II : Linear momentum is conserved just before and just after collision because no external
force on the string.
Comments on Angular Momentum :
In case I : Hinge force acts at an external force during collision but except point A all the other
reference point given net  0. So angular momentum is conserved only for point A.
In case II : angular momentum is conserved at all points in the world.

Ex.32 A uniform rod of mass m and length  can rotate freely on a smooth horizontal plane
about a vertical axis hinged at point H. A point mass having same mass m coming with
an initial speed u perpendicular to the rod, strikes the rod in-elastically at its free end.
Find out the angular velocity of the rod just after collision?
m, 
×
H
u

m
Sol. Angular momentum is conserved about H because no external force is present in horizontal
plane which is producing torque about H.

 m 2 2
 3u
mu =  3  m    w =
  4

Ex.33 A uniform rod of mass m and length  can rotate freely on a smooth horizontal plane
about a vertical axis hinged at point H. A point mass having same mass m coming with
an initial speed u perpendicular to the rod, strikes the rod and sticks to it at a distance
of 3/4 from hinge point. Find out the angular velocity of the rod just after collision?
m, 

m,
Sol. H Initial position
3/4 u
m
from angular momentum conservation about H
initial angular momentum = final angular momentum
2
3  3  ml 2
m.u  m   + 
4  4 3

: 0744-2209671, 08003899588 | url : www.motioniitjee.com, : info@motioniitjee.com


Page # 116 ROTATIONAL DYNAMICS

m, 
3mu 1 9  H
  m 2    m
4  3 16 
3u  16  27 
 
4  48 

36 u
 
43 

Ex.34 A uniform rod AB of mass m and length 5a is free to rotate on a smooth horizontal
table about a pivot through P, a point on AB such that AP = a. A particle of mass 2m
moving on the table strikes AB perpendicularly at the point 2a from P with speed v, the
1
rod being at rest. If the coefficient of restitution between them is , find their speeds
4
immediately after impact.
Sol. Let the point of impact be Q so that
PQ = 2a
Let P be the point of pivot that AP = a
5a
Pm Q
A B
C v
a
2m
Before Collision
Let the velocities of point, Q and the particle after impact
be vq and vp respectively then from momentum conservation about point P.
Li = Lf
2a(2mv) = Ip + (2a) (2mvp) ...(i)  Vq
P
2 2 C Q
1  5a   3a  use parallel 
IP  m   m     3a/2
3  2  2 axis theorem Vp
After Collision
13 ma 2
 ...(ii)
3
use equation (ii) in equation (i)
13 ma2
4ma(v – vp) = 
3
12(v – vp) = 13a ....(iii)
velocity of seperation
coefficient of restitution e =
velocity of approach
1 v q  vp

4 v
v
 v q – vp = ...(iv)
4
vq = 2a ...(v)
Put value of  from eq (iii) to equation (v)
 12 
vq  2   (v  vp )
 13 

Corporate Head Office : Motion Education Pvt. Ltd., 394 - Rajeev Gandhi Nagar, Kota-5
ROTATIONAL DYNAMICS Page # 117

So now from equation (iv)


24 v 83 v
( v – vp )  v p   vp 
13 4 148
15 v
So in this way we get  
37 a
Ex.35 A person of mass m stands at the edge of a circular platform of radius R and moment
of inertia. A platform is at rest initially. But the platform rotate when the person jumps
off from the platform tangentially with velocity u with respect to platfrom. Determine
the angular velocity of the platform.
Sol. Let the angular velocity of platform is . Then the velocity of person with respect to ground
v.
vmD = vmG – VDG

u = vm + R R
vm = u –  R M
Now from angular momentum conservation
R
Li = Lf
0 = mvmR – I 
 I  = m (u –  R) . R u
muR
 =
I  mR 2
Ex.36 Consider the situation of previous example. If the platform is rotating intially with
angular velocity 0 and then person jumps off tangentially. Determine the new angular
velocity of the platform.
Sol. Let the angular velocity of platfrom after jumps off the mass is . Then velocity Of man.
0

R
R

u
Initially
vm = vmp + vp
vm = u – R
From Angular momentum conservation
(I + mR2) 0 = I  – m (u –  R) R
I0 + mR2 0 = I  – m u R + m  R2
(I  mR2 )0  mu R
 
(I  mR 2 )

10. ANGULAR IMPULSE


t2 
The angular impulse of a torque in a given time interval is defined as  t1
 dt

Here, 
 is the resultant torque acting on the body. Further, since

 dL 
     dt  d L
dt

: 0744-2209671, 08003899588 | url : www.motioniitjee.com, : info@motioniitjee.com


Page # 118 ROTATIONAL DYNAMICS

t2   
or  t1
 dt = angular impulse = L 2 – L 1

Thus, the angular impulse of the resultant torque is equal to the change in angular momentum.
Let us take few examples based on the angular impulse.

Ex.37 Figure shows two cylinders of radii r1 and r2 having moments of inertia I1 and I2 about
their respective axes. Initially, the cylinders rotate about their axes with angular speeds
1 and 2 as shown in the figure. The cylinders are moved closer to touch each other
keeping the axes parallel. The cylinders first slip over each other at the contact but the
slipping finally ceases due to the friction between them. Find the angular speeds of
the cylinders after the slipping ceases.
2
1
r I1 r I2
1 2

Sol. When slipping ceases, the linear speeds of the points of contact of the two cylinders will be
equal. If  1' and  2 ' be the respective angular speeds, we have
 1 ' r1   2 ' r2 .....(i)
The change in the angular speed is brought about by the frictional force which acts as long
as the slipping exists. If this force f acts for a time t, the torque on the first cylinder is fr1 and
that on the second is f r2. Assuming 1 r1 > 2 r2, the corresponding angular impluses are – f
r1 t and f r2 t. We, therefore, have
– f r1 t = I1 ( 1 '   1)
and f r2 t = I2 ( 2 '   2 )
I1 I
or, – ( 1'–1 ) = 2 ( 2 '– 2 ) ...(ii)
r1 r2
Solving (i) and (ii),
I1 1r2  I2  2r1 I1 1r2  I2 2r1
 1' = r2 and  2 '  r1
I2r12  I1r22 I2r12  I1r22
Kinetic Energy of a rigid body rotating about a fixed axis.
Suppose a rigid body is rotating about a fixed axis with angular speed . 
Then, kinetic energy of the rigid body will be :
ri
1 1 mi
K=  mi v i2 =  mi (ri ) 2
i
2 i
2

1 2 2 1 2
= 2 m r
i
ii =
2
I (as m r ii
2
 I)
i

1 2
Thus, KE = I
2
Sometimes it is called the rotational kinetic energy.
Ex.38 A uniform rod of mass m and length  is kept vertical with the lower end clamped. It is
slightly pushed to let it fall down under gravity. Find its angular speed when the rod is
passing through its lowest position. Neglect any friction at the clamp. What will be the
linear speed of the free end at this instant?
Sol. As the rod reaches its lowest position, the centre of mass is lowered by a distance . Its
gravitational potential energy is decreased by mg. As no energy is lost against friction, this
should be equal to the increase in the kinetic energy. As the rotation occurs about the
horizontal axis through the clamped end, the moment of inertia is I = m 2/3. Thus,

Corporate Head Office : Motion Education Pvt. Ltd., 394 - Rajeev Gandhi Nagar, Kota-5
ROTATIONAL DYNAMICS Page # 119

1 2 1  m 2  2
I  mg     = mg 
2 2  3 

6g 
or =



The linear speed of the free end is
v =  = 6g

11. COMBINED TRANSLATIONAL AND ROTATIONAL MOTION OF A RIGID BODY :


We have already learnt about translational motion caused by a force and rotational motion
about a fixed axis caused by a torque. Now we are going to discuss a motion in which body
undergoes translational as well as rotational motion. Rolling is an example of such motion. If
the axis of rotation is moving then the motion is combined translational and rotational motion.
To understand the concept of combined translational and rotational motion we consider a
uniform disc rolling on a horizontal surface. Velocity of its centre of mass is Vcom and its
angular speed is  as shown in figure.

R A v
Let us take a point A on the disc and concentrate on its motion.
Path of point A with respect to ground will be a cycloid as shown in figure.

A

v A A

A A
Motion of point A with respect to center of mass is pure rotational while center of mass itself
is moving in a straight line. So for the analysis of rolling motion we deal translational motion
seperately and rotational motion seperately and then we combine the result to analyses the
over all motion.
The velocity of any point A on the rigid body can be obtained as
  
VA  VCOM  VA COM

| VCOM |  V

| VA.COM | r in the direction  to line OA
 
Thus, the velocity of point A is the vector sum of VCOM and VP.COM as shown in figure


r VA O

A VCOM

Important points in combined Rotational + translation motion :

: 0744-2209671, 08003899588 | url : www.motioniitjee.com, : info@motioniitjee.com


Page # 120 ROTATIONAL DYNAMICS

1. Velocity of any point of the rigid body in combined R + T motion is the vector sum of
v(velocity of centre of mass) and r
for example
A disc of radius r has linear velocity v and angular velocity  as shown in figure then find
velocity of point A. B, C, D on the disc
C

r
B v
D

A
We divide our problem in two parts
(1) Pure Rotational + (2) Pure Translational
about centre of mass.
C
r
r v

B D v
r + v
r
r A
v
Then combine the result of above both
r C v  r
2 )
r
(
v2

D
B v

v  (r )2
2

A v – r
r

2. In combined rotational and translational motion angular velocity of any point of a


rigid body with respect to other point in the rigid body is always same.
For example :
C
2v
v
2v
v D
B v D
v
(v = r)
2r

r 2v
v=0 2v
v
A A

2v v
Now for DA DA = =
2r r
C 2v

2R
For CA :

Corporate Head Office : Motion Education Pvt. Ltd., 394 - Rajeev Gandhi Nagar, Kota-5
ROTATIONAL DYNAMICS Page # 121

2v v
CA = =
2r r
For DB :

2v
2r D
B 2v 2v
vDB =
2v
2v

2v v
 DB = =
2r r

3. Distance moved by the centre of mass of the rigid body in one full rotation is 2R.

This can be shown as under :


In one rotation angular displacement  = 2 = t
 2 
s = v . T = ( R)    2R
 
In forward slipping s > 2R (as v > R)
and in backward slipping s < 2R (as v < R)
t
4. The speed of a point on the circumference of the body at any instant t is 2R sin
2
Proof :
vxp = v – v cos  = v[1 – cos ] 
vyp = v sin 
 v
| v p |  v 2 sin 2   v 2 (1 – cos )2 v = R

v= 2v 2 – 2v 2 cos  P v

= 2v(1 – cos )1/2

 t   ωt 
= 2v sin    = 2v sin  = 2R sin 
 2  2   2

5. The path of a point on circumference is a cycloid and the distance moved by this point
in one full rotation is 8R.
A3
A2 A4

A1 A5

: 0744-2209671, 08003899588 | url : www.motioniitjee.com, : info@motioniitjee.com


Page # 122 ROTATIONAL DYNAMICS

In the figure, the dotted line is a cycloid and the distance A1 A2 ......A5 is 8R. This can be
proved as under.
According to point (3), speed of point A at any moment is,

 t 
vA = 2R sin  2 

Distance moved by A in time dt is,


c c
 t  v
ds = vA dt = 2R sin   dt  A 
2

A t=t
Therefore, total distance moved in one full rotation is, t=0
T  2 / 

S=  ds
0

T2/ 
 t 
or S=  2 R sin  dt
 2
0
On integration we get, s = 8R

6. x and y coordinates of the bottommost point at any time t.


At time t the bottommost point will rotate an angle  = t with respect to the centre of the
disc C. The centre C will travel a distance s = vt.
In the figure, PQ = R sin  = R sin t y
CQ = R cos = R cost
Coordinates of point P at time t are,
C C
R

x = OM – PQ = vt – R sin t P Q
and y = CM – CQ = R – R cos t O M
 (x, y)  (vt – R sin t, R – R cos t) s=vt

11.1 Angular momentum of a rigid body in combined rotation and translation

COM

 v0

r0
O
Let O be a fixed point in an inertial frame of reference. Angular momentum of the body about
O is.
   
L  L cm  M( r 0  v 0 )

The first term L cm represents the angular momentum of the body as seen from the centre of
 
mass frame. The second term M( r 0  v 0 ) equals the angular momentum of centre of mass
about point O.

Corporate Head Office : Motion Education Pvt. Ltd., 394 - Rajeev Gandhi Nagar, Kota-5
ROTATIONAL DYNAMICS Page # 123

Ex.39 A circular disc of mass m and radius R is set into motion on a horizontal floor with a
v
linear speed v in the forward direction and an angular speed   in clockwise direction
R
as shown in figure. Find the magnitude of the total angular momentum of the disc
about bottommost point O of the disc.
   
Sol. L  L cm  m( r0  v 0 ) ...(i)
 v
Here, L cm  I (perpendicular to paper inwards)

1 2 v  O
  mR   
2  R

1 r0
 mvR
2
 
and m( r0  v 0 )  mRv (perpendicular to paper inwards) 90º 
v0
Since, both the terms of right hand side of Eq. (i) are in the
same direction.
O
 1
 | L | mvR  mvR
2
 3
or | L | mvR Ans.
2
11.2 Kinetic Energy of a Rolling Body
If a body of mass M is rolling on a plane such that velocity of its centre of mass is V and its
angular speed is , its kinetic energy is given by

1 1
KE = Mv 2  I  2
2 2
I is moment of inertia of body about axis passing through centre of mass.
In case of rolling without slipping.

1 1
KE = M  2 R 2 + I  2 [ v = R]
2 2

1 1

2
 
MR2  I  2 =
2
Ic  2

Ic is moment of inertia of the body about the axis passing through point of contact.
Ex.40 A uniform rod of mass M and length a lies on a smooth horizontal plane. A particle of
mass m moving at a speed v perpendicular to the length of the rod strikes it at a
distance a/4 from the centre and stops after the collision. Find (a) the velocity of the
centre of the rod and (b) the angular velocity of the rod about its centre just after the
collision.

r0
A a A v
a/4
Sol.

(a) (b)

: 0744-2209671, 08003899588 | url : www.motioniitjee.com, : info@motioniitjee.com


Page # 124 ROTATIONAL DYNAMICS

The situation is shown in figure. Consider the rod and the particle together as the system. As
there is no external resultant force, the linear momentum of the system will remain constant.
Also there is no resultant external torque on the system and so the angular momentum of the
system about any line will remain constant.
Suppose the velocity of the centre of the rod is V and the angular velocity about the centre
is .
(a) The linear momentum before the collision is mv and that after the collision is MV. Thus,

m
mv = MV, or V = v
M
(b) Let A be the centre of the rod when it is at rest. Let AB be the line perpendicular to the
plane of the figure. Consider the angular momentum of "the rod plus the particle" system
about AB. Initially the rod is at rest. The angular momentum of the particle about AB is
L = mv(a/4)
After the collision, the particle mass to rest. The angular momentum of the rod about A is
   
L  L cm  M( r 0  V )

   
As r 0 || V , r0  V  0

 
Thus, L  L cm
Hence the angular momentum of the rod about AB is

Ma 2
L  I  
12

mva Ma2 3mv


Thus,   or, =
4 12 Ma

Ex.41 A uniform rod of length  lies on a smooth horizontal table A particle moving on the
table has a mass m and a speed v before the collision and it sticks to the rod after the
collision. The rod has a mass M then find out.
(a) The moment of inertia of the system about the vertical axis passing through the centre
of mass C after the collision.
(b) The velocity of the centre of mass C and the angular velocity of the system about the
centre of mass after the collision.
Sol. Figure shows the situation of system just before and just after collision.
Initially the centre of mass of the rod is at point O. After collision when the particle sticks to
the rod. Centre of mass is shifted from point O to C as shown in figure. Now the system is
rotated about axis passing through C
v A M
1 
m 2(m  M)
v'
/2 C m
 2 
2(M  m)
O M O

/2
M
Before collision After collision

Corporate Head Office : Motion Education Pvt. Ltd., 394 - Rajeev Gandhi Nagar, Kota-5
ROTATIONAL DYNAMICS Page # 125

Now from linear momentum conservation


mv
mv = (M + m) v  v' 
Mm
(a) Let us assume that moment of inertia of the system about C is 1. Then I  I(rod)C  I(part ) C

I  I0  M 22  m 21

M 2 Mm2  2 mM2  2 M(M  4m) 2


I  2
 2  I 
12 4(m  M) 4(m  M) 12(m  M)
(b) From Angular momentum conservation about A
Li = Lf
0 + 0 = I  – (m + M) v 1  I = (m + M) v 1
Put the value of I, v, & 1 we get

6mv
=
(M  4m)

11.3 Acceleration of a point on the circumference of the body in R + T motion :


(A) Both  & v are constant :
C 
r
 2r
D
 2r  2r
 2r

(B) When  is constant and v is variable.

 

v1, a v2= v1+at

t=0 t=t
So acceleration of different point on the body is given by following figure.

a
2
 r  2r
 2r  2r
a a
+  2r = a  2r
2
 r  2r
(Translational) (Rotational) a
(Combined R + T)

(C) When  is variable and v is constant :


   i  t
 i,
v v

t=0 t=t

: 0744-2209671, 08003899588 | url : www.motioniitjee.com, : info@motioniitjee.com


Page # 126 ROTATIONAL DYNAMICS

So acceleration of different point on the body is given by following way

R
 R 2
 R
 2r  2R
 2r a=0
 2  2R
 2r  R
 2r R
R

(D) When both  & v are variable :


i,  f   i  t

v1, a v2=v1+at

time t =0 time t =t
Now the net acceleration of different points on the rigid body is given by following way.
R a
R R
R 2
 2r
 R  2r
 2R =
a + a
2  2R  2
r
 R R
R R a
R
(Translational) (combined Rotational
(Rotational)
+ Translational)
Ex.42 A force F acts at the centre of a thin spherical shell of mass m and radius R.
Find the acceleration of the shell if the surface is smooth. N
Sol.  Force F, mg & N passes through centre so
net = 0, i.e., body is in rotational equilibrium F
R
 F
But F net  F so body moves with constant acceleration a =
m mg
Ex.43 In a previous problem if force F applied at a distance x above the centre then find out
linear and angular acceleation.
Sol. This force F translate the body linearly as well as rotate it. So,
Net toruqe about O it 0 = Fx
N
From rotational motion 0 = I 
 Fx F
  3Fx x R
2
I 2MR   O a
2
3 2MR mg
From linear motion of sphere smooth
F
F = ma  a=
m
Ex.44 A rigid body of mass m and radius r starts coming down an inclined plane of inclination
. Then find out the acceleration of centre of mass if friciton is absent.
Sol. Friction is absent so body is moving down the incline with out
rolling so acceleration of centre of mass is g sin  
in
gs

Corporate Head Office : Motion Education Pvt. Ltd., 394 - Rajeev Gandhi Nagar, Kota-5
ROTATIONAL DYNAMICS Page # 127

12. UNIFORM PURE ROLLING


Pure rolling means no relative motion (or no slipping at point of contact between two bodies.)
For example, consider a disc of radius R moving with linear velocity v and angular velocity 
on a horizontal ground. The disc is said to be moving without slipping if velocities of points P and
Q (shown in figure b) are equal, i.e.,

v 
COM
 P
R v
Q
(a) (b)
vp = vQ
or v – R = 0 or v = R
If vp > vQ or v > R, the motion is said to be forward slipping and if vp < vQ < R, the motion is
said to be backward slipping.
Now, suppose an external force is applied to the rigid body, the motion will no longer remain
uniform. The condition of pure rolling on a stationary ground is,
a = R
Thus, v = R, a = R is the condition of pure rolling on a stationary ground. Sometime it is
simply said rolling.

Note : We can represent the moment of inertia of a different rigid body in a following way.
I = CMR2
1
value of C = 1 for circular ring (R), C = for circular disc (D) and solid cylinder (S.C.)
2
2 2
C= for Hollow sphere (H.S) , C= for solid sphere (S.S)
3 5
Ex.45 A rigid body I = CMR2 is set into a motion on a rough horizontal surface with a linear
speed v0 in the forward direction at time t = 0 as shown in figure. After what time
slipping finally stop and pure rolling starts. Find the linear speed of the body after it
starts pure rolling on the surface.

v0
O
R
at t = 0

Sol. According to the given condition in problem the point P in the body move with speed v0 while
the point Q on the ground is at rest. So the friciton acts on the body is in backward direction
which gives the resultant torque on the body and increase the angualr speed  as shown in
figure.
1 1
v1  1R
O
v0 O v1 O v1 v1 R

P  1R
f v0 v1 R v1
f
(kinetic) Q (kinetic) friction
Q Q
at t = 0 at t = t1 static
at t = t
As shown in above figure initially v > R so forward slipping takes place. After introducing the
friciton speed decreases and  increases and at time t = t the relation v = r  is satisfied.
Therefore pure rolling starts. Initially the friciton is kinetic untill the motion is in slipping
condition. Afterwards at v = r fricition is static. We divide the above problem in two parts.

: 0744-2209671, 08003899588 | url : www.motioniitjee.com, : info@motioniitjee.com


Page # 128 ROTATIONAL DYNAMICS

(1) Translational Motion :


Linear acceleration a = – g
So after time t, v = v0 – gt ...(1)
(2) Rotational Motion :
From net = I 
Only friction force is responsible for providing torque. So torque about O is
f. R = I 
mgR = CmR2 ...(2)
 is angular acceleration of the body

μg
from eq. (2) =
CR
from f = i +  t

g
 = t   = .t
CR

v
 = at pure rolling condition.
R
μgt
So, v= ...(3)
C
from eq. (1) & (3)

μgt v 0C
 v0 – gt =  t= ...(4)
C μg(1  C)
Equation (4) gives the time after the pure rolling starts.
Put the value from eq. (4) to eq. (1)

v 0C v0
v = v0 –  v= ...(5)
(1  C) 1 C

Equation (5) gives the linear speed at pure rolling situation.


Alternate solution :
Net torque on the body about the bottom most point A is zero. Therefore angular momentum
of the body will remain conserved about the bottom most point
Net torque about A A = 0
v
 from Angular momentum conservation Li = Lf 
R
mv0R = I + mvR
v0 v
v
mv0R = CmR2 + mvR
R

v0 A f A f
v0 = Cv + v  v=
1 C

Corporate Head Office : Motion Education Pvt. Ltd., 394 - Rajeev Gandhi Nagar, Kota-5
ROTATIONAL DYNAMICS Page # 129

Ex.46 In the previous problem take rigid body a solid cylinder then find out the work done by
friciton from time t = 0 to t = t (at v = r)
Sol. Let us suppose that in between time t = 0 to t = t cylinder displaced s.
t=0 t=t

v0
v  R

S
Translational work done by friciton + Rotational work Done by friciton
Now calculate each type of work done one by one

(A) Translational work done by friciton :


1
for solid cylinder c =
2

v0 2
from eq. (5) v   v0
1 3
1
2
2
2  2
from eq. v 2f  ui2  2as   v 0   ( v 0 ) – 2gs
3 

5 v 20
s=
18g
Translation W.D by friciton = – f.s

5 v 20 5mv 20
( w.D) f T  – mg. = –
18g 18
(B) Rotational W.D. by friciton :
We known that  = I 

= ...(a)
I
from  f 2 – i2  2 
2
 2v 0  2
   .
 3R  I

mR2 v 02m
Put I =  =
2 9
Rotation W.D by friciton W = .

v 02m
Wf R =
9

5 v 2m
(C) So total W.D. by friciton W = Wf T + Wf R
= – mv 20  0
18 9
mv 20
W= –
6

: 0744-2209671, 08003899588 | url : www.motioniitjee.com, : info@motioniitjee.com


Page # 130 ROTATIONAL DYNAMICS

Alternative Method :
from work – Energy Theorem
work done by friciton = change in kinetic energy
(W.D)f = K = kf – ki
Now
1 1
kf = mv 2f + I 2
2 2
2 2
1  2v 0  1 mR2  2v 0   2v 0 
kf = m  +    v f  
2  3  2 2  3R   3 

mv 20
kf =
3
1
ki = mv 20
2

mv 20 1 mv 20
So., (w.D)f = – mv 20  ( w.D)f  –
3 2 6
To calculate work done mostly prefer alternative method.

Ex.47 A solid sphere of radius r is gently placed on a rough horizontal ground with an initial
angular speed 0 and no linear velocity. If the coefficient of friciton is , find the linear
velocity v and angular velocity  at the end of slipping.

0

Sol. m be the mass of the sphere.


Since, it is a case of backward slipping, force of
friction is in forward direction. Limiting friciton will
act in this case.
Net torque on the sphere about the bottommost
point is zero. Therefore, angular momentum of
the sphere will remain conserved about the
bottommost point.
Li = Lf v
 I0 = I + mrv 0 fmax 0
2 2 2
or mr  0 = mr 2   mr(r )
5 5
2 2
 =  0 and v = r = r 0
7 7
12.1 Pure rolling when force F act on a body :
Suppose a force F is applied at a distance x above the centre of a rigid body of radius R, mass
M and moment of inertia CMR2 about an axis passing through the centre of mass. Now, the
applied force F can produces by itself
(i) a linear acceleration a and
(ii) an angular acceleration 
If a = R, then there is no need of friction and force of friction f = 0,
If a < R, then to support the linear momentum the force of friciton f will act in forward
direction,

Corporate Head Office : Motion Education Pvt. Ltd., 394 - Rajeev Gandhi Nagar, Kota-5
ROTATIONAL DYNAMICS Page # 131

Similarly, if a > R, then no support the angular motion the force of friciton will act in
backward direction.
So, in this case force of friction will be either backward, forward or even zero also. It all
depends on M, I and R. For calculation you choose any direction of friction. Let we assume it
in forward direction.
Let, a = linear acceleration, 
 = angualr acceleration
F
from linear motion x
F + f = Ma ...(1) C a
from rotational motion.
Fx – f R = I  f
a
Fx – f R = CMR2.
R
Fx – f R = CMaR ...(2)
from eq. (1) and (2)
F(x+r) = MaR (C + 1)
F(R  x )
a= ...(3)
MR(C  1)
Put the value from eq. (3) to eq. (1)
F(x – RC)
f=
R(C  1)
f should be   s mg for pure rolling

Ex.48 Consider the arrangment shown in figure. The string is wrapped around a uniform
cylinder which rolls without slipping. The other end of the string is passed over a
massless, frictionless pulley to a falling weight. Determine the acceleration of the falling
mass m in terms of only the mass of the cylinder M, the mass m and g.
M

Sol. Let T be the tension the string and f the force of (static) friction, between the cylinder and
the surface
a1 = acceleration of centre of mas of cylinder towards right
a2 = downward acceleration of block m
 = angular acceleration of cylinder (clockwise)
Equations of motion are :
For block mg – T = ma2 ...(i)
For cylinder, T + f = Ma1 ...(ii)
( T – f )R

1 ...(iii)
MR2
2
The string attaches the mass m to the highest point of the cylinder, hence
vm = vCOM + R
Differentiating, we get
a2 = a1 + R ...(iv)
We also have (for rolling without slipping)
a1 = R ...(v)
8mg
Solving these equations, we get a2 
3M  8m

: 0744-2209671, 08003899588 | url : www.motioniitjee.com, : info@motioniitjee.com


Page # 132 ROTATIONAL DYNAMICS

Note : Work done by friction in pure rolling on a stationary ground is zero as the point of application
of the force is at rest. Therefore, machanical energy can be conserved if all other dissipative
forces are ignored.
12.2 Pure Rolling on an Inclined Plane:
A rigid body of radius R, and mass m is released
at rest from height h on the incline whose
inclination with horizontal is  and assume that
 f
friciton is sufficient for pure rolling then.
a = R and v = R
a
From figure
mg sin  – f = ma ...(1) 
in
{Fnet = ma} gs
m
2 a
f.R = cmR . ...(2)
R
{Fnet = I}
from eq. (1) & (2)
g sin
a=
1 c
So body which have low value of C have greater acceleration.
value of C = 1 for circular ring (R)
1
C= for circular disc (D) and solid cylinder (S.C.)
2
2
C= for Hollow sphere (H.S)
3
2
C= for solid sphere (S.S)
5
So, descending order of a
aS.S > aD = aS.C > aH.S. > aR
and order of time of descend is
ts.s < tD = ts.c < tH.S < tR

Kinetic Eneregy : Work done by friction in pure rolling is zero. Therefore,


Increase in kinetic energy = change in potential energy  K.E. = mgh
i.e., kinetic energy is constant for all rigid body rolling down the incline.

Requirement of Friction :
From eq. ...(2)
f = Cma

mg sin 
f ...(3)
 1
1  
 C
from eq. (3) as the value of C increase requirement of friciton is increases.

Ex.49 A cylinder of mass M is suspended through two strings wrapped around it as shown in
figure. Find the tension in the string and the speed of the cylinder as it falls through a
distance h.
Sol. The portion of the strings between ceiling and cylinder are at rest. Hence the points of the
cylinder where the strings leave it are at rest also. The cylinder is thus rolling without slipping
on the strings. Suppose the centre of cylinder falls with an acceleration a. The angular
acceleration of cylinder about its axis given by

Corporate Head Office : Motion Education Pvt. Ltd., 394 - Rajeev Gandhi Nagar, Kota-5
ROTATIONAL DYNAMICS Page # 133

a
= ...(i)
R
as the cylinder does not slip over the strings.
The equation of motion for the centre of mass of cylinder is
Mg – 2T = Ma T T
and for the motion about the centre of mass it is
 MR2  MR2
2T.R =   , where I =
 2  2
mg

MR2 a Ma
2TR=  2T= ...(ii)
2 R 2
From (i) and (ii) on adding
Ma 3a
Mg =  Ma ; g
2 2
2g
a=
3
M 2g Mg
 2T= .  T=
2 3 6
As the centre of cylinder starts moving from rest, the velocity after it has fallen a height h is
given by
 2g  4gh
v2 = 2  3 h or v =
  3

Ex.50 A thin massless thread is wound on a reel of mass 3kg and moment of inertia 0.6 kg-
m2. The hub radius is R = 10 cm and peripheral radius is 2R = 20 cm. The reel is placed
on a rough table and the friction is enough to prevent slipping. Find the acceleration of
the centre of reel and of hanging mass of 1 kg.

2R
R

Sol. Let, a1 = acceleration of centre of mass of reel


a2 = acceleration of 1 kg block
 = angular acceleration of reel (clockwise)
T = tension in the string 
and f = force of friction
Free body diagram of reel is as shown below :
(only horizontal forces are shown). a1
Equations of motion are : T
T – f = 3a1 ...(i) f
 f ( 2R) – T.R 0.2f – 0.1T f T T
   = – ...(ii)
I I 0.6 3 6
Free body diagram of mass is, a2
Equation of motion is,
10 – T = a2 ...(iii)

10N

: 0744-2209671, 08003899588 | url : www.motioniitjee.com, : info@motioniitjee.com


Page # 134 ROTATIONAL DYNAMICS

For no slipping condition,


a1 = 2R or a1 = 0.2 ...(iv)
and a2 = a1 – R or a2 = a1 – 0.1 ...(v)
Solving the above five equations, we get
a1 = 0.27 m/s2 and a2 = 0.135 m/s2

Ex.51 Determine the maximum horizontal force F that may be applied to the plank of mass
m for which the solid sphere does not slip as it begins to roll on the plank. The sphere
has a mass M and radius R. The coefficient of static and kinetic friction between the
sphere and the plank are s and k respectively.
M
R

m F

Sol. The free body diagrams of the sphere and the plank are as shown below :
Writing equations of motion : 
For sphere : Linear acceleration a1
 Mg
a1 = s  sg ...(i)  s Mg
M
Angular acceleration  s Mg a2
F M
( sMg)R 5  s g
 
2 2 R ..(ii) R
MR2
5 B a1  R
For plank : Linear acceleration A a2
m F
F –  sMg ..(iii)
a2 
m
For no slipping acceleration of point B and A is same,
so : a2 = a1 + R
Solving the above four equation, we get
 7 
F   s g M  m
 2 
 7 
Thus, maximum value of F can be  s g M  2 m

Ex.52 Find out the maximum height attained by the solid sphere on a friciton less track as
shown in figure.

v0
0 
R
R v0

Corporate Head Office : Motion Education Pvt. Ltd., 394 - Rajeev Gandhi Nagar, Kota-5
ROTATIONAL DYNAMICS Page # 135

Sol. Let us assume that sphere attain a maximum height H on the track.
0
Final Position
v=0

v0 H
0 
R
R v0

Initial Position
As the sphere move upward speed is decreased due to gravity but there is no force to
change the 0 (friction less track). So from energy conservation
1 1 1 v2
mv 20  I 20 = mg Hmax + I 20 , Hmax = 0
2 2 2 2g
13. TOPPLING
You might have seen in your practical life that if a force F is applied to a block A of smaller
width it is more likely to topple down, before sliding while if the same force F is applied to an
another block B of broader base, chances of its sliding are more compared to its toppling.
Have you ever throught why it happens so. To understand it better let us take an example.
F F
A B

Suppose a force F is applied at a height b above the base AE of the block. Further, suppose
the friction f is sufficient to prevent sliding. In this case, if the normal reaction N also passes
through C, then despite the fact that the block is in translational equilibium (F = f and N =
mg), an unbalanced torque (due to the couple of forces F and f) is there. This torque has a
tendency to topple the block about point E. To cancel the effect of this unbalanced torque
the normal reaction N is shifted towards right a distance 'a' such that, net anticlockwise
torque is equal to the net clockwise torque or
N
B D
F
C b
f
A E
W=mg
Fb = (mg) a
Fb
or a=
mg
N N
B D B D
F F
C b C b
a
f f
A E A E
mg mg
(a) (b)
Now, as F or b (or both) are increased, distance a also increases. But it can not go beyond
the right edge of the block. So, in extreme case (beyond which the block will topple down),
the normal reaction passes through E as shown in figure.
Now, if F or b are further increased, the block will topple down. This is why the block having
the broader base has less chances of toppling in comparison to a block of smaller base.
Because the block of larger base has more margin for the normal reaction to shift.

: 0744-2209671, 08003899588 | url : www.motioniitjee.com, : info@motioniitjee.com


Page # 136 ROTATIONAL DYNAMICS

Why the rolling is so easy on the ground.


N
F

mg
Because in this case the normal reaction has zero margin to shift. so even if the body is in
translational equilibrium (F = f, N = mg) an unbalanced torque is left behind and the body
starts rolling clockwise. As soon as the body starts rolling the force of friction is so adjusted
(both in magnitude and direction) that either the pure rolling starts (if friciton is sufficient
enough) or the body starts sliding. Let us take few examples related to toppling.

Ex.53 A uniform block of height h and width a is placed on a rough inclined plane and the
inclination of the plane to the horizontal is gradually increased. If  is the coefficient of
friction then under condition the block will
(A) slide before toppling :
The block will slide when
mg sin  > f
 mg sin  >  mg cos 
 tan  > 
i.e., block is at rest when
tan . ...(1)
(B) Now suppose the friction f is sufficient to prevent sliding. Then we assume that N is shifted
towards downward a distance x to prevent toppling Therefore. torque about O is zero.

h h f
 f. =Nx N
2
xO
h
mg sin . = mg cos .x a
2 
cos
x=
tan .h
i n m
g
2 gs
m
Maximum value of x is a/2 
a
so to prevent toppling x 
2
tan .h
  a/2
2
a
 tan   ...(2)
h
So, the block topple before sliding from (1) & (2)
a
s >
h

Corporate Head Office : Motion Education Pvt. Ltd., 394 - Rajeev Gandhi Nagar, Kota-5
ROTATIONAL DYNAMICS Page # 137

14. INSTANTANEOUS AXIS OF ROTATION


The combined effects of translation of the centre of mass and rotation about an axis through
the centre of mass are equivalent to a pure rotation with the same angular speed about an
axis passing through a point of zero velocity. Such an axis is called the instantaneous axis of
rotation. (IAOR). This axis is always perpendicular to the plane used to represent the motion
and the intersection of the axis with this plane defines the location of instantaneous centre
of zero velocity (IC).


IC
For example consider a wheel which rolls without slipping. In this case the point of contact
with the ground has zero velocity. Hence, this point represents the IC for the wheel. If it is
imagined that the wheel is momentarily pinned at this point, the velocity of any point on the
wheel can be found using v = r. Here r is the distance of the point from IC. Similarly, the
kinetic energy of the body can be assumed to be pure rotational about IAOR or,
P v
vP P
r r 
v vP  r  
r


IC
1
K I IAOR 2
2
Rotation + Translation  Pure rotation about IAOR passing through IC

1 1 1
KE = mv 2COM  ICOM 2  KE  I IAOR 2
2 2 2
14.1 Location of the IC
If the location of the IC is unknown, it may be determined by using the fact that the relative
position vector extending from the IC to a point is always perpendicular to the velocity of the
point. Following three possibilities exist.
(i) Given the velocity of a point (normally the centre of mass) on the body and the
angular velocity of the body

If v and  are known, the IC is located along the line drawn perpendicular to v at P, such that
v
the distance from P to IC is, r  . Note that IC lie on that side of P which causes rotation

 
about the IC, which is consistent with the direction of motion caused by  and v .


P v

IC

: 0744-2209671, 08003899588 | url : www.motioniitjee.com, : info@motioniitjee.com


Page # 138 ROTATIONAL DYNAMICS

Ex.54 A rotating disc moves in the positive direction of the x-axis. Find the equation y(x)
describing the position of the instantaneous axis of rotation if at the initial moment
the centre c of the disc was located at the point O after which it moved with constant
velocity v while the disc started rotating counter clockwise with a constant angular
acceleration . The initial angular velocity is equal to zero.
y

O c v x

x x y
Sol. t and   t 
v v
IC
The position of IAOR will be at a distance y
O c v x
v v
y or y 
 x
v x
v2 v2
or y or xy   constant
x 
This is the desired x-y equation. This equation represents a rectangular hyperbola.

(ii) Given the lines of action of two non-parallel velocities


 
Consider the body shown in figure where the line of action of the velocities v A and v B are
known. Draw perpendiculars at A and B to these lines of action. The point of intersection of
these perpendiculars as shown locates the IC at the instant considered.

 A 
vB vA

IC
B

(iii) Given the magnitude and direction of two parallel velocities


When the velocities of points A and B are parallel and
have known magnitudes vA and vB then the location of
the IC is determined by proportional triangles as shown
in figure. A  IC
vA
vA
In both the cases, rA,IC  
 d A vA
IC
v d
and rB,IC  B  
 vB B vB
B
In fig. (a) rA, I C + rB, I C = d
(a) (b)
and in fig (b) rB, I C – rA, I C = d
As a special case, if the body is translating, vA = vB and the IC would be located at infinity, in
which case  = 0.

Corporate Head Office : Motion Education Pvt. Ltd., 394 - Rajeev Gandhi Nagar, Kota-5
ROTATIONAL DYNAMICS Page # 139

Ex.55 A uniform thin rod of mass m and length l is standing on a smooth horizontal surface.
A slight disturbance causes the lower end to slip on the smooth surface and the rod
starts falling. Find the velocity of centre of mass of the rod at the instant when it
makes an angle  with horizontal.
Sol. As the floor is smooth, mechanical energy of the rod will remain conserved. Further, no
horizontal force acts on the rod, hence the centre of mass moves vertically downwards in a
straight line. Thus velocities of COM and the lower end B are in the direction shown in figure.
 
The location of IC at this instant can be found by drawing perpendiculars to v C and v B at
respective points. Now, the rod may be assumed to be in pure rotational motion about IAOR
passing through IC with angular speed .
A

COM
l
h (1  sin )
 IC 2
vC 
l
sin
2


B vB
Applying conservation of mechanical energy. Decrease in gravitational potential energy of the
rod = increase in rotational kinetic energy about IAOR

1 l 1  ml 2 ml 2 2
 2
 mgh  I IAOR 2 or mg 2 (1  sin )  2  12  4 cos  
2  
Solving this equation, we get
12g(1  sin )

l (1  3 cos 2 )

 l 
Now, | v C |   cos  
2 

3gl (1  sin ) cos 2 


 Ans.
(1  3 cos 2 )

: 0744-2209671, 08003899588 | url : www.motioniitjee.com, : info@motioniitjee.com


Page # 140 ROTATIONAL DYNAMICS

Exercise - I OBJECTIVE PROBLEMS (JEE MAIN)


(A) MOMENT OF INERTIA 4. The M.I. of a disc about its diameter is 2
units. Its M.I. about axis through a point on
1. The moment of inertia of a body depends
its rim and in the plane of the disc is
upon -
(A) 4 units. (B) 6 units
(A) mass only
(C) 8 units (D) 10 units
(B) angular velocity only
Sol.
(C) distribution of particles only
(D) mass and distribution of mass about the axis
Sol.

2. Two spheres of same mass and radius are


in contact with each other. If the moment of
inertia of a sphere about its diameter is I, then 5. A solid sphere and a hollow sphere of the
the moment of inertia of both the spheres same mass have the same moments of inertia
about the tangent at their common point would about their respective diameters, the ratio of
be - their radii is
(A) 3I (B) 7I (C) 4I (D) 5I (A) (5)1/2 : (3)1/2 (B) (3)1/2 : (5)1/2
Sol. (C) 3 : 2 (D) 2 : 3
Sol.

3. A disc of metal is melted to recast in the


form of a solid sphere. The moment of inertias
about a vertical axis passing through the centre
would - 6. A stone of mass 4kg is whirled in a horizontal
circle of radius 1m and makes 2 rev/sec. The
(A) decrease (B) increase
moment of inertia of the stone about the axis
(C) remains same (D) nothing can be said of rotation is
Sol. (A) 64 kg × m2 (B) 4 kg × m2
(C) 16 kg × m 2 (D) 1 kg × m2
Sol.

Corporate Head Office : Motion Education Pvt. Ltd., 394 - Rajeev Gandhi Nagar, Kota-5
ROTATIONAL DYNAMICS Page # 141

7. Three rings, each of mass P and radius Q Sol.


are arranged as shown in the figure. The
moment of inertia of the arrangement about
YY’ axis will be

10. Let IA and IB be moments of inertia of a body


about two axes A and B respectively. The axis A
passes through the centre of mass of the body
but B does not.
7 2 (A) IA < IB
(A) PQ2 (B) PQ2
2 7 (B) If IA < IB, the axes are parallel.
(C) If the axes are parallel, IA < IB
2 5
(C) PQ2 (D) PQ2 (D) If the axes are not parallel, IA  IB
5 2 Sol.
Sol.

8. A circular disc A of radius r is made from 11. For the same total mass which of the following
an iron plate of thickness t and another circular will have the largest moment of inertia about an
disc B of radius 4r is made from an iron plate axis passing through its centre of mass and
of thickness t/4. The relation between the perpendicular to the plane of the body
moments of inertia IA and IB is (A) a disc of radius a
(A) IA > IB (B) a ring of radius a
(B) IA = IB (C) a square lamina of side 2a
(D) four rods forming a square of side 2a
(C) IA < IB
Sol.
(D) depends on the actual values of t and r.
Sol.

12. Moment of inertia of a thin semicircular disc


(mass = M & radius = R) about an axis through
point O and perpendicular to plane of disc, is
given by :
O

9. The moment of inertia of a uniform semicircular R


wire of mass M and radius r about a line
perpendicular to the plane of the wire through
the centre is 1 1
(A) MR 2 (B) MR2
4 2
1 2 1 2 2 2
(A) Mr2 (B) Mr (C) Mr (D) Mr 1 2
2 4 5 (C) MR (D) MR2
8

: 0744-2209671, 08003899588 | url : www.motioniitjee.com, : info@motioniitjee.com


Page # 142 ROTATIONAL DYNAMICS

Sol. Sol.

13. A rigid body can be hinged about any point


on the x-axis. When it is hinged such that the
hinge is at x, the moment of inertia is given by 16. One end of a uniform rod of mass m and
I = 2x2 – 12x + 27 The x-coordinate of centre of length I is clamped. The rod lies on a smooth
mass is horizontal surface and rotates on it about the
(A) x = 2 (B) x = 0 clamped end at a uniform angular velocity . The
(C) x = 1 (D) x = 3 force exerted by the clamp on the rod has a
Sol. horizontal component
(A) m2 l (B) zero
1 2
(C) mg (D) m 
2
Sol.

14. Consider the following statements


Assertion (A) : The moment of inertia of a rigid
body reduces to its minimum value as compared
to any other parallel axis when the axis of rotation
passes through its centre of mass.
Reason (R) : The weight of a rigid body always
acts through its centre of mass in uniform 17. A rod of length 'L' is hinged from one end. It
gravitational field. Of these statements : is brought to a horizontal position and released.
(A) both A and R are true and R is the correct The angular velocity of the rod when it is in vertical
explanation of A position is
(B) both A and R are true but R is not a correct
explanation of A 2g 3g g g
(A) (B) (C) (D)
(C) A is true but R is false L L 2L L
(D) A is false but R is true Sol.
Sol.

(B) TORQUE AND PURE


15. A body is rotating uniformly about a vertical
ROTATIONAL MOTION
axis fixed in an inertial frame. The resultant force
on a particle of the body not on the axis is 18. A disc of radius 2m and mass 200kg is
(A) vertical acted upon by a torque 100N-m. Its angular
(B) horizontal and skew with the axis acceleration would be
(C) horizontal and intersecting the axis (A) 1 rad/sec2 (B) 0.25 rad/sec2
(D) none of these (C) 0.5 rad/sec2 . (D) 2 rad/sec2 .

Corporate Head Office : Motion Education Pvt. Ltd., 394 - Rajeev Gandhi Nagar, Kota-5
ROTATIONAL DYNAMICS Page # 143

Sol. 22. The moment of inertia and rotational kinetic


energy of a fly wheel are 20kg-m2 and 1000
joule respectively. Its angular frequency per
minute would be -

600 25 5 300
(A) (B) 2 (C) (D)
   
Sol.

19. On applying a constant torque on a body-


(A) linear velocity may be increases
(B) angular velocity may be increases
(C) it will rotate with constant angular velocity
(D) it will move with constant velocity
Sol.

23. The ang ul ar vel oci ty of a body i s

2 i 3 j

 = + + 4 k and a t orque

 = i + 2 j + 3 k acts on it. The rotational


power will be
(A) 20 watt (B) 15 watt
20. A wheel starting with angular velocity of (C) 17 watt (D) 14 watt
10 radian/sec acquires angular velocity of 100 Sol.
radian/sec in 15 seconds. If moment of inertia
is 10kg-m2 , then applied torque (in newton-
metre) is
(A) 900 (B) 100 (C) 90 (D) 60
Sol.

24. A torque of 2 newton-m produces an


angular acceleration of 2 rad/sec2 a body. If
21. An automobile engine develops 100H.P. its radius of gyration is 2m, its mass will be:
when rotating at a speed of 1800 rad/min. The (A) 2kg (B) 4 kg (C) 1/2 kg (D) 1/4 kg
torque it delivers is Sol.
(A) 3.33 W-s (B) 200W-s
(C) 248.7 W-s (D) 2487 W-s
Sol.

: 0744-2209671, 08003899588 | url : www.motioniitjee.com, : info@motioniitjee.com


Page # 144 ROTATIONAL DYNAMICS

25. A particle is at a distance r from the axis 28. In an experiment with a beam balance on
of rotation. A given torque  produces some unknown mass m is balanced by two known mass
angular acceleration in it. If the mass of the m is balanced by two known masses of 16 kg and
particle is doubled and its distance from the 4 kg as shown in figure.
axis is halved, the value of torque to produce l1 l2 l1 l2
the same angular acceleration is
(A) /2 (B)  (C) 2 (D) 4 m m
16kg 4kg
Sol.
The value of the unknown mass m is
(A) 10 kg (B) 6 kg (C) 8 kg (D) 12 kg
Sol.

26. A weightless rod is acted on by upward parallel


forces of 2N and 4N ends A and B respectively.
The total length of the rod AB = 3m. To keep the
rod in equilibrium a force of 6N should act in the
following manner :
(A) Downwards at any point between A and B.
(B) Downwards at mid point of AB. 29. A homogeneous cubical brick lies motionless
(C) Downwards at a point C such that AC = 1m. on a rough inclined surface. The half of the brick
(D) Downwards at a point D such that BD = 1m. which applies greater pressure on the plane is :
Sol.

(A) left half


(B) right half
(C) both applies equal pressure
(D) the answer depend upon coefficient of friction
27. A right triangular plate ABC of mass m is free Sol.
to rotate in the vertical plane about a fixed
horizontal axis through A. It is supported by a
string such that the side AB is horizontal. The
reaction at the support A is :

A l
B
l

C
mg 2 mg mg
(A) (B) (C) (D) mg
3 3 2 30. Consider the following statements
Sol. Assertion (A) : A cyclist always bends inwards
while negotiating a curve
Reason (R) : By bending he lowers his centre of
gravity Of these statements,
(A) both A and R are true and R is the correct
explanation of A
(B) both A and R are true but R is not the correct
explanation of A
(C) A is true but R is false
(D) A is false but R is true

Corporate Head Office : Motion Education Pvt. Ltd., 394 - Rajeev Gandhi Nagar, Kota-5
ROTATIONAL DYNAMICS Page # 145

Sol. Sol.

31. A rod is hinged at its centre and rotated by 33. The angular momentum of a flywheel having
applying a constant torque starting from rest. a moment of inertia of 0.4 kg m2 decreases from
The power developed by the external torque as a 30 to 20 kg m2/s in a period of 2 second. The
function of time is : average torque acting on the flywheel during this
Pext Pext period is :
(A) 10 N.m (B) 2.5 N.m
(A) (B) (C) 5 N.m (D) 1.5 N.m
Sol.
time time

Pext Pext

(C) (D)
time time

Sol.

34. A particle starts from the point (0m, 8m) and

32. A pulley is hinged at the centre and a massless moves with uniform velocity of 3 i m/s. After 5
thread is wrapped around it. The thread is pulled seconds, the angular velocity of the particle about
with a constant force F starting from rest. As the the origin will be :
time increases, y
F
3m/s

8m
(A) its angular velocity increases, but force on
hinge remains constant x
O
(B) its angular velocity remains same, but force
on hinge increases 8 3
(A) rad / s (B) rad / s
(C) its angular velocity increases and force on 289 8
hinge increases
(D) its angular velocity remains same and force 24 8
(C) rad / s (D) rad / s
on hinge is constant. 289 17

: 0744-2209671, 08003899588 | url : www.motioniitjee.com, : info@motioniitjee.com


Page # 146 ROTATIONAL DYNAMICS

Sol. Sol.

38. A dog of mass m is walking on a pivoted


disc of radius R and mass M in a circle of
radius R/2 with an angular frequency n: the
(C) ANGULAR MOMENTUM disc will revolve in opposite direction with
35. The rate of change of angular momentum frequency -
is called R/2
(A) angular velocity (B)angular acceleration R
(C) force (D) torque
Sol.

mn mn 2mn 2Mn
(A) (B) (C) (D)
M 2M M M
Sol.

36. The rotational kinetic energy of a rigid


body of moment of inertia 5 kg-m2 is 10 joules.
The angular momentum about the axis of
rotation would be -
(A) 100 joule-sec (B) 50 joule-sec 39. A particle moves with a constant velocity
(C) 10 joule-sec (D) 2 joule -sec parallel to the X-axis. Its angular momentum with
Sol. respect to the origin.
(A) is zero
(B) remains constant
(C) goes on increasing
(D) goes on decreasing.
Sol.

37. The angular velocity of a body changes


from one revolution per 9second to 1 revolution
per second without applying any torque. The ratio
of its radius of gyration in the two cases is
(A) 1 : 9 (B) 3 : 1
(C) 9 : 1 (D) 1 : 3

Corporate Head Office : Motion Education Pvt. Ltd., 394 - Rajeev Gandhi Nagar, Kota-5
ROTATIONAL DYNAMICS Page # 147

40. A person sitting firmly over a rotating stool 43. A ball of mass m moving with velocity v, collide
has his arms streatched. If he folds his arms, his with the wall elastically as shown in the figure.
angular momentum about the axis of rotation After impact the change in angular momentum
(A) increases (B) decreases about P is :
P
(C) remains unchanged (D) doubles.
Sol. d

(A) 2 mvd (B) 2 mvd cos


(C) 2 mvd sin (D) zero
Sol.

41. A man, sitting firmly over a rotating stool has


his arms streched. If he folds his arms, the work
done by the man is
(A) zero
(B) positive
(C) negative
(D) may be positive or negative.
44. A uniform rod of mass M has an impulse applied
Sol. at right angles to one end. If the other end begins
to move with speed V, the magnitude of the
impulse is
MV 2MV
(A) MV (B) (C) 2MV (D)
2 3
Sol.

42. A particle of mass 2 kg located at the position


( i  j ) m has a veloci ty 2(  i – j  k ) m/s. Its
angular momentum about z-axis in kg-m2 /s is :
(A) zero (B) +8
(C) 12 (D) – 8
Sol.

(D) COMBINED TRANSLATIONAL


+ ROTATIONAL MOTION
45. A circular ring of wire of mass M and
radius R is making n revolutions/sec about an
axis passing through a point on its rim and
perpendicular to its plane. The kinetic energy
of rotation of the ring is given by-
(A) 42MR 2n2 (B) 22MR2n2

1 2
(C)  MR 2n2 (D) 82MR2n2
2

: 0744-2209671, 08003899588 | url : www.motioniitjee.com, : info@motioniitjee.com


Page # 148 ROTATIONAL DYNAMICS

Sol. 48. A disc rolls on a table. The ratio of its K.E.


of rotation to the total K.E. is -
(A) 2/5 (B) 1/3 (C) 5/6 (D) 2/3
Sol.

46. Rotational kinetic energy of a disc of


constant moment of inertia is -
(A) directly proportional to angular velocity
(B) inversely proportional to angular velocity 49. A disk and a ring of the same mass are
(C) inversely proportional to square of angular rolling to have the same kinetic energy. What
velocity is ratio of their velocities of centre of mass
(D) directly proportional to square of angular (A) (4:3)1/2 (B) (3 : 4)1/2
velocity (C) (2)1/2 : (3) 1/2 (D) (3)1/2 : (2)1/2
Sol. Sol.

50. If the applied torque is directly proportional


to the angular displacement , then the work
47. A circular disc has a mass of 1kg and done in rotating the body through an angle 
radius 40 cm. It is rotating about an axis would be - (C is constant of proportionality)
passing through its centre and perpendicular
1 1
to its plane with a speed of 10rev/s. The work (A) C (B) C (C) C2 (D) C2
done in joules in stopping it would be- 2 2
(A) 4 (B) 47.5 (C) 79 (D) 158 Sol.
Sol.

Corporate Head Office : Motion Education Pvt. Ltd., 394 - Rajeev Gandhi Nagar, Kota-5
ROTATIONAL DYNAMICS Page # 149

51. The centre of a wheel rolling without slipping Sol.


in a plane surface moves with speed v0. A particle
on the rim of the wheel at the same level as the
centre will be moving at speed.
(A) zero (B) v0 (C) 2v0 (D) 2v0
Sol.

54. There is rod of length l. The velocities of its


two ends are v1 and v2 in opposite directions
normal to t he rod . The di st ance of the
instantaneous axis of rotation from v1 is :
v2 v1l
(A) zero (B) v  v l (C) v  v (D) l/2
1 2 1 2
Sol.

52. A solid sphere, a hollow sphere and a disc, all


having smooth incline and released. Least time
will be taken in reaching the bottom by
(A) the solid sphere (B) the hollow sphere
(C) the disc (D) all will take same time.
Sol.

55. A ring of radius R rolls without sliding with a


constant velocity. The radius of curvature of the
path followed by any particle of the ring at the
highest point of its path will be
(A) R (B) 2R (C) 4R (D) none
Sol.

53. A wheel of radius r rolling on a straight line,


the velocity of its centre being v. At a certain
instant the point of contact of the wheel with
the grounds is M and N is the highest point on
the wheel (diametrically opposite to M). The
incorrect statement is :
(A) The velocity of any point P of the wheel is
proportional to MP. 56. The linear speed of a uniform spherical shell
(B) Points of the wheel moving with velocity after rolling down an inclined plane of vertical height
greater than v form a larger area of the wheel h from rest, is :
than points moving with velocity less than v. 10gh 4gh
(C) The point of contact M is instantaneously at (A) (B)
7 5
rest.
(D) The velocities of any two parts of the wheel 6gh
which are equidistant from centre are equal. (C) (D) 2gh
5

: 0744-2209671, 08003899588 | url : www.motioniitjee.com, : info@motioniitjee.com


Page # 150 ROTATIONAL DYNAMICS

Sol. Sol.

57. A body kept on a smooth horizontal surface


is pulled by a constant horizontal force applied at 60. A uniform circular disc placed on a rough
the top point of the body. If the body rolls purely horizontal surface has initially a velocity v0 and
on the surface, its shape can be : an angular velocity 0 as shown in the figure.
(A) thin pipe (B) uniform cylinder The disc comes to rest after moving some distance
(C) uniform sphere (D) thin spherical shell v0
Sol. in the direction of motion. Then r is
0
0

v0

1 3
(A) (B) 1 (C) (D) 2
2 2
Sol.

58. A solid sphere with a velocity (of centre of


mass) v and angular velocity  is gently placed
on a rough horizontal surface. The frictional force
on the sphere :
(A) must be forward (in direction of v)
(B) must be backward (opposite to v)
(C) cannot be zero
(D) none of the above
Sol.
61. A Cubical bloc of mass M and edge a slides
down a rough inclined plane of inclination  with a
uniform velocity. The torque of the normal force
on the block about its centre has a magnitude.
(A) zero (B) Mga
1
(C) Mga sin  (D) Mgasin
2
Sol.

59. A cylinder is pure rolling up an incline plane.


It stops momentarily and then rolls back. The
force of friction.
(A) on the cylinder is zero throughout the journey
(B) is directed opposite to the velocity of the
centre of mass throughout the journey
(C) is directed up the plane throughout the journey
(D) is directed down the plane throughout the
journey

Corporate Head Office : Motion Education Pvt. Ltd., 394 - Rajeev Gandhi Nagar, Kota-5
ROTATIONAL DYNAMICS Page # 151

Exercise - II
(A) MOMENT OF INERTIA Sol.
1. Three bodies have equal masses m. Body A is
solid cylinder of radius R, body B is a square lamina
of side R, and body C is a solid sphere of radius R.
Which body has the smallest moment of inertia
about an axis passing through their centre of mass
and perpendicular to the plane (in case of lamina)
(A) A (B) B
(C) C (D) A and C both
Sol.
4. A thin uniform rod of mass M and length L has
its moment of inertia I1 about its perpendicular
bisector. The rod is bend in the form of a
semicircular arc. Now its moment of inertia
through the centre of the semi circular arc and
perpendicular to its plane is I2. The ratio of I1 : I2
will be _________________
(A) < 1 (B) > 1 (C) = 1 (D) can’t be said
Sol.
2. Two rods of equal mass m and length l lie along
the x axis and y axis with their centres origin.
What is the moment of inertia of both about the
line x = y :
ml 2 ml 2
(A) (B)
3 4
ml 2 ml 2
(C) (D)
12 6
Sol. 5. A square plate of mass M and edge L is shown in
figure. The moment of inertia of the plate about
the axis in the plane of plate passing through one
of its vertex making an angle 15° from horizontal is.
axis
15°

L
ML2 11ML2 7 ML2
(A) (B) (C) (D) none
12 24 12
3. Moment of inertia of a rectangular plate about Sol.
an axis passing through P and perpendicular to
the plate is I. Then moment of PQR about an axis
perpendicular to the plane of the plate :
P Q

S R
(A) about P = I/2 (B) about R = I/2
(C) about P > I/2 (D) about R > I/2

: 0744-2209671, 08003899588 | url : www.motioniitjee.com, : info@motioniitjee.com


Page # 152 ROTATIONAL DYNAMICS

Question No. 6 to 9 (4 questions) Sol.


The figure shows an isosceles triangular plate of
mass M and base L. The angle at the apex is 90°.
The apex lies at the origin and the base is parallel
to X - axis.
Y

M

X
6. The moment of inertia of the plate about the
z-axis is
ML2 ML2 ML2 9. The moment of inertia of the plate about the
(A) (B) (C) (D) none of these
12 24 6 y-axis is
Sol.
ML2 ML2
(A) (B)
6 8
ML2
(C) (D) none of these
24
Sol.

7. The moment of inertia of the plate about the 10. ABCD is a square plate with centre O. The
x-axis is moments of inertia of the plate about the per-
pendicular axis through O is I and about the axes
ML2 ML2 ML2 ML2
(A) (B) (C) (D) 1, 2, 3 & 4 are I1, I2, I3 & I4 respectively. It follows
8 32 24 6 that :
Sol. 1
2
A B

3
O

D C
4
(A) I2 = I3 (B) I = I1 + I4
(C) I = I2 + I4 (D) I1 = I3
Sol.

8. The moment of inertia of the plate about its


base parallel to the x-axis is
ML2 ML2 ML2
(A) (B) (C) (D) none of these
18 36 24

Corporate Head Office : Motion Education Pvt. Ltd., 394 - Rajeev Gandhi Nagar, Kota-5
ROTATIONAL DYNAMICS Page # 153

(B) TORQUE & PURE 13. A uniform cube of side ‘b’ and mass M rest on
ROTATIONAL MOTION a rough horizontal table. A horizontal force F is
applied normal to one of the face at a point, at a
height 3b/4 above the base. What should be the
11. A horizontal force F = mg/3 is applied on the
coefficient of friction () between cube and table
upper surface of a uniform cube of mass ‘m’ and
so that is will tip about an edge before it starts
side ‘a’ which is resting on a rough horizontal
slipping?
surface having s = 1/2. The distance between
lines of action of ‘mg’ and normal reaction ‘N’ is :
(A) a/2 (B) a/3 (C) a/4 (D) None F
Sol. b
3b/4

2 1 3
(A)   (B)   (C)   (D) none
3 3 2
Sol.

12. A man can move on a horizontal plank


supported symmetrically as shown. The variation
of normal reaction on support A with distance x
of the man from the end of the plank is best
represented by : 14. A solid cone hangs from a frictionless pivot
x=0 at the origin O, as shown. If i , j and k are unit
A B vectors, and a, b, and c are positive constants,
which of the following forces F applied to the rim
1m 4m 1m of the cone at a point P results in a torque  on
the cone with a negative component Z ?
N N z

(A) (B) k o
i y
j
x x
x c
N N

(C) (D) b
x x (A) F = a k , P is (0, b, –c)
Sol.
(B) F = –a k , P is (0, –b, –c)
(C) F = a j , P is (–b, 0, –c)
(D) None
Sol.

: 0744-2209671, 08003899588 | url : www.motioniitjee.com, : info@motioniitjee.com


Page # 154 ROTATIONAL DYNAMICS

15. A block of mass m is attached to a pulley 17. A block with a square base measuring axa
disc of equal mass m, radius r by means of a and height h, is placed on an inclined plane. The
slack string as shown. The pulley is hinged about coefficient of friction is . The angle of inclina-
its centre on a horizontal table and the block is tion () of the plane is gradually increased. The
projected with an initial velocity of 5 m/s. Its block will
velocity when the string becomes taut will be a
(A) topple before sliding if  
h
a
(B) topple before sliding if  
h
a
(C) slide before toppling if  
(A) 3 m/s (B) 2.5 m/s h
(C) 5/3 m/s (D) 10/3 m/s a
(D) slide before toppling if  
Sol. h
Sol.

16. A rod of weight w is supported by two paral- 18. A body is in equilibrium under the influence of
lel knife edges A and B and is in equilibrium in a a number of forces. Each force has a different
horizontal position. The knives are at a distance line of action. The minimum number of forces re-
d from each other. The centre of mass of the rod quired is
is at a distance x from A. (A) 2, if their lines of action pass through the
wx centre of mass of the body
(A) the normal reaction at A is (B) 3, if their lines of action are not parallel
d
w(d  x) (C) 3, if their lines of action are parallel
(B) the normal reaction at A is (D) 4, if their lines of action are parallel and all
d
wx the forces have the same magnitude
(C) the normal reaction at B is Sol.
d
w(d  x)
(D) the normal reaction at B is
d
Sol.

Corporate Head Office : Motion Education Pvt. Ltd., 394 - Rajeev Gandhi Nagar, Kota-5
ROTATIONAL DYNAMICS Page # 155

19. A block of mass m moves on a horizontal Sol.


rough surface with initial velocity v. The height
of the centre of mass of the block is h from the
surface. Consider a point A on the surface
(A) angular momentum about A is mvh initially
(B) the velocity of the block decreases at time
passes
(C) torque of the forces acting on block is zero
about A
(D) angular momentum is not conserved about A
Sol.

21. A particle falls freely near the surface of the


earth. Consider a fixed point O (not vertically
below the particle) on the ground.
(A) Angular momentum of the particle about O is
increasing
(B) Torque of the gravitational force on the par-
ticle about O is decreasing
(C) The moment of inertia of the particle about O
is decreasing
(D) The angular velocity of the particle about O
is increasing
Sol.

20. Four point masses are fastened to the cor-


ners of a frame of negligible mass lying in the xy
plane. Let w be the angular speed of rotation.
Then
y-axis
m
b

M M x-axis
m a
z-axis
(A) rotational kinetic energy associated with a
given angular speed depends on the axis of rota-
tion.
(B) rotational kinetic energy about y-axis is inde-
pendent of m and its value is Ma22
(C) rotational kinetic energy about z-axis depends
on m and its value is (Ma2 + mb2)2
(D) rotational kinetic energy about z-axis is inde-
pendent of m and its value is Mb22

: 0744-2209671, 08003899588 | url : www.motioniitjee.com, : info@motioniitjee.com


Page # 156 ROTATIONAL DYNAMICS

22. A rod hinged at one end is released from the Sol.


horizontal position as shown in the figure. When
it becomes vertical its lower half separates without
exerting any reaction at the breaking point. Then
the maximum angle ‘’ made by the hinged upper
half with the vertical is :
C B A

B B
C
(A) 30° (B) 45° (C) 60° (D) 90°
Sol.

25. A thin circular ring of mass 'M' and radius 'R'


is rotating about its axis with a constant angular
velocity . Two objects each of mass m, are
attached gently to the opposite ends of a diameter
of the ring. The ring now rotates with an angular
velcoity.

M M
(A) (B)
(M  m) (M  2m)

M (M  3m)
(C) (D)
(C) ANGULAR MOMENTUM (M – 2m) M
23. If a person sitting on a rotating stool with his Sol.
hands outstretched, suddenly lowers his hands,
then his
(A) Kinetic energy will decrease
(B) Moment of inertia will decrease
(C) Angular momentum will increase
(D) Angular velocity will remain constant
Sol.

26. A small bead of mass m moving with velocity


v gets threaded on a stationary semicircular ring
of mass m and radius R kept on a horizontal table.
The ring can freely rotate about its centre. The
bead comes to rest relative to the ring. What will
be the final angular velocity of the system?
24. A man spinning in free space changes the
shape of his body, eg. by spreading his arms or
curling up. By doing this, he can change his
(A) moment of inertia R
(B) angular momentum O
(C) angular velocity v
m
(D) rotational kinetic energy (A) v/R (B) 2v/R (C) v/2R (D) 3v/R

Corporate Head Office : Motion Education Pvt. Ltd., 394 - Rajeev Gandhi Nagar, Kota-5
ROTATIONAL DYNAMICS Page # 157

Sol. Sol.

Question No. 29& 30 (2 questions)


A uniform rod is fixed to a rotating turntable so
that its lower end is on the axis of the turntable
and it makes an angle of 20° to the vertical.
27. A thin uniform straight rod of mass 2 kg and (The rod is thus rotating with uniform angular
length 1 m is free to rotate about its upper end velocity about a vertical axis passing through one
when at rest. It receives an impulsive blow of 10 end.) If the turntable is rotating clockwise as
Ns at its lowest point, normal to its length as seen from above.
shown in figure. The kinetic energy of rod just after
impact is 20°

10 NS

(A) 75 J (B) 100 J


(C) 200 J (D) none 29. What is the direction of the rod’s angular
Sol. momentum vector (calculated about its lower end)
(A) vertically downwards
(B) down at 20° to the horizontal
(C) up at 20° to the horizontal
(D) vertically upwards
Sol.

28. A child with mass m is standing at the edge


of a disc with moment of inertia I, radius R, and
initial angular velocity . See figure given below. 30. Is there a torque acting on it, and if so in
The child jumps off the edge of the disc with what direction?
tangential velocity v with respect to the ground. (A) yes, vertically (B) yes, horizontally
The new angular velocity of the disc is (C) yes at 20° to the horizontal
(D) no
v
Sol.

I 2 – mv 2 (I + mR 2 ) 2 – mv 2
(A) (B)
I I

I – mvR ( I + mR 2 )  mvR
(C) (D)
I I

: 0744-2209671, 08003899588 | url : www.motioniitjee.com, : info@motioniitjee.com


Page # 158 ROTATIONAL DYNAMICS

31. One ice skater of mass m moves with speed Sol.


2v to the right, while another of the same mass
m moves with speed v toward the left, as shown
in figure I. Their paths are separated by a distance
b. At t = 0, when they are both at x = 0, they
grasp a pole of length b and negligible mass. For
t > 0, consider the system as a rigid body of two
masses m separated by distance b, as shown in
figure II. Which of the following is the correct
formula for the motion after t = 0 of the skater
initially at y = b/2 ?
y y
m 2v

b/2 33. A uniform rod AB of mass m and length l is at


b x rest on a smooth horizontal surface. An impulse J
x is applied to the end B, perpendicular to the rod
t=0
in the horizontal direction. Speed of particle P at
m l
v a distance from the centre towards A of the
(t<0) 6
Figure 1 Figure II ml
rod after time t  is
(A) x = 2vt, y = b/2 12J
(B) x = vt + 0.5b sin (3vt/b), y = 0.5b cos(3vt/b) J J J J
(C) x = 0.5vt + 0.5b sin (3vt/b), y = 0.5b cos(3vt/b) (A) 2 (B) (C) (D) 2
m 2 m m m
(D) x = 0.5vt + 0.5b sin (6vt/b), y = 0.5b cos(6vt/b) Sol.
Sol.

32. A uniform rod AB of length L and mass M is


lying on a smooth table. A small particle of mass
m strike the rod with a velocity v0 at point C at a 34. A uniform rod of mass M is hinged at its upper
distance x from the centre O. The particle comes end. A particle of mass m moving horizontally
to rest after collision. The value of x, so that strikes the rod at its mid point elastically. If the
point A of the rod remains ststionary just after particle comes to rest after collision find the value
collision is : of M/m = ?
B
m v0
C v
x m
O
M
(A) 3/4 (B) 4/3
A
(A) L/3 (B) L/6 (C) L/4 (D) L/12 (C) 2/3 (D) none

Corporate Head Office : Motion Education Pvt. Ltd., 394 - Rajeev Gandhi Nagar, Kota-5
ROTATIONAL DYNAMICS Page # 159

Sol. (D) COMBINED TRANSLATIONAL


+ ROTATIONAL MOTION
37. A ring rolls without slipping on the ground.
Its centre C moves with a constant speed u. P is
any point on the ring. The speed of P with re-
spect to the ground is v.
(A) 0  v  2u
(B) v = u, if CP is horizontal
(C) v = u, if CP makes an angle of 30º with the
horizontal and P is below the horizontal level of C

35. Two equal masses each of mass M are joined (D) v  2 u , if CP is horizontal
by a massless rod of length L. Now an impulse MV Sol.
is given to the mass M making an angle of 30º
with the length of the rod. The angular velocity
of the rod just after imparting the impulse is
M M

30°

MV
v 2v
(A) (B)
L L
v
(C) (D) none of these
2L
Sol.

38. A yo-yo is resting on a perfectly rough hori-


zontal table. Forces F1, F2 and F3 are applied
36. Two particles of equal mass m at A and B are
connected by a rigid light rod AB lying on a smooth separately as shown.The correct
F2 statement is
F3
horizontal table. An impulse J is applied at A in the
plane of the table and perpendicular at AB. Then
the velocity of particle at A is :
J J 2J
(A) (B) (C) (D) zero
2m m m
Sol. F1

(A) when F3 is applied the centre of mass will


move to the right
(B) when F2 is applied the centre of mass will
move to the left
(C) when F1 is applied the centre of mass will
move to the right
(D) when F2 is applied the centre of mass will
move to the right

: 0744-2209671, 08003899588 | url : www.motioniitjee.com, : info@motioniitjee.com


Page # 160 ROTATIONAL DYNAMICS

Sol. 40. A plank with a uniform sphere placed on it,


rests on a smooth horizontal plane. Plank is pulled
to right by a constant force F. If the sphere does
not slip over the plank.

(A) acceleration of centre of sphere is less than


that of the plank
(B) acceleration of centre of sphere is greater
than the plank because friction acts rightward
on the sphere
(C) acceleration of the centre of sphere may be
towards left
(D) acceleration of the centre of sphere relative
to plank may be greater than that of the plank
relative to floor
39. A disc of circumference s is at rest at a point Sol.
A on a horizontal surface when a constant hori-
zontal force begins to act on its centre. Between
A and B there is sufficient friction to prevent
slipping, and the surface is smooth to the right of
B. AB = s. The disc moves from A to B in time T.
To the right of B,

Force

A B
(A) the angular acceleration of the disc will dis-
appear, linear acceleration will remain unchanged 41. A hollow sphere of radius R and mass m is
fully filled with water of mass m. It is rolled down
(B) linear acceleration of the disc will increase
a horizontal plane such that its centre of mass
(C) the disc will make one rotation in time T/2
moves with a velocity v. If it purely rolls
(D) the disc will cover a distance greater than s
5 2
in further time T. (A) Kinetic energy of the sphere is mv
Sol. 6
4 2
(B) Kinetic energy of the sphere is mv
5
(C) Angular momentum of the sphere about a
8
fixed point on ground is mvR
3
(D) Angular momentum of the sphere about a
14
fixed point on ground is mvR
5
Sol.

Corporate Head Office : Motion Education Pvt. Ltd., 394 - Rajeev Gandhi Nagar, Kota-5
ROTATIONAL DYNAMICS Page # 161

42. In the figure shown, the plank is being pulled 44. Which of the following statements are cor-
to the right with a constant speed v. If the cylin- rect
der does not slip then : (A) friction acting on a cylinder without sliding on
an inclined surface is always upward along the
incline irrespective of any external force acting
R on it.
(B) friction acting on a cylinder without sliding on
v an inclined surface is may be upward may be
downwards depending on the external force act-
ing on it.
(A) the speed of the centre of mass of the cylin- (C) friction acting on a cylinder rolling without
der is 2v sliding may be zero depending on the external
(B) the speed of the centre of mass of the cylin-
force acting on it.
der is zero
(D) nothing can be said exactly about it as it
(C) the angular velocity of the cylinder is v/R
depends on the friction coefficient on inclined
(D) the angular velocity of the cylinder is zero plane
Sol. Sol.

Question No. 45 to 47 (3 Questions)


A cylinder and a ring of same mass M and radius
43. If a cylinder is rolling down the incline with
,
R are placed on the top of a rough inclined plane
sliding
of inclination . Both are released simultaneously
(A) after some time it may start pure rolling
from the same height h.
(B) after sometime it will start pure rolling
(C) it may be possible that it will never start pure 45. Choose the correct statement(s) related to
rolling the motion of each body
(D) none of these (A) The friction force acting on each body
Sol. opposes the motion of its centre of mass
(B) The friction force provides the necessary torque
to rotate the body about its centre of mass
(C) without friction none of the two bodies can roll
(D) The friction force ensures that the point of
contact must remain stationary
Sol.

: 0744-2209671, 08003899588 | url : www.motioniitjee.com, : info@motioniitjee.com


Page # 162 ROTATIONAL DYNAMICS

46. Identify the correct statement(s) Question No. 48 to 51 (4 Questions)


(A) The friction force acting on the cylinder may A ring of mass M and radius R sliding with a ve-
be more than that acting on the ring locity v0 suddenly enters into rough surface where
(B) The friction force acting on the ring may be the coefficient of friction is , as shown in figure.
more than that acting on the cylinder
(C) If the friction is sufficient to roll the cylinder v0
then the ring will also roll
(D) If the friction is sufficient to roll the ring then Rough ( )
the cylinder will also roll 48. Choose the correct statement(s)
Sol. (A) As the ring enters on the rough surface, the
Kinetic friction force acts on it
(B) The direction of friction is opposite to the
direction of motion
(C) The friction force accelerates the ring in the
clockwise sense about its centre of mass
(D) As the ring enters on the rough surface it
starts rolling
Sol.

47. When these bodies roll down to the foot of


the inclined plane, then
(A) the mechanical energy of each body is con-
served
(B) the velocity of centre of mass of the cylinder
gh 49. Choose the correct statement(s)
is 2
3 (A) The momentum of the ring is conserved
(C) the velocity of centre of mass of the ring is (B) The angular momentum of the ring is con-
gh served about its centre of mass
(D) the velocity of centre of mass of each body (C) The angular momentum of the ring conserved
about any point on the horizontal surface
is 2 gh (D) The mechanical energy of the ring is con-
Sol. served
Sol.

Corporate Head Office : Motion Education Pvt. Ltd., 394 - Rajeev Gandhi Nagar, Kota-5
ROTATIONAL DYNAMICS Page # 163

50. Choose the correct statement(s) 52. Consider a sphere of mass ‘m’ radius ‘R’ doing
(A) The ring starts its rolling motion when the pure rolling motion on a rough surface having
centre of mass stationary 
velocity v 0 as shown in the Figure. It makes an
(B) The ring starts rolling motion when the point
of contact becomes stationary elastic impact with the smooth wall and moves
back and starts pure rolling after some time again.
(C) The time after which the ring starts rolling is
v0
2 g
v v0
(D) The rolling velocity is 0
2
Sol. O
(A) Change in angular momentum about ‘O’ in the
entire motion equals 2mv0 R in magnitude.
(B) Moment of impulse provided by wall during
impact about O equals 2mv0R in magnitude
3
(C) Final velocity of ball will be v 0
7
3
(D) Final velocity of ball will be  v 0
7
Sol.

51. Choose the correct alternative(s)


(A) The linear distance moved by the centre of
3 v 20
mass before the ring starts rolling is
8 g
3 53. A solid sphere, a hollow sphere and a disc, all
(B) The net work done by friction force is  mv 20 having same mass and radius, are placed at the
8
top of an incline and released. The friction
mv 20 coefficients between the objects and the incline
(C) The loss is kinetic energy of the ring is
4 are same and not sufficient to allow pure rolling.
mv 20 The smallest kinetic energy at the bottom of the
(D) The gain in rotational kinetic energy is 
8 incline will be achieved by
Sol. (A) the solid sphere (B) the hollow sphere
(C) the disc
(D) all will achieve same kinetic energy.
Sol.

: 0744-2209671, 08003899588 | url : www.motioniitjee.com, : info@motioniitjee.com


Page # 164 ROTATIONAL DYNAMICS

54. Fig. shows a smooth inclined plane fixed in a 56. A ladder of length L is slipping with its ends
car accelerating on a horizontal road. The angle against a vertical wall and a horizontal floor. At a
of incline  is related to the acceleration a of the certain moment, the speed of the end in contact
car as a = g tan . If the sphere is set in pure with the horizontal floor is v and the ladder makes
rotation on the incline. an angle  = 30º with the horizontal. Then the
speed of the ladder’s center must be
(A) 2v / 3 (B) v/2
a
(C) v (D) none
 Sol.

(A) it will continue pure rolling


(B) it will slip down the plane
(C) its linear velocity will increase
(D) its linear velocity will decrease.
Sol.

57. In the previous question, if dv/dt = 0, then


the angular acceleration of the ladder when  =
45º is
(A) 2v2/L2 (B) v2/2L2
(C) 2 [ v 2 / L2 ] (D) None
Sol.
55. A straight rod of length L is released on a
frictionless horizontal floor in a vertical position.
As it falls + slips, the distance of a point on the
rod from the lower end, which follows a quarter
circular locus is
(A) L/2 (B) L/4
(C) L/8 (D) None
Sol.

58. A time varying force F = 2t is applied on a


spool rolling as shown in figure. The angular
momentum of the spool at time t about bottom
most point is :

F=2t
r

r 2t2 (R  r ) 2 2
(A) (B) t
R r
(C) (R + r)t2 (D) data is insufficient

Corporate Head Office : Motion Education Pvt. Ltd., 394 - Rajeev Gandhi Nagar, Kota-5
ROTATIONAL DYNAMICS Page # 165

Sol. Sol.

59. A ball rolls down an inclined plane, figure.


The ball is first released from rest from P and
then later from Q. Which of the following
statement is/ are correct ?
Q

P
2h
61. In the figure shown a ring A is initially rolling
h
without sliding with a velocity v on the horizontal
O surface of the body B (of same mass as A). All
(i) The ball takes twice as much time to roll from surfaces are smooth. B has no initial velocity.
Q to O as it does to roll from P to O. What will be the maximum height reached by A
(ii) The acceleration of the ball at Q is twice as on B.
large as the acceleration at P.
(iii) The ball has twice as much K.E. at O when v
rolling from Q as it does when rolling from P. A
Smooth
(A) i, ii only (B) ii, iii only B
(C) i only (D) iii only
Sol.
3v 2 v2
(A) (B)
4g 4g
v2 v2
(C) (D)
2g 3g
Sol.

60. Starting from the rest, at the same time, a


ring, a coin and a solid ball of same mass roll
down an incline without slipping. The ratio of their
translational kinetic energies at the bottom will
be
(A) 1 : 1 : 1 (B) 10 : 5 : 4
(C) 21 : 28 : 30 (D) none

: 0744-2209671, 08003899588 | url : www.motioniitjee.com, : info@motioniitjee.com


Page # 166 ROTATIONAL DYNAMICS

62. Inner and outer radii of a spool are r and R Sol.


respectively. A thread is wound over its inner
surface and placed over a rough horizontal
surface. Thread is pulled by a force F as shown
in fig. then in case of pure rolling

(A) Thread unwinds, spool rotates anticlockwise


and friction act leftwards
(B) Thread winds, spool rotates clockwise and
friction acts leftwards
(C) Thread winds, spool moves to the right anf
friction act rightwards
(D) Thread winds, spool moves to the right and
friction does not come into existence.
Sol.

64. A plank of mass M is placed over smooth


inclined plane and a sphere is also placed over
the plank. Friction is sufficient between sphere
and plank. If plank and sphere are released from
rest, the frictional force on sphere is :


(A) up the plane (B) down the plane
(C) horizontal (D) zero
Sol.

63. Portion AB of the wedge shown in figure is


rough and BC is smooth. A solid cylinder rolls
without slipping from A to B. The ratio of
translational kinetic energy to rotational kinetic
energy, when the cylinder reaches point C is :

D AB=BC C

(A) 3/4 (B) 5 (C) 7/5 (D) 8/3

Corporate Head Office : Motion Education Pvt. Ltd., 394 - Rajeev Gandhi Nagar, Kota-5
ROTATIONAL DYNAMICS Page # 167

65. A plank with a uniform sphere placed on it Sol.


rests on a smooth horizontal plane. Plank is pulled
to right by a constant force F. If sphere does not
slip over the plank. Which of the following is
incorrect.

(A) Acceleration of the centre of sphere is less


than that of the plank
(B) Work done by friction acting on the sphere is
equal to its total kinetic energy.
(C) Total kinetic energy of the system is equal to
work done by the force F
(D) None of the above
Sol.

67. A uniform sphere of radius R is placed on a


rough horizontal surface and given a linear velocity
v0 angular velocity 0 as shown. The sphere comes
to rest after moving some distance to the right.
It follows that :

v0

0
(A) v0 = 0R (B) 2v0 = 50R
(C) 5v0 = 20R (D) 2v0 = 0R
Sol.

66. A ring of mass m and radius R has three


particles attached to the ring as shown in the
figure. The centre of the ring has speed v0. The
kinetic energy of the system is (Slipping is absent)
m
2m m

(A) 6mv02 (B) 12 mv02


(C) 4 mv02 (D) 8 mv02

: 0744-2209671, 08003899588 | url : www.motioniitjee.com, : info@motioniitjee.com


Page # 168 ROTATIONAL DYNAMICS

Exercise - III (JEE ADVANCED)

(A) MOMENT OF INERTIA 3. Find the radius of gyration of a circular ring of


1. Find the moment of inertia of a uniform half- radius r about a line perpendicular to the plane of
disc about an axis perpendicular to the plane and this ring and tangent to the ring.
passing through its centre of mass. Mass of this
disc is M and radius is R.
Sol.

Sol.

4. Moment of inertial of a triangle plane of mass


2. Find the moment of inertia of a pair of solid
M shown in figure about vertical axis AB is :
spheres, each having a mass m and radius r, kept
in contact about the tangent passing through
the point of contact. A
Sol. l
45°
l m

B
Sol.

Corporate Head Office : Motion Education Pvt. Ltd., 394 - Rajeev Gandhi Nagar, Kota-5
ROTATIONAL DYNAMICS Page # 169

 
5. A uniform rod of mass m is bent into the form 7. Two forces F1  2i – 5 j – 6k and F2  – i  2j – k
of a semicircle of radius R. The moment of inertia are acting on a body at the points (1, 1, 0) and
of the rod about an axis passing through A and (0, 1, 2). Find torque acting on the body about
perpendicular to the plane of the paper is point (–1, 0, 1).
A Sol.
R

Sol.

8. Assuming frictionless contacts, determine the


(B) TORQUE & PURE ROTATIONAL magnitude of external horizontal force P applied
MOTION at the lower end for equilibrium of the rod. The
6. A simple pendulum of length  is pulled aside to rod is uniform and its mass is 'm'.
made an angle  with the vertical. Find the
magnitude of the torque of the weight w of the Wall
bob about the point of suspension. When is the
torque zero ?
Sol.
P

Sol.

: 0744-2209671, 08003899588 | url : www.motioniitjee.com, : info@motioniitjee.com


Page # 170 ROTATIONAL DYNAMICS

9. A rod of mass m and length L, lying horizontally, 11. Figure shows two blocks of mass m and m
is free to rotate about a vertical axis through its connected by a string passing over a pulley. The
centre. A horizontal force of constant magnitude horizontal table over which the mass m slides is
F acts on the rod at a distance of L/4 from the smooth. The pulley (uniform disc) has mass m
centre. The force is always perpendicular to the and it can freely rotate about this axis. Find the
rod. Find the angle rotated by the rod during the acceleration of the mass m assuming that the
time t after the motion starts. string does not slip on the pulley.
Sol.
m
m

Sol.

10. The uniform rod AB of mass m is released


from rest when  = 60°. Assuming that the friction
force between end A and the surface is large
enough to prevent sliding, determine (for the
instant just after release) 12. A solid cylinder of mass M = 1kg & radius R =
0.5m is pivoted at its centre & has three particles
B of mass m = 0.1 kg mounted at its perimeter as
L shown in the figure. The system is originally at
rest. Find the angular speed of the cylinder, when
it has swung through 90° in anticlockwise
A 
direction.

(a) The angular acceleration of the rod


(b) The normal reaction and the friction force at A.
(c) The minimum value of , compatible with the
described motion.
Sol.
Sol.

Corporate Head Office : Motion Education Pvt. Ltd., 394 - Rajeev Gandhi Nagar, Kota-5
ROTATIONAL DYNAMICS Page # 171

13. A cube is in limiting equilibrium on an inclined 15. An inverted “V” is made up of two uniform
plane forming an angle of 30° with the horizontal. boards each weighing 200 N. Each side has the
The line of action of the normal reaction of the same length and makes an angle 30° with the
plane on the cube is vertical as shown in figure. The magnitude of the
Sol. static frictional force that acts on each of the
lower end of the V is
P

30°30°
l

Sol.

14. A body weighs 6 gms when placed in one pan


16. A uniform sphere of weight W and radius 5
and 24 gms when placed on the other pan of a
cm is being held by a string as shown in the
false balance. If the beam is horizontal when both
figure. The wall is smooth. The tension in the
the pans are ampty, the true weight of the body
string will be
is :
Sol.
8cm

Sol.

: 0744-2209671, 08003899588 | url : www.motioniitjee.com, : info@motioniitjee.com


Page # 172 ROTATIONAL DYNAMICS

17. A light string is wrapped around a cylinder of 20. A rectangular plate of mass 20 kg is suspended
mass ‘m and radius ‘R’. The string is pulled vertically from points A and B as shown. If pin B is removed
upward to prevent the centre of mass from falling determine the initial angular acceleration (in rad/
as the cylinder unwinds the string. Then length s2) of plate. (g = 10m/s2)
of the string unwound when the cylinder has
reached a speed  will be : A B
Sol.
0.15m

0.2m
Sol.

18. The moment of inertia of the pulley system


as shown in the figure is 4 kgm2. The radii of
bigger and smaller pulleys 2m and 1m respectively.
The angular acceleration of the pulley system is

1m
2m

4kg
5kg
Sol.
21. A solid homogeneous cylinder of height h and
base radius r is kept vertically on a conveyer belt
moving horizontally with an increasing velocity
v = a + bt2. If the cylinder is not allowed to slip
find the time when the cylinder is about to topple.
Sol.

19. The two small spheres each have a mass of 3


kg and are attached to the rod of negligible mass.
A torque M = 8t Nm, where t is in seconds is
applied to the rod. Find the value of time when
each sphere attains a speed of 3 m/s starting
from rest.
3kg 3kg
1m 1m

M
Sol.

Corporate Head Office : Motion Education Pvt. Ltd., 394 - Rajeev Gandhi Nagar, Kota-5
ROTATIONAL DYNAMICS Page # 173

22. A square frame made up of a wire of mass m


& length l is held in horizontal plane. It is free to
rotate about AD. If the frame is released, the
work done by gravity during the time frame
rotates through an angle of 90° is equal to
P
D

C
24. In the figure A & B are two blocks of mass 4
kg & 2 kg respectively attached to the two ends
B of a light string passing over a disc C of mass 40
Sol.
kg and radius 0.1m. The disc is free to rotate
about a fixed horizontal axes, coinciding with its
own axis. The system is released from rest and
the string does not slip over the disc. Find :

B
(i) the linear acceleration of mass B.
(ii) the number of revolutions made by the disc
at the end of 10 sec. from the start.
(iii) the tension in the string segment supporting
the block A.
Sol.

23. Three equal masses m are rigidly connected


to each other by massless rods of length l forming
an equilateral triangle, as shown above. The
assembly is to be given an angular velocity 
about an axis perpendicular to the triangle. For
fixed , the ratio of the kinetic energy of the
assembly for an axis through B compared with
that for an axis through A is equal to
m

l l

A
Bm l m
Sol.

: 0744-2209671, 08003899588 | url : www.motioniitjee.com, : info@motioniitjee.com


Page # 174 ROTATIONAL DYNAMICS

25. A mass m is attached to a pulley through a 27. A particle having mass 2 kg is moving with
cord as shown in the fig. The pulley is a solid disk
velcoity (2i  3 j )m / s . Find angular momentum of
with radius R. The cord does not slip on the disk.
The mass is released from rest at a height h from the particle about origin when it is at (1, 1, 0).
the ground and at the instant the mass reaches Sol.
the ground, the disk is rotating with angular
velocity . Find the mass of the disk.
R

m
h
Sol.

28. A uniform square plate of mass 2.0 kg and


edge 10 cm rotates about one of its diagonals
under the action of a constant torque of 0.10
N.m. Calculate the angular momentum and the
kinetic energy of the plate at the end of the fifth
second after the start.
(C) ANGULAR MOMENTUM Sol.
26. A particle having mass 2 kg is moving along
straight line 3x+ 4 y = 5 with speed 8m/s. Find
angular momentum of the particle about origin, x
and y are in meters.
Sol.

Corporate Head Office : Motion Education Pvt. Ltd., 394 - Rajeev Gandhi Nagar, Kota-5
ROTATIONAL DYNAMICS Page # 175

31. Two identical disks are positioned on a vertical


axis. The bottom disk is rotating at angular
velocity 0 and has rotational kinetic energy KE0.
The top disk is initially at rest. It is allowed to
fall, and sticks to the bottom disk. What is the
rotational kinetic energy of the system after the
collision?
29. A wheel of moment of inertia 0.500 kg-m2
and radius 20.0 cm is rotating about its axis at
an angular speed of 20.0 rad/s. It picks up a
stationary particle of mass 200 g at its edge.
Find new angular speed of the wheel. 0
Sol. Sol.

30. A uniform circular disc can rotate freely about 32. A uniform ring is rotating about vertical axis
a rigid vertical axis through its centre O. A man with angular velocity  initially. A point insect (S)
stands at rest at A on the edge due east of O. having the same mass as that of the ring starts
The mass of the disc is 22 times the mass of the walking from the lowest point P1 and finally
man. The man starts walking anticlockwise. When reaches the point P2 (as shown in figure). The
he reaches the point A after completing one final angular velocity of the ring will be equal to
rotation relative to the disc he will be :
axis of
Sol. rotation

O P2
90°

P1 S

Sol.

: 0744-2209671, 08003899588 | url : www.motioniitjee.com, : info@motioniitjee.com


Page # 176 ROTATIONAL DYNAMICS

33. A particle of mass 10 kg is moving with a 35. A cylinder rolls on a horizontal plane surface.
uniform speed of 6m/sec. in x-y plane along the If the speed of the centre is 25 m/s, what is the
line 3y = 4x+ 10 the magnitude of its angular speed of the highest point ?
Sol.
momentum about the origin in kg –m2/s is
Sol.

36. A small spherical ball is released from a point


(D) COMBINED TRANSLATIONAL + at a height h on a rough track shown in figure.
Assuming that it does not slip anywhere, find its
ROTATIONAL MOTION linear speed when it rolls on the horizontal part
34. A sphere of mass m rolls on a plane surface. of the track.
Find its kinetic energy at an instant when its
centre moves with speed v.
Sol.
h

Sol.

Corporate Head Office : Motion Education Pvt. Ltd., 394 - Rajeev Gandhi Nagar, Kota-5
ROTATIONAL DYNAMICS Page # 177

39. Two small spheres A & B respectively of mass


m & 2m are connected by a rigid rod of length  &
negligible mass. The two spheres are resting on a
horizontal, frictionless surface. When A is suddenly
given the velocity v0 as shown. Find velocities of
A & B after the rod has rotated through 180°.
A v0
37. A sphere starts rolling down an incline of
inclination . Find the speed of its centre when it 
has covered a distance .
Sol.
B
Sol.

38. A solid uniform sphere of mass m is released 40. A uniform rod of mass m and length  is struck
from rest from the rim of a hemispherical cup so at an end by a force F perpendicular to the rod
that it rolls without sliding along the surface. If for a short time interval t. Calculate
the rim of the hemisphere is kept horizotnal, find (a) the speed of the centre of mass,
the normal force exerted by the cup on the ball (b) the angular speed of the rod about the centre
when the ball reaches the bottom of the cup. of mass,
Sol. (c) the kinetic energy of the rod and
(d) the angular momentum of the rod about the
centre of mass after the force has stopped to
act. Assume that t is so small that the rod does
not appreciably change its direction while the force
acts.
Sol.

: 0744-2209671, 08003899588 | url : www.motioniitjee.com, : info@motioniitjee.com


Page # 178 ROTATIONAL DYNAMICS

42. The cylinder shown, with mass M and radius


R, has a radially dependent density. The
cylinder starts from rest and rolls without
slipping down an inclined plane of height H. At
the bottom of the plane of height H. At the
bottom of the plane its translational speed is
(8gH/7)1/2. Which of the following is the
rotational inertia of the cylinder?
R
M

Sol.
41. A hollow cylinder with inner radius R, outer
radius 2R mass M is rolling with speed of its axis v.
Its kinetic energy is

Sol.

Corporate Head Office : Motion Education Pvt. Ltd., 394 - Rajeev Gandhi Nagar, Kota-5
ROTATIONAL DYNAMICS Page # 179

1. A thin uniform rod of mass M and length L is


hinged at its upper end, and released from rest in
a horizontal position. The tension at a point
located at a distance L/3 from the hinge point,
R R/2
when the rod becomes vertical, will be

2. A rigid horizontal smooth rod AB of mass 0.75 7. A slightly loosely fit window is balanced by
kg and length 40 cm can rotate freely about a two strings which are connected to weights w/2
fixed vertical axis through its mid point O. Two each. The strings pass over the frictionless pulleys
rings each of mass 1 kg are initially at rest a as shown in the figure. The strings are tied almost
distance of 10 cm from O on either side of the at the corner of the window. The string on the
rod. The rod is set in rotation with an angular right is cut and then the window accelerates
velocity of 30 radians per second. The velocity downwards. If the coefficients of friction between
of each ring along the length of the rod in m/s the window and the side supports is  then
then they reach the ends of the rod is calculate the acceleration of the window in terms
of , a, b and g, where a is width and b is the
length of the window.
C D
A B
O

w/2 w/2
3. A straight rod AB of mass M and length L is w b
placed on a frictionless horizontal surface. A
horizontal force having constant magnitude F and a
a fixed direction starts acting at the end A. The
fixed window support
rod is initially perpendicular to the force. The initial
acceleration of end B is 8. A uniform wood door has mass m, height h,
and width w. It is hanging from two hinges
4. A wheel is made to roll without slipping, towards attached to one side; the hinges are located h/3
right, by pulling a string wrapped around a coaxial and 2h/3 from the bottom of the door. Suppose
spool as shown in figure. With what velocity the that m = 20.0 kg, h = 2.20 m, and w = 1.00 m
string should be pulled so that the centre of the and the bottom smooth hinge is not screwed into
wheel moves with a velocity of 3 m/s? the door frame. Find the forces acting on the
door.

0.3m C W
0.1m
B String
com
A Hinges h

5. A solid uniform disk of mass m rolls without


slipping down a fixed inclined plane with an
acceleration a. The frictional force on the disk 9. A hole of radius R/2 is cut from a solid sphere
due to surface of the plane is : of radius R. If the mass of the remaining plate is
M, then moment of inertia of the body about an
6. A carpet of mass ‘M’ made of inextensible axi s through O perpendi cul ar to pl ane i s
material is rolled along its length in the form of a _________.
cylinder of radius ‘R’ and is kept on a rough floor.
The carpet starts unrolling without sliding on the
floor when a negligibly small push is given to it.
R
The horizontal velocity of the axis of the cylindrical
O R/2
part of the carpet when its radius reduces to R/2
will be :

: 0744-2209671, 08003899588 | url : www.motioniitjee.com, : info@motioniitjee.com


Page # 180 ROTATIONAL DYNAMICS

10. A uniform beam of length L and mass m is 14. A spool of inner radius R and outer radius 3R
supported as shown. If the cable suddenly breaks, has a moment of inertia = MR2 about an axis
determine ; passing through its geometric centre, where M is
the mass of the spool. A thread woudn on the
(1/4)L
inner surface of the spool is pulled horizontally
with a constant force = Mg. Find the acceleration
of the point on the thread which is being pulled
A B assuming that the spool rolls purely on the floor.
L

(a) the acceleration of end B.


(b) the reaction at the pin support.

11. A thin rod AB of length a has variable mass


 x 15. A sphere of mass m and radius r is pushed
per unit length  0  1 a  where x is the distance onto the fixed horizontal surface such that it rolls
without slipping from the beginning. Determine
measured from A and 0 is a constant. the minimum speed v of its mass centre at the
(a) Find the mass M of the rod.
bottom so that it rolls completely around the loop
(b) Find the position of centre of mass of the
of radius (R + r) without leaving the track in
rod. between.
(c) Find moment of inertia of the rod about an
axis passing through A and perpendicular to AB.
Rod is freely pivoted at A and is hanging in (R+r)
equilibrium when it is struck by a horizontal impulse Sphere
of magnitude P at the point B. r V
(d) Find the angular velocity with which the rod
begins to rotate.
(e) Find minimum value of impulse P if B passes 16. Two uniform cylinders, each of mass m = 10
through a point vertically above A. kg and radius r = 150 mm, are connected by a
rough belt as shown. If the system is released
12. Two separate cylinders of masses m (= 1kg) from rest, determine
and 4m and radii R(=10cm) and 2R rotating in
r
clockwise direction with 1 = 100 rad/sec and 2
= 200 rad/sec. Now they are held in contact with
each other as in fig. Determine their angular
velocities after the slipping between the cylinders
stops.

r
(a) the velocity of the centre of cylinder A after
13. A circular disc of mass 300 gm and radius 20 it has moved through 1.2 m &
cm can rotate freely about a vertical axis passing (b) the tension in the portion of the belt
through its centre of O. A small insect of mass connecting the two cylinders.
100 gm is initially at a point A on the disc (which
is initially stationary) the insect starts walking 17. A uniform rod of mass m and length l is resting
from rest along the rim of the disc with such a on a smooth horizontal surface. A particle of mass
time varying relative velocity that the disc rotates m/2 travelling with a speed v0 hits the rod normally
in the opposite direction with a constant angular and elastically. Find final velocity of particle and
acceleration = 2 rad/s2. After some time T, the the angular velocity of the rod.
insect is back at the point A. By what angle has l/4 C
the disc rotated till now ; as seen by a stationary Rod
earth observer ? Also find the time T. (m, l)
v0
m/2
Top view

Corporate Head Office : Motion Education Pvt. Ltd., 394 - Rajeev Gandhi Nagar, Kota-5
ROTATIONAL DYNAMICS Page # 181

18. One side of a spring of initial, unstretched


length l0 = 1m, lying on a frictionless table, is
fixed, the other one is fastened to a small puck of
mass m = 0.1kg. The puck is given velocity in a
direction perpendicular to the spring, at an initial
speed v0 = 11 m/s. In the course of the motion,
the maximum elongation of the spring is l = l0/10.
What is the force constant of the spring (in SI
units) ?

v0
m
l0

19. A block X of mass 0.5 kg is held by a long


massless string on a frictionless inclined plane of
inclination 30º to the horizontal. The string is
wound on a uniform solid cylindrical drum Y of
mass 2kg and of radius 0.2 m as shown in the
figure. The drum is given an initial angular velocity
such that the block X starts moving up the plane.

Y
X

(i) Find the tension in the string during the motion


(ii) At a certain instant of time the magnitude of
the angular velocity of Y is 10 rad/sec. Calculate
the distance travelled by X from that instant of
time until it comes to rest.

: 0744-2209671, 08003899588 | url : www.motioniitjee.com, : info@motioniitjee.com


Page # 182 ROTATIONAL DYNAMICS

Exercise - IV PREVIOUS YEAR QUESTIONS

LEVEL - I JEE MAIN


1. A solid sphere, a hollow sphere and a ring are 3. A particle of mass m moves along line PC with
released from top of an inclined plane (frictionless) velocity v as shown. What is the angular
so that they slide down the plane. Then maximum momentum of the particle about O ?
acceleration down the plane is for (no rolling) [AIEEE 2002]
[AIEEE 2002]
(A) solid sphere (B) hollow sphere
(C) ring (D) All same C
Sol.
L

P r
l

O
(A) mvL (B) mvl (C) mvr (D) zero
Sol.

2. Moment of inertia of a circular wire of mass M


and radius R about its diameter is [AIEEE 2002]
4. Initial angular velocity of a circular disc of
MR 2 MR 2 mass M is 1. Then two small spheres of mass m
(A) (B) MR2 (C) 2MR2 (D) are attached gently to two diametrically opposite
2 4
points on the edge of the disc. What is the final
Sol.
angular velocity of the disc ? [AIEEE 2002]
 M  m  M  m
(A) 
  1 (B)   1
M   m 
 M   M 
(C)    1
 M  4m  1 (D) 
 M  2m 
Sol.

Corporate Head Office : Motion Education Pvt. Ltd., 394 - Rajeev Gandhi Nagar, Kota-5
ROTATIONAL DYNAMICS Page # 183

 7. A circular disc X of radius R is made from an


5. Let be the force acting on a particle having iron plate of thickness t, and another disc y of
F
 
radius 4R is made from an iron plate of thickness
position vector
r and  be the torque of this t/4. Then the relation between the moment of
force about the origin. Then [AIEEE 2003] inertia Ix and IY is [AIEEE 2003]
    (A) IY  32 I x (B) IY  16 I X
(A) r .   0 and F.   0
(C) IY  I X (D) IY  64 I X
   
(B) r .   0 and F.   0 Sol.

   
(C) r .   0 and F.   0

   
(D) r .   0 and F.   0
Sol.

8. A solid sphere is rotating in free space. If the


radius of the sphere is increased keeping mass
same, which one of the following will not be
affected ? [AIEEE 2004]
(A) Moment of inertia
(B) Angular momentum
6. A particle performing uniform circular motion (C) Angular velocity
has angular momentum L. If its angular frequency (D) Rotational kinetic energy
is doubled and its kinetic energy halved, then the Sol.
new angular momentum is [AIEEE 2003]
L
(A) (B) 2L (C) 4L (D) L/2
4
Sol.

: 0744-2209671, 08003899588 | url : www.motioniitjee.com, : info@motioniitjee.com


Page # 184 ROTATIONAL DYNAMICS

9. One solid sphere A and another hollow sphere Sol.


B are of same mass and same outer radii. Their
moment of inertia about their diameters are
respectively IA and IB such that [AIEEE 2004]
(A) I A  I B (B) I A  I B

I A dA
(C) I A  IB (D) 
I B dB
Where dA and dB are their densities.
Sol.

11. The moment of inertia of uniform semicircular


disc of mass M and radius r about a l ine
perpendicular to the plane of the disc through
the centre is [AIEEE 2005]
1 2 1
(A) Mr 2 (B) Mr 2 (C) Mr 2 (D) Mr 2
4 5 2
Sol.

10.A T shaped object with dimensions shown in



the figure, is lying on a smooth floor. A force F
is applied at the point P parallel to AB, such that
the object has only the translational motion
without rotation. Find the location of P with
respect to C. [AIEEE 2005]
l
A B
12. A thin circular ring of mass m and radius R is
rotating about its axis with a constant angular
velocity . Two objects each of mass M are
 P
F 2l attached gently to the opposite ends of a diameter
of the ring. The ring now rotates with an angular
velocity ' = [AIEEE 2006]
 (m  2M )  (m  2M )
(A) (B)
m (m  2M )
C
2 3 4 m m
(A) l (B) l (C) l (D) l (C) (D)
3 2 3 (m  M ) (m  2M )

Corporate Head Office : Motion Education Pvt. Ltd., 394 - Rajeev Gandhi Nagar, Kota-5
ROTATIONAL DYNAMICS Page # 185

Sol. Sol.

13. Four point masses, each of value m, are


placed at the corners of a square ABCD of side l. 15.A round uniform body of radius R, mass M and
The moment of inertia of this system about an moment of inertia I, rolls down (without slipping)
axis passing through A and parallel to BD is an inclined plane making an angle  with the
[AIEEE 2006] horizontal. Then its acceleration is [AIEEE 2007]
(A) 2ml 2 (B) 3ml 2 (C) 3ml 2 (D) ml 2 g sin  g sin 
(A) (B)
Sol. 1  I / MR 2 1  MR 2 / I
g sin  g sin 
(C) (D)
1  I / MR 2 1  MR 2 / I
Sol.

14.Angular momentum of the particle rotating with


a central force is constant due to [AIEEE 2007]
(A) constant force
(B) constant linear momentum
(C) zero torque
(D) constant torque

: 0744-2209671, 08003899588 | url : www.motioniitjee.com, : info@motioniitjee.com


Page # 186 ROTATIONAL DYNAMICS

16.For the given uniform square lamina ABCD, 18.A thin uniform rod of length l and mass m is
whose centre is O [AIEEE 2007] swinging freely about a horizontal axis passing
through its end. Its maximum angular speed is .
F C Its centre of mass rises to maximum height of
D 
[AIEEE 2009]

1 l 2 2 1 l
 (A) (B)
O 3 g 6 g

 1 l 2 2 1 l 2 2
A E B (C) (D)
2 g 6 g
(A) 2 I AC  I EF (B) I AD  3I EF Sol.

(C) I AD  4 I EF (D) I AD  2 I EF
Sol.

19.A pulley of radius 2 m is rotated about its axis


by a force F = (20t – 5t2) N (where t is measured
17. Consider a uniform square plate of side a and in seconds) applied tangentially. It the moment
mass m. The moment of inertia of this plate about of inertia of the pulley about its axis of rotation is
an axis perpendicular to its plane and passing 10 kg-m2 the number of rotations made by the
through one of its corners is [AIEEE 2008] pulley before its direction of motion if reserved,
5 1 7 2 is [AIEEE 2011]
(A) ma 2 (B) ma 2 (C) ma 2 (D) ma 2 (A) more than 3 but less than 6
6 12 12 3 (B) more than 6 but less than 9
Sol. (C) more than 9
(D) less than 3
Sol.

Corporate Head Office : Motion Education Pvt. Ltd., 394 - Rajeev Gandhi Nagar, Kota-5
ROTATIONAL DYNAMICS Page # 187

20.A thin horizontal circular disc is rotating about 21. A hoop of radius r and mass m rotating with
a vertical axis passing through its centre. An insect an angular velocity 0 is placed on a rough
is at rest at a point near the rim of the disc. The horizontal surface. The initial velocity of the
insect now moves along a diameter of the disc to centre of the hoop is zero. What will be the
reach its other end. During the journey of the velocity of the centre of the hoop when it cases
insect, the angular speed of the disc. to slip ? [JEE Mains 2013]
[AIEEE 2011]
r0 r0 r0
(A) continuously decreases (1) (2) r0 (3) (4)
(B) continuously increases 2 4 3
(C) first increases and then decreases Sol.
(D) remains unchanged
Sol.

: 0744-2209671, 08003899588 | url : www.motioniitjee.com, : info@motioniitjee.com


Page # 188 ROTATIONAL DYNAMICS

LEVEL - II JEE ADVANCED


1. Three particles A, B and C, each of mass m, Sol.
are connected to each other by three massless
rigid rods to form a rigid, equilateral triangular
body of side l. This body is placed on a horizontal
frictionless table (x-y plane) and is hinged to it
at the point A so that it can move without friction
about the vertical axis through A (see figure).
The body is set into rotational motion on the
table about A with a constant angular velocity .
y

A
x
3. Two particles each of mass M are connected

by a massless rod of length l. The rod is lying on
the smooth surface. If one of the particle is
F given an impulse MV as shown in the figure then
angular velocity of the rod would be
B l C
[JEE’(Scr) 2002]
(a) Find the magnitude of the horizontal force
exerted by the hinge on the body
(b) At time T, when the side BC is parallel to the
x-axis, a force F is applied on B along BC (as
shown). Obtain the x-component and the y-
component of the force exterted by the hinge on
the body, immediately after time T. Mv M
Sol. (A) v/l (B) 2v/l

(C) v/2 l (D) none


[JEE’(Scr) 2003]
Sol.

4. A disc is rolling (without slipping) on a horizontal


surface. C is its center and Q and P are two
points equidistant from C. Let Vp, VQ and VC be
the magnitude of velocities of points P, Q and C
respectively, then
Q
C
P
2. A particle is moving in a horizontal uniform
circular motion. The angular momentum of the
particle is conserved about the point :
(A) VQ > VC > VP (B) VQ < VC < VP
[JEE’(Scr) 2003]
(A) Centre of the circle (B) Outside the circle 1
(C) VQ = Vp, VC  VP (D) VQ < VC > VP
(C) Inside the circle 2
(D) Point on circumference [JEE’(Scr) 2004]

Corporate Head Office : Motion Education Pvt. Ltd., 394 - Rajeev Gandhi Nagar, Kota-5
ROTATIONAL DYNAMICS Page # 189

Sol. Sol.

7. A disc has mass 9m. A hole of radius R/3 is cut


5. A child is standing with folded hands at the from it as shown in the figure. The moment of inertia
centre of a platform rotating about its central of remaining part about an axis passing through
axis. The kinetic energy of the system is K. The the centre ‘O’ of the disc and perpendicular to the
child now stretches his arms so that the moment plane of the disc is :
of inertia of the system doubles. The kinetic
energy of the system now is
[JEE’(Scr) 2004] R/3
2R/3
(A) 2K (B) K/2
(C) K/4 (D) 4K O
Sol. R

(A) 8 mR2 (B) 4 mR2


40 37
(C) mR2 (D) mR2
9 9
[JEE’(Scr) 2005]
Sol.

6. A block of mass m is held fixed against a wall


by a applying a hor izontal force F. Which of the
following option is incorrect :

F
2a
a 8. A particle moves in circular path with decreasing
speed. Which of the following is correct

2a (A) L is constant
(A) friction force = mg 
(B) only direction of L is constant
(B) F will not produce torque 
(C) acceleration a is towards the centre
(C) normal will not produce torque
(D) it will move in a spiral and finally reach the
(D) normal reaction = F centre
[JEE’(Scr) 2005] [JEE’(Scr) 2005]

: 0744-2209671, 08003899588 | url : www.motioniitjee.com, : info@motioniitjee.com


Page # 190 ROTATIONAL DYNAMICS

Sol. 11. Two identical ladders, each of mass M and


length L are resting on the rough horizontal surface
as shown in the figure. A block of mass m hangs
from P. If the system is in equilibrium, find the
magnitude and the direction of frictional force at
A and B. [JEE’ 2005]
P

L m

 
9. A wooden log of mass M and length L is hinged A B
by a frictionless nail at O. A bullet of mass m Sol.
strikes with velocity v and sticks to it. Find angular
velocity of the system immediately after the
collision about O.
O

L
M

m
v
[JEE’ 2005]
Sol.

12. A solid sphere of mass M, radius R and having


moment of inertia about an axis passing through
the centre of mass as I, is recast into a disc of
thickness t, whose moment of inertia about an
axis passing through its edge and perpendicular
to its plane remains I. Then, radius of the disc
will be [JEE’ 2006]
(A) 2R / 15 (B) R 2 / 15

(C) 4 R / 15 (D) R/4


Sol.

10. A cylinder of mass m and radius R rolls down


an inclined plane of inclination . Calculate the
linear acceleration of the axis of cylinder.
[JEE’ 2005]
Sol.

13. A solid cylinder of mass m and radius r is


rolling on a rough inclined plane of inclination .
The coefficient of friction between the cylinder
and incline is . Then [JEE’ 2006]
(A) frictional force is always mg cos 
(B) friction is a dissipative force
(C) by decreasing , frictional force decreases
(D) friction opposes translation and supports
rotation

Corporate Head Office : Motion Education Pvt. Ltd., 394 - Rajeev Gandhi Nagar, Kota-5
ROTATIONAL DYNAMICS Page # 191

Sol. b

It is given n = 100, M = 3 kg, m = 0.01 kg; b = 2


m, a = 1m; g = 10 m/s2.
Sol.

14. A ball moves over a fixed track as shown in


the figure. From A to B the ball rolls without
slipping. Surface BC is frictionless. KA, KB and KC
are kinetic energies of the ball at A, B and C,
respe0ctively. Then [JEE’ 2006]

A C
hA hC
B Paragraph Q.16 to Q.18 (3 questions)
Two discs A and B are mounted coaxially on a
vertical axle. The discs have moments of inertia I
(A) hA > hC ; KB > KC (B) hA > hC ; KC > KA and 2I respectively about the common axis. Disc
(C) hA = hC ; KB = KC (D) hA < hC ; KB > KC A is imparted an initial angular velocity 2 using
Sol. the entire potential energy of a spring compressed
by a distance x1. Disc B is imparted an angular
velocity  by a spring having the same spring
constant and compressed by a distance x2. Both
the discs rotate in the clockwise direction.
16. The ratio x1/x2 is [JEE’ 2007]
(A) 2 (B) 1/2
(C)  2 (D) 1/ 2
Sol.

15. There is a rectangular plate of mass M kg of


dimensions (a × b). The plate is held in horizontal 17. When disc B is brought in contact with disc
position by striking n small balls each of mass m A, they acquire a common angular velocity in time
per unit area per unit time. These are striking in t. The average frictional torque on one disc by
the shaded half region of the plate. The balls are the other during this period is [JEE’ 2007]
colliding elastically with velocity v. What is v ? (A) 2I/(3t) (B) 9I/(2t)
[JEE’ 2006] (C) 9I/(4t) (D) 3I/(2t)

: 0744-2209671, 08003899588 | url : www.motioniitjee.com, : info@motioniitjee.com


Page # 192 ROTATIONAL DYNAMICS

Sol. 20. STATEMENT-1 If there is no external torque


on a body about its center of mass, then the
velocity of the center of mass remains constant
because
STATEMENT-2
The linear momentum of an isolated system
remains constant.
(A) Statement-1 is True, Statement-2 is True;
Statement-2 is a correct expl anation for
Statement-1
(B) Statement-1 is True, Statement-2 is True;
Statement-2 is NOT a correct explanation for
Statement-1
(C) Statement-1 is True, Statement-2 is False
(D) Statement-1 is False, Statement-2 is True
[JEE 2007]
18. The loss of kinetic energy during the above Sol.
process is [JEE’ 2007]
(A) I2 / 2 (B) I2 /3
(C) I2 /4 (D) I2 / 6
Sol.

19. A small object of uniform density rolls up a


curved surface with an initial velocity v. It reaches
up to a maximum height of 3v2 / (4g) with respect
to the initial position. The object is [JEE’ 2007]
21. STATEMENT-1
Two cylinders, one hollow (metal) and the other
solid (wood) with the same mass and identical
v dimensions are simultaneously allowed to roll with-
out slipping down an inclined plane from the same
(A) ring (B) solid sphere height. The hollow cylinder will reach the bottom
(C) hollow sphere (D) disc of the inclined plane first.
Sol. STATEMENT-2
By the principle of conservation of energy, the
total kinetic energies of both the cylinders are
identical when they reach the bottom of the in-
cline.
(A) STATEMENT-1 is True, STATEMENT-2 is True;
STATEMENT-2 is a correct explanation for STATE-
MENT-1
(B) STATEMENT-1 is True, STATEMENT-2 is True;
STATEMENT-2 is NOT a correct explanation for
STATEMENT-1
(C) STATEMENT-1 is True, STATEMENT-2 is False
(D) STATEMENT-1 is False, STATEMENT-2 is True
[JEE-2008]

Corporate Head Office : Motion Education Pvt. Ltd., 394 - Rajeev Gandhi Nagar, Kota-5
ROTATIONAL DYNAMICS Page # 193

Sol. Sol.

22. If the resultant of all the external forces acting


on a system of particles is zero, then from an
inertial frame, one can surely say that
[JEE 2009]
(A) linear momentum of the system does not 24. A boy is pushing a ring of mass 2 kg and
change in time radius 0.5 m with a stick as shown in the figure.
(B) kinetic energy of the system does not change
The stick applies a force of 2 N on the ring and
in time
(C) angular momentum of the system does not rolls it without slipping with an acceleration of
change in time 0.3 m/s2. The coefficient of friction between the
(D) potential energy of the system does not ground and ring is large enough that rolling always
change in time occurs and the coefficient of friction between
Sol. the stick and the ring is (P/10). The value of P
is? stick [JEE 2011]

Ground

Sol.

23. A sphere is rolling without slipping on a fixed


horizontal plane surface. In the figure A is the
point of contact, B is the centre of the sphere
and C is its topmost point Then, [JEE 2009]
C

A
       
(A) VC – VA  2( VB – VC ) (B) VC – VB  VB – VA
      
(C) | VC – VA |  2| VB – VC | (D) | VC – VA |  4| VB |

: 0744-2209671, 08003899588 | url : www.motioniitjee.com, : info@motioniitjee.com


Page # 194 ROTATIONAL DYNAMICS

25. A thin ring of mass 2 kg and radius 0.5 m is Z


rolling without slipping on a horizontal plane with
velocity 1 m/s. A small ball of mass 0.1 kg, moving
with velocity 20 m/s in the opposite direction,
hits the ring at a height of 0.75 m and goes
O
vertically up with velocity 10 m/s. Immediately
after the collision [JEE 2011]
10m/s
20m/s

(A) (B)
0.75m 0 t 0 t
1m/s
T T

(A) The ring has pure rotation about its stationary CM


(B) The ring comes to a complete stop
(C) Friction between the ring and the ground is (C) (D)
to the left 0 t 0 t
T T
(D) There is no friction between the ring and the ground
[JEE 2012]
Sol.
Sol.

27. A small mass m is attached to a massless


string whose other end is fixed at P as shown in
the figure. The mass is undergoing circular motion
in the x-y plane with centre at O and constant
angular speed .If the angular momentum of the
system, calculated about O and P are denoted
 
by L 0 and LP respectively, then.
z

P
26. A thin uniform rod, pivoted at O is rotating in
the horizontal plane with constant angular speed
, as shown in the figure. At time t = 0, small m
O
insect starts from O and moves with constant
speed  with respect to the rod towards the other
end. it reaches the end of the rod at t = T and  
stops. The angular speed of the system remains (A) L 0 and LP do not vary with time
 
 (B) L 0 varies with time while LP remains constant
 throughout. The magnitude of the torque    on  
(C) L 0 remains constant while LP varies with time
the system about O, as a function of time is best  
represented by which plot? (D) L 0 and LP both vary with time.
[JEE 2012]

Corporate Head Office : Motion Education Pvt. Ltd., 394 - Rajeev Gandhi Nagar, Kota-5
ROTATIONAL DYNAMICS Page # 195

Sol. 29. Consider a disc rotating in the horizontal plane


with a constant angular speed  about its centre
O. The disc has a shaded region on one side of
the diameter and an unshaded region on the other
side as shown in the figure. When the disc is in
the orientation as shown, two pebbles P and Q
are simultaneously projected at an angle towards
R. The velocity of projection is in the y-z plane
and is same for both pebbles with respect to the
disc. Assume that (i) they land back on the disc
before the disc has completed 1/8 rotation, (ii)
their range is less than half the disc radius, and
(iii)  remains constant throughout. Then

y R

x Q
28. A lamina is made by removing a small disc of O
diameter 2R from a bigger disc of uniform mass P
density and radius 2R, as shown in the figure. (A) P lands in the shaded region and Q in the
The moment of inertia of this lamina about axes unshaded region
passing through O and P is Io and IP, respec- (B) P lands in the unshaded region and Q in the
tively. Both these axes are perpendicular to the shaded region
IP (C) Both P and Q land in the unshaded region
plane of the lamina. The ratio I to the nearest
o (D) Both P and Q land in the shaded region
integer is [JEE 2012]
Sol.

[JEE 2012]
Sol.

: 0744-2209671, 08003899588 | url : www.motioniitjee.com, : info@motioniitjee.com


Page # 196 ROTATIONAL DYNAMICS

Paragraph for Question Nos. 30 to 31 Sol.


The general motion of a rigid body can be
considered to be a combination of (i) a motion of
its centre of mass about an axis, and (ii) its motion
about an instantaneous axis passing through the
centre of mass. These axes need not be
stationary. Consider, for example, a thin uniform
disc welded (rigidly fixed) horizontally at its rim
to a massless stick, as shown in the figure. When
the disc-stick system is rotated about the origin
on a horizontal frictionless plane with angular
speed , the motion at any instant can be taken
as a combination of (i) a rotation of the centre
of mass of the disc about the z-axis, and (ii) a
rotation of the disc through an instantaneous
vertical axis passing through its centre of mass
(as is seen from the changed orientation of points
P and Q). Both these motions have the same 31. Which of the following statements regarding
angular speed in this case. the angular speed about the instantaneous axis
(passing through the centre of mass) is correct
(A) It is 2 for both the cases.


(B) It is  for case (a); and for case (b).
2
Now consider two similar systems as shown in (C) It is for case (a); and 2 for case (b).
the figure: Case (a) the disc with its face vertical
(D) It is  for both the cases.
and parallel to x-z plane; case (b) the disc with
[JEE 2012]
its face making an angle of 45o with x-y plane
Sol.
and its horizontal diameter parallel to x-axis. In
both the cases, the disc is welded at point P,
and the systems are rotated with constant
angular speed about the z-axis.

Case (a) Case (b)

30. Which of the following statements about the


instantaneous axis (passing through the centre
of mass) is correct ?
(A) It is vertical for both the cases (a) and (b).
(B) It is vertical for case (a); and is at 45o to the 32. The figure shows a system consisting of (i) a
x-z plane and lies in the plane of the disc for ring of outer radius 3R rolling clockwise without
case (b). slipping on a horizontal surface with angular speed
(C) It is horizontal for case (a); and is at 45o to and (ii) an inner disc of radius 2R rotating anti-
the x-z plane and is normal to the plane of the clockwise with angular speed 2. The ring and
disc for case (b). disc are separated by frictionless ball bearings.
(D) It is vertical for case (a); and is at 45o to the The system is in the x-z plane. The point P on
x-z plane and is normal to the plane of the disc the inner disc is at a distance R from the origin,
for case (b). where OP makes an angle of 30 o with the
[JEE 2012] horizontal. Then with respect to the horizontal
surface.

Corporate Head Office : Motion Education Pvt. Ltd., 394 - Rajeev Gandhi Nagar, Kota-5
ROTATIONAL DYNAMICS Page # 197

Sol.

(A) the point O has a linear velocity 3R î 


(B) the poi nt P has a l i ne ar v el oc i t y

11 ˆ 3 ˆ
R i  Rk
4 4
(C ) the poi nt P has a l i ne ar v el oc i t y

13 3
R î  Rk̂
4 4
34. A uniform circular disc of mass 50 kg and
(D ) the poi nt P has a l i ne ar v el oc i t y
radius 0.4 m is rotating with an angular velocity
  of 10 rad s-1 about its own axis, which is vertical.
 3  3 R î  1 Rk̂ Two uniform circular rings, each of mass 6.25 kg
 4  4
 and radius 0.2 m, are gently placed symmetrically
[JEE 2012] on the disc in such a manner that they are
touching each other along the axis of the disc
Sol.
and are horizontal. Assume that the friction is
large enough such that the rings are at rest
relative to the disc and the system rotates about
the original axis. The new angular velocity (in rad
s-1) of the system is : [JEE 2013]
Sol.

33. Two solid cylinders P and Q of same mass and


same radius start rolling down a fixed inclined
plane from the same height at the same time.
Cylinder P has most of its mass concentrated
near its surface, while Q has most of its mass
concentrated near the axis. Which statement(s)
is(are) correct?
(A) Both cylinders P and Q reach the ground at
the same time.
(B) Cylinder P has larger linear acceleration than
cylinder Q.
(C) Both cylinders reach the ground with same
translational kinetic energy.
(D) Cylinder Q reaches the ground with larger
angular speed.
[JEE 2012]

: 0744-2209671, 08003899588 | url : www.motioniitjee.com, : info@motioniitjee.com


ANSWER KEY

Exercise - I OBJECTIVE PROBLEMS (JEE MAIN)

1. D 2. B 3. A 4. D 5. A 6. B

7. A 8. C 9. A 10. C 11. D 12. B

13. D 14. B 15. C 16. D 17. B 18. B

19. A 20. D 21. D 22. D 23. A 24. D

25. A 26. D 27. B 28. C 29. A 30. B

31. B 32. A 33. C 34. C 35. D 36. C

37. B 38. B 39. B 40. C 41. B 42. D

43. B 44. B 45. A 46. D 47. D 48. B

49. A 50. C 51. C 52. D 53. D 54. C

55. C 56. C 57. A 58. D 59. C 60. A

61. D

Exercise - II

1. B 2. C 3. C 4. A 5. B 6. C

7. A 8. C 9. C 10. ABCD 11. B 12. B

13. A 14. C 15. D 16. BC 17. AD 18. BCD

19. ABD 20. ABC 21. ACD 22. C 23. B 24. ACD

25. B 26. C 27. A 28. D 29. B 30. B

31. C 32. B 33. D 34. A 35. C 36. B

37. ACD 38. C 39. BCD 40. A 41. C 42. BC

43. AC 44. BC 45. ABCD 46. BD 47. ABC 48. ABC

49. C 50. BCD 51. ACD 52. ABD 53. B 54. A

55. B 56. C 57. A 58. C 59. D 60. C

61. B 62. B 63. B 64. D 65. D 66. A

67. C
Exercise - III (JEE ADVANCED)

2
MR 2  4R  14mr 2 Ml 2
1. – M  2. 3. 2r 4. 5. 2mR2
2  3  5 2

6. w  sin , when the bob is at the lowest point 7. –14 i  10 j – 9k

mg 3Ft 2
8. P= cot  9.
2 2m

3g 13mg 3 3 3 3
10. (a) (cw ) (b) N  16 , F   16  mg  (c)
4L   16

2g
11.  12. w = 5 rad/s 13. at a distance a / 2 3 from the centre down the plane.
5

100 R2 2
14. 12 gm 15. N 16. 13 W / 12 17. 18. 2.1 rad/s2 19. 15
. 2 sec
3 4g

mgl
20. 48 21. gr/bh 22. 23. 2 24. (i) 10/13 m/s2, (ii) 5000/26, (iii) 480/13 N
8

 2gh 
25. M = 2m 2 2 – 1 26.16 kg m2/s 27. 2k kg m2 / s 28. 0.5 kg – m2/s, 75 J
R  


29. 19.7 rad/s 30. 60° east of south, 30° south of east. 31. (1/2)KE0 32.
3

7 10gh 10 17
33. 120 34. mv 2 35. 50m/s 36. 37. gsin 38. mg
10 7 7 7

v0 2v Ft 6Ft 2F 2 t 2 Ft 13 3
39. (), 0 () 40. (a) (b) (c) (d) 41. Mv 2 42. MR 2
3 3 m m m 2 16 4
ROTATIONAL DYNAMICS Page # 200

14gR
1. 2mg 2. 3 3. 2F/M 4. 2m/s 5. 1/2 ma 6. v =
3

 b – a  3mgw 9g 4mg
7. a    g 8. Fdlx = and Fduy = mg 9. 57/140 MR2 10. (a)  (b) 
 3b  a  2h 7 7

3 0 a 5a 7a 3  0 18P M
11. (a) (b) , (c) , (d) , (e) 70ag 12. 300 rad/sec, 150 rad/sec
2 9 12 7Ma 9

27 3 200
13. t = 2 / 5 sec, q = 4p/5 rad 14. 16 m/s2 15. v = gR 16. (a) 4 m / s, (b) N
7 7 7

1
17. – v0 18. 210 19. 1.63 N, 1.224 m
15

Exercise - IV PREVIOUS YEAR QUESTIONS

LEVEL - I JEE MAIN

1. D 2. A 3. B 4. C 5. D 6. A

7. D 8. B 9. C 10. C 11. D 12. D

13. C 14. C 15. A 16. C 17. D 18. D

19. A 20. C 21. A

LEVEL - II JEE ADVANCED

2 2
1. (a) 3 m  l, (b) Fx = F/4, Fy = 3 m l

2. A 3. A 4. A 5. B 6. C 7. B

3 mv 2g sin 
8. B 9.   10. aaxis 
(3m  M) L 3
cot 
11. f  (M  m) g 12. A 13. C,D 14. A,B 15. 10m/s
2
16. C 17. A 18. B 19. D 20. D

21. D 22. A 23. B,C

24. 0004 25. A 26. B 27. C 28. 0003 29. C

30. A 31. D 32. A,B 33. D 34. 0008

394,50 - Rajeev Gandhi Nagar Kota, Ph. No. : 93141-87482, 0744-2209671


IVRS No : 0744-2439051, 52, 53, www. motioniitjee.com, info@motioniitjee.com

You might also like